1535 Review

¡Supera tus tareas y exámenes ahora con Quizwiz!

Coloring book Small metal cars Simple card game

A 6-year-old child is waiting with a family member in the pediatric clinic for a well-child visit. What are the most appropriate play activities for the office nurse to offer the child? Select all that apply.

B) Nociceptive pain Nociceptive pain refers to the normal functioning of physiological systems that leads to the perception of noxious stimuli (tissue injury) as being painful. Patients describe this type of pain as dull or aching, and it is poorly localized. Neuropathic pain is described as shooting, tingling, burning, or numbness that is constant in the extremities, as in diabetic neuropathy. Chronic pain lasts longer than 30 days and is characterized by a disease affecting brain structure and function, such as chronic headaches or open wounds. Mixed pain syndromes are caused by different pathophysiological mechanisms such as a combination of neuropathic and nociceptive pain; this occurs in syndromes such as sciatica, spinal cord injuries, and cervical or lumbar spinal stenosis.

A 65-year-old woman has fallen while sweeping her driveway, sustaining a tissue injury. She describes her condition as an aching, throbbing back. Which type of pain are these complaints most indicative of? A) Neuropathic pain B) Nociceptive pain C) Chronic pain D) Mixed pain syndrome

A) Calls the health care provider and questions the order Fentanyl patches are not indicated for acute pain. They are indicated for patients with chronic pain who are opioid tolerant.

A health care provider writes the following order for an opioid-naive patient who returned from the operating room following a total hip replacement: "Fentanyl patch 100 mcg, change every 3 days." On the basis of this order, the nurse takes the following action: A) Calls the health care provider and questions the order B) Applies the patch the third postoperative day C) Applies the patch as soon as the patient reports pain D) Places the patch as close to the hip dressing as possible

A) Difficulty paying his bills E) Family practice of not routinely seeing a health care provider External factors impacting health practices include family beliefs and economic impact. The way that patients' families use health care services generally affects their health practices. Their perceptions of the seriousness of diseases and their history of preventive care behaviors (or lack of them) influence how patients think about health. Economic variables may affect a patient's level of health by increasing the risk for disease and influencing how or at what point the patient enters the health care system.

A male patient has been laid off from his construction job and has many unpaid bills. He is going through a divorce from his marriage of 15 years and has been seeing his pastor to help him through this difficult time. He does not have a primary health care provider because he has never really been sick and his parents never took him to a physician when he was a child. Which external variables influence the patient's health practices? (Select all that apply.) A) Difficulty paying his bills B) Seeing his pastor as a means of support C) Age of patient (46 years) D) Stress from the divorce and the loss of a job E) Family practice of not routinely seeing a health care provider

B) The time interval Long-acting or sustained-release opioids are dosed on a scheduled basis, not prn, to provide a base of continuous opioid analgesia.

A new medical resident writes an order for oxycodone CR (Oxy Contin) 10 mg PO q2h prn. Which part of the order does the nurse question? A) The drug B) The time interval C) The dose D) The route

Chicken broth Enriched whole milk Red meats, such as beef Liver and other glandular organ meats

A nurse asks a client with ischemic heart disease to identify the foods that are most important to restrict. Which food choices by the client indicate effective learning? Select all that apply.

A) BMI of 17 C) Weight loss of 6% since last month's visit F) hemoglobin level of 8.2 g/dL A BMI of 18.5 to 24.9 is normal, and this patient's BMI is below normal; a major weight loss is defined as more than a 2% weight change over 1 week; and the expected hemoglobin level for a man is 14 to 18 g/dL. The patient's values may also indicate dehydration. The expected level for prealbumin is 15 to 36 mg/dL. A hematocrit level of 50% is within normal limits.

A nurse in a home setting is assessing a 79-year-old male patient's risk for malnutrition. The nurse suspects malnutrition when reviewing which laboratory results? (Select all that apply) A) Body mass index (BMI) of 17 B) Waist-to-hip ratio of 1.0 C) Weight loss of 6% since last month's visit D) Prealbumin level of 16 mg/dL E) Hematocrit level of 50% F) Hemoglobin level of 8.2 g/dL

A) Physiological The nurse's actions address the basic physiological needs of nutrition, fluids, elimination, and oxygen. According to Maslow, basic needs must be met before meeting higher level needs.

A nurse is conducting a home visit with an older-adult couple. While in the home the nurse weighs each individual and reviews the 3-day food diary with them. She also checks their blood pressure and encourages them to increase their fluids and activity levels to help with their voiced concern about constipation. The nurse is addressing which level of need according to Maslow? A) Physiological B) Safety and security C) Love and belonging D) Self-actualization

B) A learning objective. A learning objective describes the behavior(s) the learner will exhibit as a result of successful instruction.

A nurse is planning a teaching session about healthy nutrition with a group of children who are in first grade. The nurse determines that after the teaching session the children will be able to name three examples of foods that are fruits. This is an example of: A) A teaching plan. B) A learning objective. C) Reinforcement of content. D) Enhancing the children's self-efficacy.

A) Primary prevention Primary prevention is aimed at health promotion and includes health education programs, immunizations, and physical and nutritional fitness activities. It can be provided to an individual and includes activities that focus on maintaining or improving the general health of individuals, families, and communities. It also includes specific protection such as hearing protection in occupational settings.

A nurse is presenting a program to workers in a factory covering safety topics, including the wearing of hearing protectors when workers are in the factory. Which level of prevention is the nurse practicing? A) Primary prevention B) Secondary prevention C) Tertiary prevention D) Quaternary prevention

D) Health promotion and illness prevention Health promotion and illness prevention are the focus when nurses provide information to help patients improve their health and avoid illness.

A nurse is teaching a group of young college-age women the importance of using sunscreen when going out in the sun. Which type of content is the nurse providing? A) Simulation B) Restoring health C) Coping with impaired function D) Health promotion and illness prevention

A) How to use an inhaler during an asthma attack It is important to start with essential lifesaving information when teaching people because they usually remember what you tell them first.

A nurse needs to teach a young woman newly diagnosed with asthma how to manage her disease. Which of the following topics does the nurse teach first? A) How to use an inhaler during an asthma attack B) The need to avoid people who smoke to prevent asthma attacks C) Where to purchase a medical alert bracelet that says she has asthma D) The importance of maintaining a healthy diet and exercising regularly

B) "Yes, walking is great exercise. Do you think you could go for a 5-minute walk next week?" The patient's response indicates she is in the contemplative state, possibly intending to make a behavior change within the next 6 months. The nurse's statement reinforces the behavior and provides a specific goal for the patient to begin her walking plan.

A patient comes to the local health clinic and states: "I've noticed how many people are out walking in my neighborhood. Is walking good for you?" What is the best response to help the patient through the stages of change for exercise? A) "Walking is OK. I really think running is better." B) "Yes, walking is great exercise. Do you think you could go for a 5-minute walk next week?" C) "Yes, I want you to begin walking. Walk for 30 minutes every day and start to eat more fruits and vegetables." D) "They probably aren't walking fast enough or far enough. You need to spend at least 45 minutes if you are going to do any good."

C) Tertiary prevention Tertiary prevention involves minimizing the effects of long-term disease or disability by interventions directed at preventing complications and deterioration following surgery. Tertiary prevention activities are directed at rehabilitation rather than diagnosis and treatment. Care at this level aims to help patients achieve as high a level of functioning as possible, despite the limitations caused by illness or impairment. This level of care is called preventive care because it involves preventing further disability or reduced functioning.

A patient had surgery for a total knee replacement a week ago and is currently participating in daily physical rehabilitation sessions at the surgeon's office. In what level of prevention is the patient participating? A) Primary prevention B) Secondary prevention C) Tertiary prevention D) Quaternary prevention

A) Transcutaneous electrical nerve stimulation (TENS) C) Provide back massage D) Reposition the patient Nonpharmacological therapies may provide comfort for the patient. It is much too early to consider possible addiction. Naloxone is not appropriate at this time because the patient does not show signs of oversedation or respiratory depression.

A patient has returned from the operating room, recovering from repair of a fractured elbow, and states that her pain level is 6 on a 0-to-10 pain scale. She received a dose of hydromorphone just 15 minutes ago. Which interventions may be beneficial for this patient at this time? Select all that apply. A) Transcutaneous electrical nerve stimulation (TENS) B) Administer naloxone (Narcan) 2 mg intravenously C) Provide back massage D) Reposition the patient E) Withhold any pain medication and tell the patient that she is at risk for addiction

C) Stool softeners Constipation is a common opioid-related side effect, and patients do not become tolerant to it.

A patient is being discharged home on an around-the-clock (ATC) opioid for chronic back pain. Because of this order, the nurse anticipates an order for which class of medication? A) Opioid antagonists B) Antiemetics C) Stool softeners D) Muscle relaxants

B) 2, 5, 1, 4, 6, 3 This is the appropriate order for administering PCA.

A patient is prescribed morphine patient-controlled analgesia (PCA). What is the correct order for administering PCA? 1. Program computerized PCA pump to deliver prescribed medication dose and lockout interval. 2. Check label of medication 3 times: when removed from storage, when brought to bedside, when preparing for assembly. 3. Administer loading dose of analgesia as prescribed. 4. Attach drug reservoir to infusion device, prime tubing, and attach needleless adapter to end of tubing. 5. Identify patient using two identifiers. 6. Insert and secure needleless adapter into injection port nearest patient. A) 1, 2, 4, 1, 6, 3 B) 2, 5, 1, 4, 6, 3 C) 1, 2, 5, 4, 6, 3 D) 2, 5, 4, 1, 3, 6

B) Expressing the importance of learning the skill correctly Patients are ready to learn when they understand the importance of learning and are motivated to learn.

A patient needs to learn how to administer a subcutaneous injection. Which of the following reflects that the patient is ready to learn? A) Describing difficulties a family member has had in taking insulin B) Expressing the importance of learning the skill correctly C) Being able to see and understand the markings on the syringe D) Having the dexterity needed to prepare and inject the medication

D) Psychomotor domain Using a walker requires the integration of mental and muscular activity.

A patient needs to learn to use a walker. Which domain is required for learning this skill? A) Affective domain B) Cognitive domain C) Attentional domain D) Psychomotor domain

C) Provide only the information that the patient needs to go home This patient is in denial; thus it is appropriate to give her only information that is needed immediately.

A patient newly diagnosed with cervical cancer is going home. The patient is avoiding discussion of her illness and postoperative orders. What is the nurse's best plan in teaching this patient? A) Teach the patient's spouse B) Focus on knowledge the patient will need in a few weeks C) Provide only the information that the patient needs to go home D) Convince the patient that learning about her health is necessary

A) Patient's self-report Patient's self-report of pain. Sleep is not an indicator of pain intensity. Unless a patient is stimulated, it is difficult to distinguish sleep from sedation, which may occur as a side effect of the opioid. Patients in pain sometimes sleep from exhaustion.

A patient rates his pain as a 6 on a scale of 0 to 10, with 0 being no pain and 10 being the worst pain. The patient's wife says that he can't be in that much pain since he has been sleeping for 30 minutes. Which is the most accurate resource for assessing the pain? A) Patient's self-report B) Behaviors C) Surrogate (wife) report D) Vital sign changes

C) Preparation The individual is in the preparation stage, making small changes toward preparation for a change in the next month. In this stage the patient believes that the advantages outweigh the disadvantages.

A patient registered at the local fitness center and purchased a pair of exercise shoes. The patient is in what stage of behavioral change? A) Precontemplation B) Contemplation C) Preparation D) Action

B) Demonstration Demonstration is used to help patients learn psychomotor skills.

A patient who is hospitalized has just been diagnosed with diabetes. He is going to need to learn how to give himself injections. Which teaching method does the nurse use? A) Simulation B) Demonstration C) Group instruction D) One-on-one discussion

B) Request to have the order changed to around the clock (ATC) for the first 48 hours. The patient can be expected to have acute pain related to the G-tube insertion; in addition, she has a history of chronic pain. Her pain should be treated with ATC medication to match the timing of her pain.

A patient with a 3-day history of a stroke that left her confused and unable to communicate returns from interventional radiology following placement of a gastrostomy tube. The patient has been taking hydrocodone/APAP 5/325 up to four tablets/day before her stroke for arthritic pain. The health care provider's order reads as follows: "Hydrocodone/APAP 5/325 1 tab, per gastrostomy tube, q4h, prn." Which action by the nurse is most appropriate? A) No action is required by the nurse because the order is appropriate. B) Request to have the order changed to around the clock (ATC) for the first 48 hours. C) Ask for a change of medication to meperidine (Demerol) 50 mg IVP, q3 hours, prn. D) Begin the hydrocodone/APAP when the patient shows nonverbal symptoms of pain.

A) Telling approach The telling approach is most appropriate when preparing a patient for an emergency procedure.

A patient with chest pain is having an emergency cardiac catheterization. Which teaching approach does the nurse use in this situation? A) Telling approach B) Selling approach C) Entrusting approach D) Participating approach

D) Opioid withdrawal. The common symptoms of opioid withdrawal that are associated with physical dependence may develop when an opioid is withdrawn rapidly. Symptoms include shaking chills, abdominal cramps, and joint pain.

A patient with chronic low back pain who took an opioid around-the-clock (ATC) for the past year decided to abruptly stop the medication for fear of addiction. He is now experiencing shaking chills, abdominal cramps, and joint pain. The nurse recognizes that this patient is experiencing symptoms of: A) Opioid toxicity. B) Opioid tolerance. C) Opioid addiction. D) Opioid withdrawal.

B) Type 1 diabetes C) Celiac disease Type 1 diabetes and Celiac disease are more common in Northern heritage. African Americans and Hispanics are at increased risk for Type 2 diabetes, hypertension, and metabolic syndrome. Vitamin C deficiency is not a common deficiency related to heritage or ethnicity.

A person of Northern heritage is at an increased risk for which of the following? (Select all that apply) A) Vitamin C deficiency B) Type I diabetes C) Celiac disease D) Type 2 diabetes E) Hypertension F) Metabolic syndrome

A) The sedative administered may have helped him sleep, but it is still necessary to assess pain. A pain assessment is still needed because sleep in a postoperative patient cannot be used as an assessment of a patient's pain level. Sleep may result from sedating effects of medication, but analgesia may not be present. It is important to wake and assess the patient to ensure that the pain is controlled and the patient is not overly sedated from the medication (a sign of impending respiratory depression).

A postoperative patient currently is asleep. Therefore the nurse knows that: A) The sedative administered may have helped him sleep, but it is still necessary to assess pain. B) The intravenous (IV) pain medication given in recovery is relieving his pain effectively. C) Pain assessment is not necessary. D) The patient can be switched to the same amount of medication by the oral route.

C) Secondary prevention A mammogram and a pap smear are cancer screening measures. They are considered secondary prevention measures, which are designed to identify individuals in an early stage of a disease process so that prompt treatment can be started. Illness prevention is considered a primary prevention measure, or strategies aimed at optimizing health and disease prevention in general and not linked to a single disease entity. Tertiary prevention measures are strategies that minimize the effects of disease and disability.

A primary health care provider has recommended a mammogram and a Papanicolaou (pap) smear for a 50-year-old female patient. In response to questions, the nurse teaches the patient about health promotion activities, describing the mammogram and pap smear as which forms of prevention? A) Illness prevention B) Primary prevention C) Secondary prevention D) Tertiary prevention

By handling dietary changes in a matter-of-fact way

A toddler on the pediatric unit is required to have temporary dietary restrictions after colorectal surgery. What is the best way for the nurse to promote adherence to the restrictions?

A nurse is assessing a patient's cranial nerve IX. Which items does the nurse gather before conducting the assessment? (Select all that apply.) a. Vial of sugar b. Snellen chart c. Tongue blade d. Ophthalmoscope e. Lemon applicator

ANS: A, C, E a. Vial of sugar c. Tongue blade e. Lemon applicator Cranial nerve IX is the glossopharyngeal, which controls taste and ability to swallow. The nurse asks the patient to identify sour (lemon) or sweet (sugar) tastes on the back of the tongue and uses a tongue blade to elicit a gag reflex. Ophthalmoscopes are used for vision. A Snellen chart is used to test cranial nerve II (optic).

Which characteristics are risk factors for cerebrovascular accident? Select all that apply. a. Excessive alcohol intake b. Smoking c. Eating large amounts of smoked foods d. Obesity e. Atherosclerosis f. High blood pressure

ANS: A, B, D, E, F a. Excessive alcohol intake b. Smoking d. Obesity e. Atherosclerosis f. High blood pressure Correct: All of these are risk factors for cerebrovascular accident. Incorrect: Eating large amounts of smoked foods is a risk factor for stomach cancer.

A nurse is asked by a co-worker why patient education/teaching is important. Which statements will the nurse share with the co-worker? (Select all that apply.) a. "Patient education is an essential component of safe, patient-centered care." b. "Patient education is a standard for professional nursing practice." c. "Patient teaching falls within the scope of nursing practice." d. "Patient teaching is documented and part of the chart." e. "Patient education is not effective with children." f. "Patient teaching can increase health care costs."

ANS: A, B, C, D a. "Patient education is an essential component of safe, patient-centered care." b. "Patient education is a standard for professional nursing practice." c. "Patient teaching falls within the scope of nursing practice." d. "Patient teaching is documented and part of the chart." Patient education has long been a standard for professional nursing practice. All state Nurse Practice Acts acknowledge that patient teaching falls within the scope of nursing practice. Patient education is an essential component of providing safe, patient-centered care. It is important to document evidence of successful patient education in patients' medical records. Patient education is effective for children. Different techniques must be used with children. Creating a well-designed, comprehensive teaching plan that fits a patient's unique learning needs reduces health care costs, improves quality of care, and ultimately changes behaviors to improve patient outcomes.

On examination, a nurse finds the patient has a productive cough with green sputum and inspiratory crackles. What other findings does this nurse expect during the examination? Select all that apply. a. Dull tones to percussion b. Increased vibration on vocal fremitus c. Fever d. Decreased diaphragmatic excursion e. A sharp, abrupt pain reported when patient breathes deeply f. Muffled sounds heard when the patient says "e-e-e"

ANS: A, B, C, E a. Dull tones to percussion b. Increased vibration on vocal fremitus c. Fever e. A sharp, abrupt pain reported when patient breathes deeply Correct: These abnormal findings are consistent with consolidation that may occur with pneumonia. Incorrect: Decreased diaphragmatic excursion occurs when the lung is overinflated as in emphysema. Muffled sounds when the patient says "e-e-e" is an expected finding. With a consolidation, the sound of "e-e-e" would be clear.

Which findings does the nurse expect when assessing the mouth of a healthy adult? Select all that apply. a. Lips appear pink, smooth, moist, and symmetric b. Teeth are white, yellow, or gray, with smooth edges c. Exposed tooth neck and brown spots between teeth d. Slight roughness on the dorsum of the tongue e. Hard palate appears smooth, pale, and immovable f. Mucous membranes are dry and intact

ANS: A, B, D, E a. Lips appear pink, smooth, moist, and symmetric b. Teeth are white, yellow, or gray, with smooth edges d. Slight roughness on the dorsum of the tongue e. Hard palate appears smooth, pale, and immovable Correct: These are all expected findings from a mouth assessment of a healthy adult. Incorrect: Receding gums expose tooth neck and may indicate gingival disease. Brown spots may indicate caries. Dry and intact mucous membranes may indicate dehydration.

Alcoholism increases the risk of cancers of the gastrointestinal tract. Which cancer risk is increased in patients with alcoholism? Select all that apply. a. Esophageal cancer b. Stomach cancer c. Pancreatic cancer d. Liver cancer e. Colon cancer f. Bladder cancer

ANS: A, B, D, E a. Esophageal cancer b. Stomach cancer d. Liver cancer e. Colon cancer Correct: The risk of esophageal, stomach, liver, and colon cancers are increased by heavy intake of alcohol. Incorrect: The risk of pancreatic and bladder cancers are increased with tobacco. However, the risk for esophageal, stomach, liver, and colon cancers are also increased with tobacco use.

Which data do nurses document under the heading of Personal and Psychosocial History? Select all that apply. a. Walks for 45 minutes each day b. Eats meats, vegetables, and fruit at two meals daily c. Is allergic to milk and milk products d. Is married and has two daughters whom is he close to e. Smokes marijuana once a week f. Grandfather died from prostate cancer

ANS: A, B, D, E a. Walks for 45 minutes each day b. Eats meats, vegetables, and fruit at two meals daily d. Is married and has two daughters whom is he close to e. Smokes marijuana once a week Correct: Walks for 45 minutes each day is documented under health promotion activity in Personal and Psychosocial History; eats meats, vegetables, and fruit at two meals daily is documented about diet activity in Personal and Psychosocial History; is married and has two daughters whom is he close to is documented under family and social relationship activity in Personal and Psychosocial History; smokes marijuana once a week is documented under personal habits activity in Personal and Psychosocial History. Incorrect: Allergic to milk and milk products is an allergy, which is documented under the heading Present Health Status; Grandfather died from prostate cancer is documented under the heading Family History

To complete a symptom analysis, which questions does a nurse ask patient who recently had a seizure for the first time? Select all that apply. a. "Did you have any warning signs before the seizure started?" b. "Did you lose consciousness during the seizure?" c. "Did the room seem to be spinning around before the seizure?" d. "Did you urinate during the seizure?" e. "What did you hear while you were seizing?" f. "How did you feel after the seizure?"

ANS: A, B, D, F a. "Did you have any warning signs before the seizure started?" b. "Did you lose consciousness during the seizure?" d. "Did you urinate during the seizure?" f. "How did you feel after the seizure?" Correct: These are all appropriate questions to ask to gather more data about this patient's first seizure. Incorrect: "Did the room seem to be spinning around before the seizure?" This question is about vertigo, which does not relate to this patient. "What did you hear while you were seizing?" The answer to this question is not needed in the data for this patient.

A nurse is preparing to teach patients. Which patient finding will cause the nurse to postpone a teaching session? (Select all that apply.) a. The patient is hurting. b. The patient is fatigued. c. The patient is mildly anxious. d. The patient is asking questions. e. The patient is febrile (high fever). f. The patient is in the acceptance phase.

ANS: A, B, E a. The patient is hurting. b. The patient is fatigued. e. The patient is febrile (high fever). Any condition (e.g., pain, fatigue) that depletes a person's energy also impairs the ability to learn, so the session should be postponed until the pain is relieved and the patient is rested. Postpone teaching when an illness becomes aggravated by complications such as a high fever or respiratory difficulty. A mild level of anxiety motivates learning. When patients are ready to learn, they frequently ask questions. When the patient enters the stage of acceptance, the stage compatible with learning, introduce a teaching plan.

A nurse is assessing several patients. Which assessment findings will cause the nurse to follow up? (Select all that apply.) a. Orthopnea b. Nonpalpable lymph nodes c. Pleural friction rub present d. Crackles in lower lung lobes e. Grade 5 muscle function level f. A 160-degree angle between nail plate and nail

ANS: A, C, D a. Orthopnea c. Pleural friction rub present d. Crackles in lower lung lobes Abnormal findings will cause a nurse to follow up. Orthopnea is abnormal and indicates cardiovascular or respiratory problems. Pleural friction rub is abnormal and indicated an inflamed pleura. Crackles are adventitious breath sounds and indicate random, sudden reinflation of groups of alveoli, indicating disruptive passage of air through small airways. Lymph nodes should be nonpalpable; palpable lymph nodes are abnormal. Grade 5 muscle function is normal. A 160-degree angle between nail plate and nail is normal; a larger degree angle is abnormal and indicates clubbing.

What are the functions of the upper airways? Select all that apply. a. Conduct air to lower airway. b. Provide area for gas exchange. c. Prevent foreign matter from entering respiratory system. d. Warm, humidify, and filter air entering lungs. e. Provide transportation of oxygen and carbon dioxide between alveoli and cells.

ANS: A, C, D a. Conduct air to lower airway. c. Prevent foreign matter from entering respiratory system. d. Warm, humidify, and filter air entering lungs. Correct: These are functions of the upper airway. Incorrect: Gas exchange occurs in the alveoli. The cardiovascular system provides transportation of oxygen and carbon dioxide between alveoli and cells.

Nurses inquire about lifestyle behaviors in those patients with specific risk factors for cataracts. Which characteristics are associated with risk factors for cataracts? Select all that apply. a. Smoking more than 20 cigarettes a day b. Having parents with cataracts c. Chronic consumption of alcohol d. Having a chronic disease, such as diabetes mellitus e. Being Asian f. Being a man

ANS: A, C, D a. Smoking more than 20 cigarettes a day c. Chronic consumption of alcohol d. Having a chronic disease, such as diabetes mellitus Correct: These are all risk factors for cataracts. Incorrect: Having parents with cataracts is not a genetic or familial disorder. Being Asian or a man are not risk factors, but being an African-American or being a woman are risk factors.

Which questions are pertinent to ask when obtaining a symptom analysis from a patient who reports breathing problems? Select all that apply. a. "How long have you had this problem with your breathing?" b. "Do you have a family history of breathing problems?" c. "Does this breathing problem come and go or is it constant?" d. "What do you do to make your breathing better?" e. "How does this breathing problem affect your work or daily activities?" f. "How many packs of cigarettes do you smoke a day?"

ANS: A, C, D, E a. "How long have you had this problem with your breathing?" c. "Does this breathing problem come and go or is it constant?" d. "What do you do to make your breathing better?" e. "How does this breathing problem affect your work or daily activities?" Correct: "How long have you had this problem with your breathing?", "Does this breathing problem come and go or is it constant?", "What do you do to make your breathing better?", and "How does this breathing problem affect your work or daily activities?" are questions asked in a symptom analysis. Use the mnemonic of OLD CARTS (e.g., onset of symptoms, location and duration of symptoms, characteristics, aggravating factors, related symptoms, treatment used, and severity of symptoms). Incorrect: "Do you have a family history of breathing problems?" This question relates to the patient's history; "How many packs of cigarettes do you smoke a day?" This question relates to the patient's history.

Which questions are pertinent to ask when obtaining a symptom analysis from a patient who reports headache? Select all that apply. a. "Describe what the headache feels like?" b. "When was your last eye examination?" c. "What makes the headaches worse?" d. "How do you rate the headaches on a scale of 0 (meaning no pain) to 10 (meaning the worse pain ever)?" e. "Do you have any symptoms with the headaches, such as nausea?" f. "When did you first notice the headaches?"

ANS: A, C, D, E, F a. "Describe what the headache feels like?" c. "What makes the headaches worse?" d. "How do you rate the headaches on a scale of 0 (meaning no pain) to 10 (meaning the worse pain ever)?" e. "Do you have any symptoms with the headaches, such as nausea?" f. "When did you first notice the headaches?" Correct: "Describe what the headache feels like?", "What makes the headaches worse?", "How do you rate the headaches on a scale of 0 (meaning no pain) to 10 (meaning the worse pain ever)?", "Do you have any symptoms with the headaches, such as nausea?", and "When did you first notice the headaches?" are questions asked in a symptom analysis. Use the mnemonic of OLD CARTS (e.g., onset of symptoms, location and duration of symptoms, characteristics, aggravating factors, related symptoms, treatment used, and severity of symptoms). Incorrect: "When was your last eye examination?" assumes that the headaches are related to a vision problem. Last eye examination is documented in the history under the heading of Past Health History.

A nurse is assessing the eyes of a healthy 72-year-old adult. What findings does the nurse expect? Select all that apply. a. Bulbar conjunctiva pink and clear, with small red vessels noted b. Sclera yellow and moist, cornea transparent c. Extraocular movement symmetric with peripheral vision noted d. Newspaper held at 18 inches to see clearly e. Sclera visible between upper lid and iris f. Gray to white circle noted where the sclera merges with the cornea g. Light reflects on the cornea at 12 o'clock in each eye

ANS: A, C, G a. Bulbar conjunctiva pink and clear, with small red vessels noted c. Extraocular movement symmetric with peripheral vision noted g. Light reflects on the cornea at 12 o'clock in each eye Correct: These are expected findings from an assessment of the eyes of a healthy adult. Incorrect: Sclera should be white and moist. Newspaper held at 18 inches to see clearly is due to presbyopia due to the patient's age. Patient must hold paper further away to see clearly. The upper lid should cover the upper part of the iris. Sclera is visible in hyperthyroidism. A gray to white circle is arcus senilis, which is an abnormal finding in older adults.

What findings does the nurse expect when assessing the ears of a healthy adult? Select all that apply. a. Cerumen noted in the outer ear canal b. Pinna located below the external corner of the eye c. Cone of light located in the 5 o'clock position in the left ear d. Ratio of air conduction to bone conduction 2:1 e. Tympanic membrane pearly gray f. Whispered words repeated accurately

ANS: A, D, E, F a. Cerumen noted in the outer ear canal d. Ratio of air conduction to bone conduction 2:1 e. Tympanic membrane pearly gray f. Whispered words repeated accurately Correct: These are all expected findings from an assessment of the ears of a healthy adult. Incorrect: The pinna should align with the outer canthus of the eye. Cone of light should be located in the 7 o'clock position in the left ear and the 5 o'clock position in the right ear.

A nurse is assessing the respiratory system of a healthy adult. Which findings does this nurse expect to find? Select all that apply. a. Thoracic expansion that is symmetric bilaterally b. Respiratory rate of 24 breaths/min c. Bronchophony revealing clear voice sounds d. Breath sounds clear with vesicular breath sounds heard over most lung fields e. Anteroposterior diameter of the chest about a 1:2 ratio of anteroposterior to lateral diameter f. Symmetric thorax with ribs sloping downward at about 45 degrees relative to the spine

ANS: A, D, E, F a. Thoracic expansion that is symmetric bilaterally d. Breath sounds clear with vesicular breath sounds heard over most lung fields e. Anteroposterior diameter of the chest about a 1:2 ratio of anteroposterior to lateral diameter f. Symmetric thorax with ribs sloping downward at about 45 degrees relative to the spine Correct: These are expected findings from a lung and respiratory assessment of a healthy adult. Incorrect: A respiratory rate of 24 breaths/min is considered tachypnea. Bronchophony revealing clear voice sounds is not performed unless there is an indication of consolidation of the lung, or if there was an abnormal finding of tactile fremitus. The expected finding is muffled voiced sounds rather than clear.

During an eye assessment, the nurse asks the patient to keep the head stationary and by moving the eyes only follow the nurse's finger as it moves side to side, up and down, and obliquely. This assessment technique collects what data about which cranial nerves? Select all that apply. a. Cranial nerve II (optic) b. Cranial nerve III (oculomotor) c. Cranial nerve IV (trochlear) d. Cranial nerve VI (abducens) e. Cranial nerve V (trigeminal)

ANS: B, C, D b. Cranial nerve III (oculomotor) c. Cranial nerve IV (trochlear) d. Cranial nerve VI (abducens) Correct: Cranial nerve III (oculomotor), cranial nerve IV (trochlear), and cranial nerve VI (abducens) provide muscle movement for the eyes. Incorrect: Cranial nerve II (optic) provides vision. Cranial nerve V (trigeminal) provides movement for the jaw and sensation for cornea, conjunctive, eyelids, teeth, tongue, and mouth.

A nurse suspects appendicitis in a patient with abdominal pain. Which findings are suggestive of appendicitis? Select all that apply. a. Pain radiating to the right shoulder b. Pain around the umbilicus c. Pain relieved by lying still d. Right lower quadrant pain e. Increased peristalsis

ANS: B, C, D b. Pain around the umbilicus c. Pain relieved by lying still d. Right lower quadrant pain Correct: These are all descriptions of pain related to appendicitis. Incorrect: Pain radiating to the right shoulder is associated with gallbladder disease. Increased peristalsis can be associated with gastroenteritis or diarrhea.

Which questions are pertinent for a nurse to ask a patient while performing a review of the cardiovascular system? Select all that apply. a. "Do you remember what your last cholesterol value was?" b. "Have you had chest pain or shortness of breath?" c. "Do you have trouble breathing when you lie down?" d. "Are your feet cold, numb, or do they change color?" e. "How much do you weigh?" f. "Have you noticed edema in your ankles at the end of the day?"

ANS: B, C, D, F b. "Have you had chest pain or shortness of breath?" c. "Do you have trouble breathing when you lie down?" d. "Are your feet cold, numb, or do they change color?" f. "Have you noticed edema in your ankles at the end of the day?" Correct: "Have you had chest pain or shortness of breath?", "Do you have trouble breathing when you lie down?", "Are your feet cold, numb, or do they change color?", and "Have you noticed edema in your ankles at the end of the day?" are questions asked to give the patient an opportunity to report symptoms of the cardiovascular system. Incorrect: "Do you remember what your last cholesterol value was?" relates to a lab value, which is objective data not documented in the history; "How much do you weigh?" is objective data not documented in the history.

On inspection, the nurse finds the patient's anteroposterior diameter of the chest to be the same as the lateral diameter. What other findings does this nurse expect during the examination? Select all that apply. a. Inspiratory wheezing found on auscultation b. Hyperresonance heard on percussion c. Decreased breath sounds heard on auscultation d. Deceased diaphragmatic excursion on percussion e. A sharp, abrupt pain reported when the patient breathes deeply f. Decreased to absent vibration on vocal fremitus

ANS: B, C, D, F b. Hyperresonance heard on percussion c. Decreased breath sounds heard on auscultation d. Deceased diaphragmatic excursion on percussion f. Decreased to absent vibration on vocal fremitus Correct: These are all indications of enlargement or destruction of alveoli that occurs in emphysema. Air is trapped, which increases the anteroposterior to lateral diameter creating a barrel chest, and pushes the diaphragm down decreasing the excursion and causing hyperresonance. The destroyed alveoli decrease the breath sounds and create absent vibration on vocal fremitus. Incorrect: Inspiratory wheezing found on auscultation indicates narrowed airways as found in asthma. A sharp, abrupt pain reported when the patient breathes deeply is pleuritic chest pain associated with pleural lining irritation that may occur in a patient with pleurisy or pneumonia.

Which data do nurses document under the heading of Past Health History? Select all that apply. a. Father has Alzheimer disease b. Last tetanus in 2009 c. Had chicken pox as a child d. Drinks three to four beers each day e. Had a dental examination 6 months ago

ANS: B, C, E b. Last tetanus in 2009 c. Had chicken pox as a child e. Had a dental examination 6 months ago Correct: Last tetanus is an immunization, chicken pox as a child is a childhood illness, and last examinations, including dental, are documented under the heading of Past Health History. Incorrect: Family History documents father's Alzheimer disease; patient drinking three to four beers each day refers to alcohol use, which is documented under the heading Personal and Psychosocial History.

A patient with heart failure reports having a cough with frothy sputum and awakening during the night to urinate. Based on this information, what abnormal data might this nurse expect to find during an examination? Select all that apply. a. S4 heart sound b. Dyspnea c. Jugular vein distention d. Pericardial friction rub e. Edema of ankle and feet at the end of the day f. S3 heart sound

ANS: B, C, E, F b. Dyspnea c. Jugular vein distention e. Edema of ankle and feet at the end of the day f. S3 heart sound Correct: All of these manifestations are consistent with fluid overload that occurs in heart failure because the cardiac output is decreased. Incorrect: S4 heart sounds signifies a noncompliant or "stiff'' ventricle. Hypertrophy of the ventricle precedes a noncompliant ventricle. Also, coronary artery disease is a major cause of a stiff ventricle. Pericardial friction rubs are caused by inflammation of the layers of the pericardial sac.

During a health fair, the nurse is alert for which risk factors for hypertension? Select all that apply. a. Excessive protein intake b. Having parents with hypertension c. Excessive alcohol intake d. Being Asian e. Experiencing persistent stress f. Elevated serum lipids

ANS: B, C, E, F b. Having parents with hypertension c. Excessive alcohol intake e. Experiencing persistent stress f. Elevated serum lipids Correct: These are all risk factors for hypertension. Incorrect: Excessive protein is not a risk factor for hypertension, but excessive sodium intake is a risk factor. Being Asian is not a risk factor, but being African-American is a risk factor.

During an examination of the head and neck of a healthy adult, the nurse expects which findings? Select all that apply. a. Small red lesions with white flakes scattered on the scalp b. The head and facial bones are proportional for the size of the body c. Depressions palpated on the right and left sides over the parietal bones d. Head held flexed 15 degrees to the left e. Face and jaw are symmetric and proportional f. Temporomandibular joint moves smoothly

ANS: B, E, F b. The head and facial bones are proportional for the size of the body e. Face and jaw are symmetric and proportional f. Temporomandibular joint moves smoothly Correct: These are expected findings from an assessment of the head of a healthy adult. Incorrect: Small red lesions with white flakes scattered on the scalp is an abnormal finding. The scalp should be intact without lesion or flakes. Depressions palpated on the right and left sides over the parietal bones is an abnormal finding. Perhaps this patient had skull tongs from cervical traction at one time. Head held flexed 15 degrees to the left is an abnormal finding. The head should be erect.

What findings does the nurse expect when assessing the cardiovascular system of a healthy adult? Select all that apply. a. Heart rate of 102 beats/min b. S1 and S2 present with regular rhythm c. Capillary refill greater than 3 seconds d. Blood pressure of 124/86 e. Warm, elastic turgor f. Pulse of smooth contour with 2+ amplitude

ANS: B, E, F b. S1 and S2 present with regular rhythm e. Warm, elastic turgor f. Pulse of smooth contour with 2+ amplitude Correct: These are all expected findings. Incorrect: A heart rate of 102 beats/min is tachycardia. Capillary refill should be 2 seconds or less. Blood pressure of 124/86 is prehypertension. Normal is less than 120 and less than 80 mm Hg.

Which manifestations does a nurse correlate with a patient with suspected meningitis? Select all that apply. a. Ptosis b. Loss of balance when standing with feet together and the eyes closed c. Confusion, agitation, and irritability d. Severe headache e. Stiff neck f. Lethargy

ANS: C, D, E, F c. Confusion, agitation, and irritability d. Severe headache e. Stiff neck f. Lethargy Correct: Confusion, agitation, and irritability; severe headache—this is a symptom of meningeal irritation due to inflammation of the meninges; stiff neck; lethargy. Patients may have changes in level of consciousness. Incorrect: Ptosis is drooping of eyelids controlled by the oculomotor cranial nerve. Loss of balance when standing with feet together and the eyes closed. This describes a positive Romberg test indicating a cerebellar problem.

D) 1 slice of bread One slice of bread represents one serving from this group. One-half cup cooked rice represents one serving from this group. Three to four crackers represent one serving from this group. One hamburger bun represents two servings from this group.

According to the food plan, what represents one serving from the bread, cereal, and grain products group? A) 1 cup cooked rice B) 6 soda crackers C) 1 hamburger bun D) 1 slice of bread

B) "Perceived self-efficacy is not related to the model." Within the model, perceived self-efficacy is one of the behavior-specific cognitions and affect. The behavior-specific cognitions have motivational significance within the model.

After a class on Pender's health promotion model, students make the following statements. Which statement does the faculty member need to clarify? A) "The desired outcome of the model is health-promoting behavior." B) "Perceived self-efficacy is not related to the model." C) "The individual has unique characteristics and experiences that affect his or her actions." D) "Patients need to commit to a plan of action before they adopt a health-promoting behavior."

D) Allows the patient time to express himself or herself and ask questions. When teaching older adults, it is important to establish rapport, involve them in their care, and allow them to progress at their own pace.

An older adult is being started on a new antihypertensive medication. In teaching the patient about the medication, the nurse: A) Speaks loudly. B) Presents the information once. C) Expects the patient to understand the information quickly. D) Allows the patient time to express himself or herself and ask questions.

A) Schedule a visit by another resident who is diabetic. B) Demonstrate food choices using food photographs. E) Encourage the patient's family to participate in teaching sessions. F) Ask the patient about past experiences with lifestyle changes. Strategies to promote learning in older adults include peer teaching, visual aids, family participation, and relating new learning to past experiences. Discussion of the patient's favorite foods is needed to determine how old favorites can be adapted to the new diet. Reminders about the damage already done will indicate that the changes are not worth the effort.

Appropriate approaches used by the long-term care nurse to provide education for a 73 year old who has just been diagnosed with diabetes include which of the following? (Select all the apply) A) Schedule a visit by another resident who is diabetic. B) Demonstrate food choices using food photographs. C) Avoid discussion of the patient's favorite foods. D) Remind the patient that a lot of damage has already occurred. E) Encourage the patient's family to participate in teaching sessions. F) Ask the patient about past experiences with lifestyle changes.

B) "Since my mother had breast cancer, I know that I am at increased risk for developing breast cancer." On the basis of health belief model, the statement indicates that the patient is concerned about developing breast cancer and feels that there is a risk or susceptibility based on recognition of a familial link for the disease. Once this link is recognized, the patient may perceive the personal risk.

As part of a faith community nursing program in her church, a nurse is developing a health promotion program on breast self-examination for the women's group. Which statement made by one of the participants is related to the individual's perception of susceptibility to an illness? A) "I have a door hanging tag in my bathroom to remind me to do my breast self-examination monthly." B) "Since my mother had breast cancer, I know that I am at increased risk for developing breast cancer." C) "Since I am only 25 years of age, the risk of breast cancer for me is very low." D) "I participate every year in our local walk/run to raise money for breast cancer research."

C) "I understand. Can you think of the greatest reason why stopping smoking would be challenging for you?" The patient's response indicates that he is in the precontemplation stage and not intending to make a change in behavior in the next 6 months. In this stage the patient is not interested in information about the behavior and may be defensive when confronted with the information. Asking an open-ended question about smoking may stimulate the patient to identify a reason to begin a behavior change. Nurses are challenged to motivate and facilitate health behavior change in working with individuals.

Based on the transtheoretical model of change, what is the most appropriate response to a patient who states: "Me, stop smoking? I've been smoking since I was 16!" A) "That's fine. Some people who smoke live a long life." B) "OK. I want you to decrease the number of cigarettes you smoke by one each day, and I'll see you in 1 month." C) "I understand. Can you think of the greatest reason why stopping smoking would be challenging for you?" D) "I'd like you to attend a smoking cessation class this week and use nicotine replacement patches as directed."

A) With each new report of pain B) Before and after administration of narcotic analgesics Following the initiation of a pain management plan, pain should be reassessed and documented on a regular basis as a way to evaluate the effectiveness of treatments. At a minimum, pain should be reassessed with each new report of pain and before and after administration of analgesics.

Following the initiation of a pain management plan, pain should be reassessed and documented on a regular basis as a way to evaluate the effectiveness of treatments. Pain should be reassessed at which minimum interval? Select all that apply. A) With each new report of pain B) Before and after administration of narcotic analgesics C)Every 10 minutes D)Every shift

A) Address needs common to a group. Group needs are often the focus of formal patient education courses or classes. Self-directed learning refers to an educational activity completed independently from the nurse or other health care providers. Describing nursing interventions with formal patient education courses or classes is not the most appropriate strategy, because most patient education is done by nurses during the explanation of an intervention, and that is a spontaneous, one-to-one activity. Formal courses or classes are not the most appropriate strategy to address a patient's or a family's questions; from a time perspective, it is not appropriate to have the patient or family wait for a class.

Formal patient education courses or classes would be the most appropriate strategy in which situation? A) Address needs common to a group. B) Explain self-directed learning. C) Describe nursing interventions. D) Respond to questions of a patient's family.

B) Adolescents Eating patterns may reveal poor eating habits associated with multiple school or athletic activities. The eating patterns of school-age children usually are influenced by their parents. Although many busy adults may skip meals, as a group most adults eat consistently. Although some older adults may skip meals, as a group most eat consistently.

In which age group is skipping meals most commonly seen? A) School-age children B) Adolescents C) Adults D) Older adults

A) Adherence Adherence, culture, developmental level, family dynamics, and motivation are considered interrelated concepts regarding patient attributes and preferences. Interrelated concepts regarding the professional role of a nurse include health promotion, leadership, technology and informatics, quality, collaboration, and communication.

Interrelated concepts regarding patient attributes and preferences that a nurse would consider when addressing patient education include which concept? A) Adherence B) Health promotion C) Quality D) Technology

A) Culture Culture, development, adherence, and motivation are patient attribute concepts. Interrelated concepts regarding professional nursing include evidence, health care economics, health policy, and patient education. Nutrition is a health and illness concept.

Interrelated concepts regarding patient attributes that a nurse manager would consider when addressing concerns about the quality of health promotion include which type of attribute? A) Culture B) Evidence C) Health policy D) Nutrition

D) Nonsteroidal anti-inflammatory drug (NSAID) Unless contraindicated, all surgical patients should routinely be given acetaminophen and an NSAID in scheduled doses throughout the postoperative course. Opioid analgesics are added to the treatment plan to manage moderate-to-severe postoperative pain. A local anesthetic is sometimes administered epidurally or by continuous peripheral nerve block.

Postoperative surgical patients should be given alternating doses of acetaminophen and which medication throughout the postoperative course, unless contraindicated? A) Antihistamine B) Local anesthetic C) Opioids D) Nonsteroidal anti-inflammatory drug (NSAID)

B) Using mind-body therapies such as music therapy, distraction techniques, meditation, prayer, hypnosis, guided imagery, relaxation techniques, and pet therapy Mind-body therapies are designed to enhance the mind's capacity to affect bodily functions and symptoms and include music therapy, distraction techniques, meditation, prayer, hypnosis, guided imagery, relaxation techniques, and pet therapy, among many others. Although getting exercise, drinking chamomile tea and applying gels, and receiving acupuncture and attending church services may be beneficial, they are not classified as mind-body therapies in combination as specified in these answer choices.

Stephanie is a 70-year-old retired schoolteacher who is interested in nondrug, mind-body therapies, self-management, and alternative strategies to deal with joint discomfort from rheumatoid arthritis. Which of the following options should you suggest for her plan of care, considering her expressed wishes? A) Using a stationary exercise bicycle and free weights and attending a spinning class B) Using mind-body therapies such as music therapy, distraction techniques, meditation, prayer, hypnosis, guided imagery, relaxation techniques, and pet therapy C) Drinking chamomile tea and applying icy/hot gel D) Receiving acupuncture and attending church services

D) "My 4-month-old baby does not respond to loud noise." An expected finding from a 4-month-old baby is response in the form of startle reflex or responding in some way to noises. Option A is a normal behavior for an infant of this age. A 3-year-old is distracted easily and may not listen. The speech of a 15-month-old is rarely clear.

The mother of a child tells the nurse that she is concerned that her child may be having trouble hearing. Which statement made by a parent suggests a possible hearing impairment in the child? A) "My 5-month-old baby is babbling but not saying any words." B) "My 3-year-old son does not listen to me." C) "I have a hard time understanding my 15-month-old baby." D) "My 4-month-old baby does not respond to loud noise."

B) Mother died from CAD at age 48 C) History of hypertension E) Elevated cholesterol level Genetic and physiological risk factors include those related to heredity, genetic predisposition to an illness, or those that involve the physical functioning of the body. Certain physical conditions such as being pregnant or overweight place increased stress on physiological systems (e.g., the circulatory system), increasing susceptibility to illness in these areas. A person with a family history of diabetes mellitus is at risk for developing the disease later in life because of a hereditary and genetic predisposition to the disease.

The nurse assesses the following risk factors for coronary artery disease (CAD) in a female patient. Which factors are classified as genetic and physiological? (Select all that apply.) A) Sedentary lifestyle B) Mother died from CAD at age 48 C) History of hypertension D) Eats diet high in sodium E) Elevated cholesterol level

60 g

The nurse instructs a pregnant client in the sources of protein that can be used to meet the increased daily requirement during pregnancy. How many grams of protein should the client eat each day?

C) Observation of sternal and supraclavicular retractions with breathing Sternal retractions are a significant finding that must not be overlooked. 30 breaths/minute is a normal respiratory rate for infants and young children. An irregular pattern of respiration may be considered normal and is not an uncommon finding for a 1-year-old. Auscultation of bronchovesicular sounds are normal lung sounds for the lung field.

The nurse is assessing a 1-year-old and suspects respiratory distress. Which finding indicates this? A) A respiratory rate of 30 breaths/minute B) An irregular respiratory pattern C) Observation of sternal and supraclavicular retractions with breathing D) Auscultation of bronchovesicular sounds throughout the lung field

A) The infant's toes fan when the lateral surface of the sole is stroked. Option A is the Babinski reflex and does not disappear until approximately 18 months of age. Option B is called the palmar grasp reflex and should disappear by 4 months of age. Option C is called the palmar grasp reflex and should disappear by 4 months of age. Option D is called the rooting reflex and should disappear by 4 months of age.

The nurse is assessing a 13-month-old. The nurse notes which reflexes as normal for this age group? A) The infant's toes fan when the lateral surface of the sole is stroked. B) The infant demonstrates a positive Moro reflex when startled. C) The infant grasps a finger that is touched against ulnar side of his or her hand. D) The infant turns his or her head in response to brushing the infant's cheek.

B) An African-American female who is breastfeeding. Vitamin D deficiency is more frequently found among persons of African heritage and has increased in prevalence, especially among the infants of breastfeeding African-American mothers. Caucasian females do not share these risk factors. There is no known risk of hypoglycemia and vitamin D deficiency; however, diabetes increases the risk for vitamin D deficiency. There is no known risk of vitamin D deficiency in normal-weight females of Hispanic heritage; however, obesity is a risk factor.

The nurse is assessing a group of patients to determine their risk of vitamin D deficiency. Which of the following patients has the highest risk for vitamin D deficiency? A) A Caucasian female who is 39 weeks gestation. B) An African-American female who is breastfeeding. C) An Asian female diagnosed with hypoglycemia. D) A Hispanic female who has a BMI of 24.1

B) Speech impairments E) Hearing impairment F) Failure to thrive G) Premature birth Answer options B, E, F, and G are all risk factors associated with cleft palate. Hearing, speech, and failure to thrive are associated with unrepaired cleft palates, particularly if the repair of the palate does not occur early on in the infant's life. Other options are normal behaviors and occurrences that do not place the infant at risk for cleft palate.

The nurse is assessing a newborn and notices the child has a cleft palate. What are the risks associated with this finding? Select all that apply. A) A vaginal breech delivery B) Speech impairments C) A birth weight of 1900 g D) Exposure to music while in utero E) Hearing impairment F) Failure to thrive G) Premature birth

A) Acute pain By definition, acute pain is pain that lasts less than 6 months. Chronic pain typically lasts longer than 6 months. Cancer pain changes over time as the disease progresses. Nonmalignant pain may slowly increase and usually occurs over a long period of time.

The nurse is assessing a patient who has pain with a sudden onset and a limited duration and that subsides as healing occurs. Which type of pain would this be considered? A) Acute pain B) Chronic pain C)Cancer pain D) Nonmalignant pain

C) Wong/Baker Faces Rating Scale Wong/Baker Faces Rating Scale works well for children over 3 years of age because it has pictures. Visual Analog Scale is ideal for older adults secondary to hearing or visual deficits but not for young children. Numeric Pain Intensity Scale ranges from 0 to 10; toddlers may not be able to articulate numbers. Pain Intensity Scale ranges from 0 to 5; toddlers may not be able to articulate numbers.

The nurse is performing a pain assessment of a 4-year-old toddler. Which pain assessment scale would be best for this patient? A) Visual Analog Scale B) Numeric Pain Intensity Scale C) Wong/Baker Faces Rating Scale D) Pain Intensity Scale

D) Asking the patient to do a 24-hour dietary recall Having the patient do a 24-hour food recall will assist the nurse in collaborating with the patient to include foods that the patient enjoys. Option A will likely lead to adherence to the plan for two reasons: (1) The patient is involved in the plan. (2) The patient will not be deprived of favorite foods. Comparing what is recommended requires the patient to know what is recommended. Patients who need to lose weight may not have mastered this skill. Filling out a food plan may not include the patient's favorite foods. The utilization of the USDA MyPlate is a good intervention for implementation of the teaching plan.

The nurse is assessing a patient's dietary intake to help the patient lose weight. What is the easiest way to assess the patient's normal dietary intake? A) Comparing established eating habits with Dietary Reference Intakes B) Asking the nurse to fill out a food plan C) Comparing the recommended dietary allowances to the USDA MyPlate D) Asking the patient to do a 24-hour dietary recall

C) Fats Macronutrients include carbohydrates, proteins, and fats. Minerals are considered micronutrients. Vitamins are considered micronutrients. Water is an essential dietary component, but it is not a macronutrient.

The nurse is assessing a patient's nutritional status and suspects the patient needs more macronutrients. Which of the following are considered macronutrients? A) Minerals B) Vitamins C) Fats D) Water

B) Decreased activities of daily living It is important to determine if the patient is capable of obtaining and preparing adequate food. Elevations in blood pressure may be affected by nutritional intake but are not a risk for deficiency. Many individuals have food allergies, but this in itself should not increase the risk of nutritional deficiency. Exercise pattern may provide insight to the nurse's activity level but not necessarily to the nutritional level.

The nurse is assessing an elderly patient's risk of nutritional deficiency. An important risk factor for nutritional deficiency in the elderly is: A) Increased blood pressure B) Decreased activities of daily living. C) An allergy to shellfish D) Exercise pattern

B) An elevated blood pressure C)An elevated heart rate D) Diaphoresis Options B to D are common objective findings associated with acute pain. Crying and pain level are subjective findings. Vital signs will most likely be elevated during acute pain.

The nurse is assessing for objective findings are associated with the patient's pain level. Which findings are commonly associated with acute pain? Select all that apply. A) The patient is crying B) An elevated blood pressure C)An elevated heart rate D) Diaphoresis E) The patient states a pain level of 8 out of 10 on pain scale F) Vital signs stable

B) perception of the pain or discomfort. Individuals may perceive a stimulus differently, making the pain experience very individualized. Pain may be experienced whether or not the stimulus can be explained. Although age may contribute to various causes of pain, age itself is not an indicator of the pain experience. Individuals may perceive the same stimulus differently.

The nurse is assessing the degree of pain or discomfort a patient is feeling. The nurse knows that this will be dependent primarily on: A) ability to explain the pain or discomfort. B) perception of the pain or discomfort. C) age of the individual. D) type of painful stimulus.

C) The pain response may be influenced by one's culture. D) Individuals may express pain differently. E) Pain management may vary depending on the source of pain. Culture influences how an individual responds to pain. Pain tolerance is highly variable. Patients may need narcotics for postoperative pain, whereas muscle strains may respond well to anti-inflammatory medications. Pain is not predictable regardless of culture or ethnicity and cannot be stereotyped by culture.

The nurse is attending an in-service on pain management for postoperative patients. Which statement regarding pain is true? Select all that apply. A) An individual's pain response is predictable based on his or her culture or ethnicity. B) Individuals from all cultures respond to pain similarly. C) The pain response may be influenced by one's culture. D) Individuals may express pain differently. E) Pain management may vary depending on the source of pain.

A) Vitamin B12 Vitamin B12 deficiency can occur as a result of the reduced gastric acidity associated with use of proton pump inhibitors, and supplementation is often warranted. Vitamin C deficiency is not a known deficiency associated with medications. Vitamin D deficiency may occur in patients who take cholesterol medication, and this link is currently being investigated. Omega-3 fatty acids may be used as monotherapy or in conjunction with cholesterol medication for patients with hyperlipidemia.

The nurse is caring for a patient diagnosed with peptic ulcer disease (PUD). The patient was prescribed the proton pump inhibitor Prevacid (lansoprazole). Which of the following supplements may be prescribed to prevent deficiency? A) Vitamin B12 B) Vitamin C C) Vitamin D D) Omega-3 fatty acids

A) Beneficence D) Nonmaleficence The ethical principles of beneficence (the duty to benefit another) and nonmaleficence (the duty to do no harm) compel health care professionals to provide pain management and comfort. Liberty and justice are ethical principles that deal with fairness. Autonomy deals with the patient's right to make decisions independently.

The nurse is compelled to address and manage a patient's pain level by which ethical principles? Select all that apply. A) Beneficence B) Liberty C) Autonomy D) Nonmaleficence E) Justice

B) Involve those affected by the problem in the planning process. E) Assess commonly held health beliefs among the cultural group. G) Include cultural practices that are relevant to the specific community. When planning a program, it is important to involve those affected by the problem to better serve the group's needs. Learning about traditional health practices/beliefs within the community enhances the nurse's understanding of a specific cultural group. Effective educational strategies require the nurse to use culturally tailored, interactive instructional methods that involve the family and community in the learning process.

The nurse is organizing a disease prevention program for a specific cultural group. To effectively meet the needs of this group the nurse will: Select all that apply. A) Assess the needs of the community in general. B) Involve those affected by the problem in the planning process. C) Develop generalized goals and objectives for the program. D) Use educational materials that are simplistic and have many pictures. E) Assess commonly held health beliefs among the cultural group. F) Educate the specific cultural group about Western concepts of health and illness. G) Include cultural practices that are relevant to the specific community.

C) 6 years By 6 years of age the globe of the eye attains the shape of an adult eye. Because the eyes of young children are less spheric than those of adults, their vision is myopic. Children can reach a similar visual acuity earlier than 10 years of age.

The nurse is performing an eye exam on a child. The nurse knows that the child will have the visual acuity of an adult at which age? A) 2 years B) 3 years C) 6 years D) 10 years

B) When the patient states that he or she is pain free C) Just before lunch, when the patient is most awake and alert Plan teaching when the patient is most attentive, receptive, alert, and comfortable.

The nurse is planning to teach a patient about the importance of exercise. When is the best time for teaching to occur? Select all that apply. A) When there are visitors in the room B) When the patient states that he or she is pain free C) Just before lunch, when the patient is most awake and alert D) When the patient is talking about current stressors in his or her life E) When the patient is being transported for a procedure

B) The parietal lobe of the cerebral cortex Pain is not actually perceived until the parietal lobe is stimulated. The dorsal horn of the spinal cord is the location of the "gate" of pain transmission. Afferent nerves are involved in the transmission of the pain impulse. Nociceptors are involved in the transduction of the pain process.

The nurse is reviewing the pathophysiology of pain. Where does the perception of pain actually occur? A) The dorsal horn of the spinal cord B) The parietal lobe of the cerebral cortex C) The afferent (sensory) nerves D) The visceral and somatic free nerve endings (nociceptors)

A) Fish Fish contains all of the essential amino acids. Cereal is a starch. Bread is a starch. Oatmeal is a grain and is considered a starch.

The nurse is teaching a patient the importance of protein for healing. Which foods should the nurse include in the teaching plan? A) Fish B) Cereal C) Bread D) Oatmeal

A) 2 - 3 Between 2 and 3 servings is the recommended daily intake of dairy. 3 to 5 servings would be recommended for children and pregnant and lactating women. 5 to 6 servings of dairy is not a standard recommendation for any age category. 0 to 2 servings is not a standard recommendation for any age category.

The nurse is teaching adult male healthy eating guidelines. How many servings of dairy should the nurse recommend for this patient? A) 2 - 3 B) 3 - 5 C) 5-6 D) 0 - 2

C) Food preferences and dislikes have a strong influence on what a person eats. Option C becomes important with dietary teaching. Chronic illness is not identified by a person's food preference. Longevity may be influenced by the foods consumed, but food preferences cannot be used to predict someone's life span. Dietary compliance cannot be determined based on food preference alone, but it helps to identify those who may struggle with special diets.

The nurse is working with a patient to develop a nutritional plan for a patient newly diagnosed with diabetes. The nurse assesses what the patient's food preferences are because: A) Food preferences can indicate a chronic disease that the nurse may be unaware of. B) Life expectancy can be predicted based on food preferences. C) Food preferences and dislikes have a strong influence on what a person eats. D) A list of food preferences will help identify individuals who will not comply with special diets.

B) Labored breathing and fever A barking cough, fever, and dyspnea are symptoms consistent with croup. Wheezing and coarse rhonchi bilaterally are associated with asthma. Hyperresonance with percussion is associated with a hyperinflated lung. Productive, blood-tinged sputum is associated with tuberculosis or possible pneumonia.

The nurse notes that a 2-year-old child has a cough that sounds like a bark. What other findings should the nurse anticipate? A) Wheezing and coarse rhonchi bilaterally B) Labored breathing and fever C) Hyperresonance with percussion D) Productive, blood-tinged sputum

C) The amount of daily acetaminophen The Food and Drug Administration (FDA) recommends a maximum daily dose of 4 g of acetaminophen, and many authorities believe that the maximum daily dose should be lower (3000 to 3200 mg/day) in the outpatient setting to reduce the risk of hepatotoxicity.

The nurse reviews a patient's medical administration record (MAR) and finds that the patient has received oxycodone/acetaminophen (Percocet) (5/325), two tablets PO every 3 hours for the past 3 days. What concerns the nurse the most? A) The patient's level of pain B) The potential for addiction C) The amount of daily acetaminophen D) The risk for gastrointestinal bleeding

C) There is a foul-smelling odor coming from the child's nose. A small toy or a piece of food in the nose will cause a foul odor. Most children prefer to be close to their parents when being examined. Purulent drainage usually indicates an infection. Foreign objects usually do not cause pain when the child is lying down.

The nurse suspects a foreign body in a young child's nose. Which finding is most likely to cause the nurse to suspect this? A) The mother states that the child plays with toys. B) There is purulent discharge coming from the child's nose. C) There is a foul-smelling odor coming from the child's nose. D) The child cries when lying down.

D) Thin upper lip A thin upper lip and small palpebral fissures are other common findings. Fetal alcohol syndrome does not change the appearance or function of the ear. "Moon face" is a finding associated with Cushing syndrome or steroid therapy. Torticollis usually is caused by constriction of the sternocleidomastoid muscle.

The nurse suspects an infant has fetal alcohol syndrome. Which assessment finding is consistent for an infant with fetal alcohol syndrome? A) Malformation of the ear B) "Moon face" C) Torticollis D) Thin upper lip

C) Kindergarten age The child can undergo sensory neurologic testing by kindergarten age. Six months' babies are too young because they need to be able to follow directions. Toddlers have trouble following commands necessary to perform testing. Early diagnosis is preferable for middle school-age children so that treatments and interventions can be implemented.

The nurse suspects that a child has sensory impairment. At what age can the child undergo sensory neurologic testing? A) At least 6 months old B) Toddlers C) Kindergarten age D) Middle school age

D) Serum albumin Serum albumin measures serum protein. Hemoglobin and hematocrit screen for anemia resulting from dietary deficiency. Serum glucose levels are a reflection of carbohydrate metabolism. Lipid profile is an indication of fat (lipid) metabolism.

The nurse suspects that the patient is suffering from malnutrition. Which laboratory test indicates a patient's protein calorie status? A) Hemoglobin and hematocrit B) Serum glucose levels C) Lipid profile D) Serum albumin

A) Primary prevention Seatbelt and helmet use are considered primary prevention measures, or strategies aimed at optimizing health and disease prevention in general. Rehabilitation strategies are tertiary prevention measures, which minimize the effects of disease and disability. Secondary prevention strategies are measures designed to identify individuals in an early stage of a disease process so that prompt treatment can be started.

The school nurse incorporates seatbelt and helmet use in a high school class on health promotion as examples of which strategies? A) Primary prevention B) Rehabilitation C) Secondary prevention D) Tertiary prevention

A) Anthropometrics B) Biochemical tests results C) Clinical evaluation of diet E) Body mass index (BMI) Options A, B, C, and E are included in routine assessment of nutritional assessment for adolescent patients. Unless clinically indicated, biochemical tests are not routinely performed with this age group.

The school nurse is assessing the nutritional status of a healthy adolescent. Which assessment will the nurse include in this assessment? (Select all that apply) A) Anthropometrics B) Biochemical tests results C) Clinical evaluation of diet D) Dietary assessment E) Body mass index (BMI)

A) explore exposure to loud noises, including music. B) perform the whisper test. D) perform screening tests for hearing loss. E) ask about previous antibiotic use. Repeated exposure to loud noise such as music places the adolescent at risk for long-term, irreversible hearing loss. The whisper test assesses hearing ability. A hearing screening is part of a routine examination for adolescents in the school setting. Previous high-dose antibiotic use may cause hearing loss. Examining the ear canal is a component of the physical examination.

The school nurse is performing a hearing screening for an adolescent. The primary focus of a history and examination for this age group would be: Select all that apply. A) explore exposure to loud noises, including music. B) perform the whisper test. C) examine the ear canal with an otoscope. D) perform screening tests for hearing loss. E) ask about previous antibiotic use.

C) The patient's subjective data The most accurate and reliable evidence of pain is the patient's report. Nurses may have experience regarding what types of situations tend to cause more pain than others, but this is not a valid method for assessing pain severity. Pain is an individual experience; thus the same stimulus may cause various responses from various individuals. Objective findings may contribute to pain assessment, but they do not describe the severity of pain the patient experiences.

Which findings by the nurse would produce the most accurate assessment of the severity level of a patient's pain? A) The nurse's experience B) The cause of the pain C) The patient's subjective data D) The patient's objective findings

D) Survey nurses about patient teaching. The first step in addressing any concern is assessment, or determining what the issues are, so conducting a verbal or written survey would be the most appropriate first step. Education of patients is integral to professional nursing practice; it would not be appropriate, or even possible, to assign one nurse to teach patients, because much patient education is informal, spontaneous, and takes place during treatments or when a nurse is responding to patient questions. There is no information to support a problem with the organization of patient teaching resources. Posting a teaching outline in the lounge could be an appropriate strategy if a need related to a specific area was identified; however, a needs assessment must first be completed.

To address administrative concerns about the effectiveness of staff nurses related to patient education, what is the nurse manager's first action? A) Assign one nurse to teach patients. B) Organize patient teaching resources. C) Post a teaching outline in the lounge. D) Survey nurses about patient teaching.

C) The individual becomes angry when the physician tells him that he needs to increase his activity to lose 30 lbs. A) The individual recognizes that he is out of shape when his daughter asks him to walk with her after school. E) The individual visits the local running store to purchase walking shoes and obtain advice on a walking plan. D) The individual walks 2 to 3 miles, 5 nights a week, with his wife. B) Eight months after beginning walking, the individual participates with his wife in a local 5K race. This sequence follows the order of the steps of transtheoretical model of change: precontemplation, contemplation, preparation, action, and maintenance.

Using the Transtheoretical Model of Change, order the steps that a patient goes through to make a lifestyle change related to physical activity. A) The individual recognizes that he is out of shape when his daughter asks him to walk with her after school. B) Eight months after beginning walking, the individual participates with his wife in a local 5K race. C) The individual becomes angry when the physician tells him that he needs to increase his activity to lose 30 lbs. D) The individual walks 2 to 3 miles, 5 nights a week, with his wife. E) The individual visits the local running store to purchase walking shoes and obtain advice on a walking plan.

Pillows covered with ultra mesh fabric Cloth drapes removed from all windows Air-conditioning unit running in the home Carpeting replaced with hard wood floors

What did the nurse observe during a home visit that indicates effective teaching about avoidance therapy provided to a client with type I rapid hypersensitivity reaction? Select all that apply.

B) Health promotion Health promotion is the process of enabling people to increase control over, and improve, their health, according to the World Health Organization. Community care refers to interventions directed at a community rather than a process. High-level wellness refers to a positive state of health for an individual, a family, or a community; it is not a process. Primary prevention refers to strategies aimed at optimizing health and disease prevention rather than a process.

What is the process of enabling people to increase control over and improve their health? A) Community care B) Health promotion C) High-level wellness D) Primary prevention

B) Analogy Analogies use familiar images when teaching to help explain complex information.

When a nurse is teaching a patient about how to administer an epinephrine injection in case of a severe allergic reaction, the nurse tells the patient to hold the injection like a dart. Which of the following instructional methods did the nurse use? A) Telling B) Analogy C) Demonstration D) Simulation

B) Lack of interest in exercise C) Difficulty falling asleep D) Lack of desire to go to work E) Anxiety while working Nurses are particularly susceptible to the development of secondary traumatic stress and burnout—the components of compassion fatigue. Symptoms include decline in health, emotional exhaustion, irritability, restlessness, impaired ability to focus and engage with patients, feelings of hopelessness, inability to take pleasure from activities, and anxiety.

Which of the following are symptoms of secondary traumatic stress and burnout that commonly affect nurses? (Select all that apply.) A) Regular participation in a book club B) Lack of interest in exercise C) Difficulty falling asleep D) Lack of desire to go to work E) Anxiety while working

D) Teach about evaluation of Internet resources. Evaluation of resources is an essential component of gathering information from the Internet, and the nurse would want to be sure the patient finds valid and reliable information. A majority of adults in the United States use the Internet to find information on many aspects of life, and this use of technology expands the role of the nurse in patient education to include teaching on how to evaluate Internet sources. Discounting the reliability of the Internet would not support the positive behavior and motivation of the patient to learn. The nurse would want to evaluate what the patient learns from the Internet rather than the patient's computer competency. Providing a list of recommended sources would be appropriate and support the patient's motivation, but it would not be the first thing the nurse would do.

When a patient tells the nurse about plans to do research about the patient's diagnosis and potential treatment on the Internet, what is the nurse's most appropriate initial response? A) Discount the reliability of the Internet. B) Evaluate the patient's computer competency. C) Provide a list of recommended sources. D) Teach about evaluation of Internet resources.

D) "What does your pain feel like?" To assess the quality of a person's pain, have the patient describe the pain in his or her own words. Assessing when the pain started may determine the type of pain but not the quality of pain. Question B deals with type of pain. The nurse would be influencing the patient's responses by suggesting the type of pain.

When assessing the quality of a patient's pain, the nurse should ask which of the following question? A) "When did the pain start?" B) "Is the pain a stabbing pain?" C) "Is it a sharp pain or dull pain?" D) "What does your pain feel like?"

C) Colored visual diagrams that categorize foods according to fat and sodium content Patients who are visual-spatial learners enjoy learning through pictures, visual charts, or any exercise that allows them to visualize concepts.

When planning for instruction on cardiac diets to a patient with heart failure, which of the following instructional methods would be the most appropriate for someone identified as a visual/spatial learner? A) Printed pamphlets on cardiovascular disease and dietary recommendations from the American Heart Association B) A role-play activity requiring the patient to select proper foods from a wide selection C) Colored visual diagrams that categorize foods according to fat and sodium content D) A lecture-style discussion on heart healthy diet options

A) Ask questions. The evaluation should match the goal. In this scenario, the goal is related to long-term effects, so providing an opportunity for the patient and family to ask questions gives the nurse information about their understanding of the content and allows the nurse to evaluate the cognitive and affective impacts of the teaching. Opportunities to inject insulin, meet exercise goals, and prepare a menu would be strategies to assess psychomotor domain learning, and this is not the goal of the teaching activity.

When planning the evaluation of a teaching activity that has the goal of educating a patient and family about the long-term effects of diabetes, it would be most appropriate for the nurse to include which opportunity for the patient? A) Ask questions. B) Inject insulin. C) Meet exercise goals. D) Prepare a menu.

A) Holistic The nurse is using a holistic model of care that takes a more holistic view of health by considering emotional and spiritual well-being and other dimensions of an individual to be important aspects of physical wellness. The holistic health model of nursing attempts to create conditions that promote optimal health. Nurses using the holistic nursing model recognize the natural healing abilities of the body and incorporate complementary and alternative interventions such as music therapy, reminiscence, relaxation therapy, therapeutic touch, and guided imagery because they are effective, economical, noninvasive, nonpharmacological complements to traditional medical care.

When taking care of patients, a nurse routinely asks if they take any vitamins or herbal medications, encourages family members to bring in music that the patient likes to help the patient relax, and frequently prays with her patients if that is important to them. The nurse is practicing which model? A) Holistic B) Health belief C) Transtheoretical D) Health promotion

D) TENS electrodes are applied near or directly on the site of pain. TENS units act on both the central and peripheral nervous systems. The peripheral effect occurs through activation of the neuroreceptors at or near the source of pain; therefore the electrodes should be placed near the site.

When teaching a patient about transcutaneous electrical nerve stimulation (TENS), which information do you include? A) TENS works by causing distraction. B) TENS therapy does not require a health care provider's order. C) TENS requires an electrical source for use. D) TENS electrodes are applied near or directly on the site of pain.

A) Apply ice using firm pressure over skin. B) Apply ice for 5 minutes or until numbness occurs. E) Use a slow, circular steady massage. Apply the ice with firm pressure over the skin; then use a slow, steady circular massage. Apply ice for 5 minutes or until the patient feels numbness. It is acceptable to apply ice 2 to 5 times a day.

When using ice massage for pain relief, which of the following is correct? Select all that apply. A) Apply ice using firm pressure over skin. B) Apply ice for 5 minutes or until numbness occurs. C) Apply ice no more than 3 times a day. D) Limit application of ice to no longer than 10 minutes. E) Use a slow, circular steady massage.

C) A school health nurse provides a program to the first-year students on healthy eating. Primary prevention aimed at health promotion includes health education programs, immunizations, and physical and nutritional fitness activities. Primary prevention includes all health promotion efforts and wellness education activities that focus on maintaining or improving the general health of individuals, families, and communities.

Which activity shows a nurse engaged in primary prevention? A) A home health care nurse visits a patient's home to change a wound dressing. B) A nurse is assessing risk factors of a patient in the emergency department admitted with chest pain. C) A school health nurse provides a program to the first-year students on healthy eating. D) A nurse schedules a patient who had a myocardial infarction for cardiac rehabilitation sessions weekly.

A) Only the patient should push the button. C) The PCA system can set limits to prevent overdoses from occurring. E) Do not push the button to go to sleep. The safety of PCA is based on the fact that it requires an awake patient to activate the button. The safety is compromised when someone else pushes the button for the patient. A limit on the number of doses per hour or 4-hour intervals may be set. Opioids (morphine PCA) are intended to provide analgesia; drowsiness is an undesirable potential side effect of opioids, and the PCA should only be used for analgesia.

Which of the following instructions is crucial for the nurse to give to both family members and the patient who is about to be started on a patient-controlled analgesia (PCA) of morphine? Select all that apply. A) Only the patient should push the button. B) Do not use the PCA until the pain is severe. C) The PCA system can set limits to prevent overdoses from occurring. D) Notify the nurse when the button is pushed. E) Do not push the button to go to sleep.

A) Body alignment and superficial heat and cooling Body alignment and thermal management are examples of nonpharmacological measures to manage pain. They can be used individually or in combination with other nondrug therapies. Proper body alignment achieved through proper positioning can help prevent or relieve pain. Thermal measures such as the application of localized, superficial heat and cooling may relieve pain and provide comfort. PCA, neurostimulation, and peripheral nerve blocks are not totally self-managed or alternative therapies, because they are used under the direction of medical professionals.

Which of the following is a priority for a nurse to include in a teaching plan for a patient who desires self-management and alternative strategies? A) Body alignment and superficial heat and cooling B) Patient-controlled analgesia (PCA) pump C) Neurostimulation D) Peripheral nerve blocks

B) Difficulty arousing the patient Sedation is a concern because it may indicate that the patient is experiencing opioid-related side effects. Advancing sedation may indicate that the patient may progress to respiratory depression.

Which of the following signs or symptoms in an opioid-naive patient is of greatest concern to the nurse when assessing the patient 1 hour after administering an opioid? A) Oxygen saturation of 95% B) Difficulty arousing the patient C) Respiratory rate of 10 breaths/min D) Pain intensity rating of 5 on a scale of 0 to 10

B) "I am enjoying my quilting group that meets each week at my church." Nurses experience secondary traumatic stress and burnout. Personal strategies to help prevent or cope with secondary traumatic stress or burnout include healthy eating, making time for yourself to pursue personal interests, getting plenty of sleep, and regular exercise and relaxation.

Which statement made by a nurse shows that the nurse is engaging in an activity to help cope with secondary traumatic stress and burnout? A) "I don't need time for lunch since I am not very hungry." B) "I am enjoying my quilting group that meets each week at my church." C) "I am going to drop my gym membership because I don't have time to go." D) "I don't know any of the other nurses who met today to discuss hospital-wide problems with nurse satisfaction."

B) Eliminating allergens Eliminating allergens is considered a tertiary prevention measure, or one that minimizes the problems with asthma and potential responses to environmental triggers, effects of asthma disease and disability. Cholesterol screening is considered a form of secondary screening, which involves measures designed to identify individuals in an early stage of a disease process so that prompt treatment can be started. Glaucoma screening is also considered a form of secondary screening. Safe sex practices are considered a form of primary prevention, or strategies aimed at optimizing health and disease prevention in general and not linked to a single disease entity.

Which tertiary prevention measure is included in the plan of care for a patient newly diagnosed with asthma? A) Cholesterol screening B) Eliminating allergens C) Glaucoma screening D) Safe sex practices

A) Using analgesics such as nonsteroidal antiinflammatory drugs (NSAIDs) along with opioids. E) The use of adjuvants (co-analgesics) such as gabapentin (Neurontin) to manage neuropathic type pain. Multimodal analgesia involves the use of a combination of drugs with at least two different mechanisms of action so pain control can be optimized. The use of acetaminophen, NSAIDs, gabapentin, and opioids represents a multimodal analgesic plan because each agent relies on a different mechanism of action to reduce pain, with the benefit of reducing the amount of opioid that is needed to control pain. This differs from polypharmacy because the combination of drugs is intentional and based on understanding of the action of each product on the pain pathway.

While caring for a patient with cancer pain, the nurse knows that a multimodal analgesia plan includes: Select all that apply. A) Using analgesics such as nonsteroidal antiinflammatory drugs (NSAIDs) along with opioids. B) Stopping acetaminophen when the pain becomes very severe. C) Avoiding polypharmacy by limiting the use of medication to one agent at a time. D) Avoiding total sedation, regardless of the severity of the pain. E)The use of adjuvants (co-analgesics) such as gabapentin (Neurontin) to manage neuropathic type pain.

A) Phantom Phantom pain occurs most often in individuals who experienced pain in the appendage or limb before the amputation. This is not a psychotic experience; it is fairly common among amputees. This type of pain could become chronic, but chronic pain is not limited to amputations. Invisible is not a term used to describe pain.

__ pain is associated with feeling pain when a limb has been amputated. A) Phantom B) Psychotic C) Chronic D)Invisible

A nurse is caring for a group of patients. Which patient will the nurse see first? a. An adult with an S4 heart sound b. A young adult with an S3 heart sound c. An adult with vesicular lung sounds in the lung periphery d. A young adult with bronchovesicular breath sounds between the scapula posteriorly

a. An adult with an S4 heart sound A fourth heart sound (S4) occurs when the atria contract to enhance ventricular filling. An S4 is often heard in healthy older adults, children, and athletes, but it is not normal in adults. Because S4 also indicates an abnormal condition, report it to a health care provider. An S3 is considered abnormal in adults over 31 years of age but can often be heard normally in children and young adults. Vesicular lungs sounds in the periphery and bronchovesicular lung sounds in between the scapula are normal findings.

During a routine physical examination of a 70-year-old patient, a blowing sound is auscultated over the carotid artery. Which assessment finding will the nurse report to the health care provider? a. Bruit b. Thrill c. Phlebitis d. Right-sided heart failure

a. Bruit A bruit is the sound of turbulence of blood passing through a narrowed blood vessel and is auscultated as a blowing sound. A bruit can reflect cardiovascular disease in the carotid artery of middle-aged to older adults. Intensity or loudness is related to the rate of blood flow through the heart or the amount of blood regurgitated. A thrill is a continuous palpable sensation that resembles the purring of a cat. Jugular venous distention, not bruit, is a possible sign of right-sided heart failure. Some patients with heart disease have distended jugular veins when sitting. Phlebitis is an inflammation of a vein that occurs commonly after trauma to the vessel wall, infection, immobilization, and prolonged insertion of IV catheters. It affects predominantly peripheral veins.

While preparing a teaching plan, the nurse describes what the learner will be able to accomplish after the teaching session about healthy eating. Which action is the nurse completing? a. Developing learning objectives b. Providing positive reinforcement c. Presenting facts and knowledge d. Implementing interpersonal communication

a. Developing learning objectives Learning objectives describe what the learner will exhibit as a result of successful instruction. Positive reinforcement follows feedback and reinforces good behavior and promotes continued compliance. Interpersonal communication is necessary for the teaching/learning process, but describing what the learner will be able to do after successful instruction constitutes learning objectives. Facts and knowledge will be presented in the teaching session.

The patient is a 45-year-old African-American male who has come in for a routine annual physical. Which type of preventive screening does the nurse discuss with the patient? a. Digital rectal examination of the prostate b. Complete eye examination every year c. CA 125 blood test once a year d. Colonoscopy every 3 years

a. Digital rectal examination of the prostate Recommended preventive screenings include a digital rectal examination of the prostate and prostate-specific antigen test starting at age 50. CA 125 blood tests are indicated for women at high risk for ovarian cancer. Patients over the age of 65 need to have complete eye examinations yearly. Colonoscopy every 10 years is recommended in patients 50 years of age and older.

A male student comes to the college health clinic. He hesitantly describes that he found something wrong with his testis when taking a shower. Which assessment finding will alert the nurse to possible testicular cancer? a. Hard, pea-sized testicular lump b. Rubbery texture of testes c. Painful enlarged testis d. Prolonged diuretic use

a. Hard, pea-sized testicular lump The most common symptoms of testicular cancer are a painless enlargement of one testis and the appearance of a palpable, small, hard lump, about the size of a pea, on the front or side of the testicle. Normally, the testes feel smooth, rubbery, and free of nodules. Use of diuretics, sedatives, or antihypertensives can lead to erection or ejaculation problems.

A nurse is teaching a patient about the Speak Up Initiatives. Which information should the nurse include in the teaching session? a. If you still do not understand, ask again. b. Ask a nurse to be your advocate or supporter. c. The nurse is the center of the health care team. d. Inappropriate medical tests are the most common mistakes.

a. If you still do not understand, ask again. If you still do not understand, ask again is part of the S portion of the Speak Up Initiatives. Speak up if you have questions or concerns. You (the patient) are the center of the health care team, not the nurse. Ask a trusted family member or friend to be your advocate (advisor or supporter), not a nurse. Medication errors are the most common health care mistakes, not inappropriate medical tests.

A nurse is preparing to teach a kinesthetic learner about exercise. Which technique will the nurse use? a. Let the patient touch and use the exercise equipment. b. Provide the patient with pictures of the exercise equipment. c. Let the patient listen to a video about the exercise equipment. d. Provide the patient with a case study about the exercise equipment.

a. Let the patient touch and use the exercise equipment. Kinesthetic learners process knowledge by moving and participating in hands-on activities. Return demonstrations and role playing work well with these learners. Patients who are visual-spatial learners enjoy learning through pictures and visual charts to explain concepts. The verbal/linguistic learner demonstrates strength in the language arts and therefore prefers learning by listening or reading information. Patients who learn through logical-mathematical reasoning think in terms of cause and effect, and respond best when required to predict logical outcomes. Specific teaching strategies could include open-ended questioning or problem solving exercises, like a case study.

A nurse is conducting Weber's test. Which action will the nurse take? a. Place a vibrating tuning fork in the middle of patient's forehead. b. Place a vibrating tuning fork on the patient's mastoid process. c. Compare the number of seconds heard by bone versus air conduction. d. Compare the patient's degree of joint movement to the normal level.

a. Place a vibrating tuning fork in the middle of patient's forehead. During Weber's test (lateralization of sound), the nurse places the vibrating tuning fork in the middle of the patient's forehead. During a Rinne test (comparison of air and bone conduction), the nurse places a vibrating tuning fork on the patient's mastoid process and compares the length of time air and bone conduction is heard. Comparing the patient's degree of joint movement to the normal level is a test for range of motion.

During a genitourinary examination of a 30-year-old male patient, the nurse identifies a small amount of a white, thick substance on the patient's uncircumcised glans penis. What is the nurse's next step? a. Record this as a normal finding. b. Avoid embarrassing questions about sexual activity. c. Notify the provider about a suspected sexually transmitted infection. d. Tell the patient to avoid doing self-examinations until symptoms clear.

a. Record this as a normal finding. A small amount of thick, white smegma sometimes collects under the foreskin in the uncircumcised male and is considered normal. Penile pain or swelling, genital lesions, and urethral discharge are signs and symptoms that may indicate sexually transmitted infections (STI). All men 15 years and older need to perform a male-genital self-examination monthly. The nurse needs to assess a patient's sexual history and use of safe sex habits. Sexual history reveals risks for STI and HIV.

A patient who is going to surgery has been taught how to cough and deep breathe. Which evaluation method will the nurse use? a. Return demonstration b. Computer instruction c. Verbalization of steps d. Cloze test

a. Return demonstration To demonstrate mastery of the skill, have the patient perform a return demonstration under the same conditions that will be experienced at home or in the place where the skill is to be performed. Computer instruction is use of a programmed instruction format in which computers store response patterns for learners and select further lessons on the basis of these patterns (programs can be individualized). Computer instruction is a teaching tool, rather than an evaluation tool. Verbalization of steps can be an evaluation tool, but it is not as effective as a return demonstration when evaluating a psychomotor skill. The Cloze test, a test of reading comprehension, asks patients to fill in the blanks that are in a written paragraph.

A nurse is describing a patient's perceived ability to successfully complete a task. Which term should the nurse use to describe this attribute? a. Self-efficacy b. Motivation c. Attentional set d. Active participation

a. Self-efficacy Self-efficacy, a concept included in social learning theory, refers to a person's perceived ability to successfully complete a task. Motivation is a force that acts on or within a person (e.g., an idea, an emotion, a physical need) to cause the person to behave in a particular way. An attentional set is the mental state that allows the learner to focus on and comprehend a learning activity. Learning occurs when the patient is actively involved in the educational session.

The nurse is preparing for a rectal examination of a nonambulatory male patient. In which position will the nurse place the patient? a. Sims' b. Knee-chest c. Dorsal recumbent d. Forward bending with flexed hips

a. Sims' Nonambulatory patients are best examined in a side-lying Sims' position. Forward bending would require the patient to be able to stand upright. Knees to chest would be difficult to maintain in a nonambulatory male and is embarrassing and uncomfortable. Dorsal recumbent does not provide adequate access for a rectal examination and is used for abdominal assessment because it promotes relaxation of abdominal muscles.

After a teaching session on taking blood pressures, the nurse tells the patient, "You took that blood pressure like an experienced nurse." Which type of reinforcement did the nurse use? a. Social acknowledgment b. Pleasurable activity c. Tangible reward d. Entrusting

a. Social acknowledgment Reinforcers come in the form of social acknowledgments (e.g., nods, smiles, words of encouragement), pleasurable activities (e.g., walks or play time), and tangible rewards (e.g., toys or food). The entrusting approach is a teaching approach that provides a patient the opportunity to manage self-care. It is not a type of reinforcement.

A patient had a stroke and must use a cane for support. A nurse is preparing to teach the patient about the cane. Which learning objective/outcome is most appropriate for the nurse to include in the teaching plan? a. The patient will walk to the bathroom and back to bed using a cane. b. The patient will understand the importance of using a cane. c. The patient will know the correct use of a cane. d. The patient will learn how to use a cane.

a. The patient will walk to the bathroom and back to bed using a cane. Outcomes often describe a behavior that identifies the patient's ability to do something on completion of teaching such as will empty a colostomy bag or will administer an injection. Understand, learn, and know are not behaviors that can be observed or evaluated.

A febrile preschool-aged child presents to the after-hours clinic. Varicella (chickenpox) is diagnosed on the basis of the illness history and the presence of small, circumscribed skin lesions filled with serous fluid. Which type of skin lesion will the nurse report? a. Vesicles b. Wheals c. Papules d. Pustules

a. Vesicles Vesicles are circumscribed, elevated skin lesions filled with serous fluid that measure less than 1 cm. Wheals are irregularly shaped, elevated areas of superficial localized edema that vary in size. They are common with mosquito bites and hives. Papules are palpable, circumscribed, solid elevations in the skin that are smaller than 1 cm. Pustules are elevations of skin similar to vesicles, but they are filled with pus and vary in size like acne.

A nurse identifies lice during a child's scalp assessment. The nurse teaches the parents about hair care. Which information from the parents indicates the nurse needs to follow up? a. We will use lindane-based shampoos. b. We will use the sink to wash hair. c. We will use a fine-toothed comb. d. We will use a vinegar hair rinse.

a. We will use lindane-based shampoos. Products containing lindane, a toxic ingredient, often cause adverse reactions; the nurse will need to follow up to correct the misconception. All the rest are correct. Instruct parents who have children with head lice to shampoo thoroughly with pediculicide (shampoo available at drugstores) in cold water at a basin or sink, comb thoroughly with a fine-toothed comb, and discard the comb. A dilute solution of vinegar and water helps loosen nits.

A parent calls the school nurse with questions regarding the recent school vision screening. Snellen chart examination revealed 20/60 for both eyes. Which response by the nurse is the best regarding the eye examination results? a. Your child needs to see an ophthalmologist. b. Your child is suffering from strabismus. c. Your child may have presbyopia. d. Your child has cataracts.

a. Your child needs to see an ophthalmologist. The child needs an eye examination with an ophthalmologist or optometrist. Normal vision is 20/20. The larger the denominator, the poorer the patient's visual acuity. For example, a value of 20/60 means that the patient, when standing 20 feet away, can read a line that a person with normal vision can read from 60 feet away. Strabismus is a (congenital) condition in which both eyes do not focus on an object simultaneously: The eyes appear crossed. Acuity may not be affected; Snellen test does not test for strabismus. Presbyopia is impaired near vision that occurs in middle-aged and older adults and is caused by loss of elasticity of the lens. Cataracts, a clouding of the lens, develop slowly and progressively after age 35 or suddenly after trauma.

When planning to evaluate a patient's satisfaction with a teaching activity, the most appropriate strategy would be to a. include a survey instrument. b. observe for level of skill mastery. c. present information more than one time. d. provide for a return demonstration.

a. include a survey instrument. A survey or questionnaires can be used to measure affective behavior change as well as patient satisfaction with the teaching experience. Observing for level of skill mastery would evaluate achievement of a psychomotor goal rather than satisfaction with the experience. Repeating information more than one time or in more than one way may be appropriate strategies to include in the teaching plan but would provide no evaluation data. Providing for a return demonstration would help in evaluating achievement of a psychomotor goal, not satisfaction with the activity.

A patient reports a change in the usual pattern of urination. What question does the nurse ask to determine if incontinence is the reason for these symptoms? a. "Do you have the feeling that you cannot wait to urinate?" b. "Are you urinating a large amount each time you go to the bathroom?" c. "Has the color of your urine changed lately?" d. "Have you noticed any swelling in your ankles at the end of the day?"

a. "Do you have the feeling that you cannot wait to urinate?" A "Do you have the feeling that you cannot wait to urinate?" is a question that asks about urgency, a symptom of incontinence. B Are you urinating a large amount each time you go to the bathroom?" is not a question related to incontinence. Usually patients with incontinence void frequently in small amounts. C Has the color of your urine changed lately?" is a question that is asked when the nurse suspects the patient has gallbladder or liver disease. D Have you noticed any swelling in your ankles at the end of the day?" is a question that relates to patients who have renal or heart disease.

During the problem-based history, a patient reports coughing up sputum when lying on the right side, but not when lying on the back or left side. The nurse suspects this patient may have a lung abscess. What additional question does the nurse ask to gather more data? a. "Does the sputum have an odor?" b. "Do you have chest pain when you take a deep breath?" c. "Have you also experienced tightness in your chest?" d. "Have you coughed up any blood?"

a. "Does the sputum have an odor?" A Sputum with odor and sputum production with change of position is associated with lung abscess or bronchiectasis. B Chest pain on deep breathing is associated with pleural lining irritation. C Tightness in the chest is associated with asthma. D Coughing up rust-colored sputum is associated with pneumonia, but coughing up blood may be associated with lung cancer.

During symptom analysis, the nurse helps the patient distinguish between dizziness and vertigo. Which description by the patient indicates dizziness? a. "I felt faint, like I was going to pass out." b. "It felt like I was on a merry-go-round." c. "The room seemed to be spinning around." d. "My body felt like it was revolving and could not stop."

a. "I felt faint, like I was going to pass out." A This is a description of lightheadedness, a form of dizziness. B This is consistent with objective vertigo because it includes a sensation of motion. C This is consistent with objective vertigo because it includes a sensation of motion. D This is consistent with subjective vertigo because it includes a sensation of one's body rotating in space.

While obtaining a symptom analysis from a patient who had a transient ischemic attack, the nurse helps the patient distinguish between dizziness and vertigo. Which description by the patient indicates dizziness? a. "I felt lightheaded when I stood up." b. "It felt like I was on a merry-go-round." c. "The room seemed to be spinning around." d. "My body felt like it was revolving and could not stop."

a. "I felt lightheaded when I stood up." A This is a description of dizziness that is often associated with transient ischemia attacks. B This report is consistent with objective vertigo because it includes a sensation of motion. C This report is consistent with objective vertigo because it includes a sensation of motion. D This report is consistent with subjective vertigo because it includes a sensation of one's body rotating in space.

Which statement or question does the nurse use during the introduction phase of the interview? a. "I'm here to learn more about the pain you're experiencing." b. "Can you describe the pain that you're experiencing?" c. "I heard you say that the pain is 'all over' your body." d. "What relieves the pain you are having?"

a. "I'm here to learn more about the pain you're experiencing." A "I'm here to learn more about the pain you're experiencing" is an example of the introduction phase when the nurse tells the patient the purpose of the interview. B "Can you describe the pain that you're experiencing?" is an example of part of a symptom analysis that occurs in the discussion phase. C "I heard you say that the pain is 'all over' your body" is an example of a summary statement by the nurse that occurs in the summary phase. D "What relieves the pain you are having?" is an example of part of a symptom analysis that occurs in the discussion phase.

What instruction does a nurse give a patient to facilitate palpation of the right lobe of the thyroid gland? a. "Swallow for me one time." b. "Flex your head down and to the left." c. "Rotate your head to the right for me." d. "Hold your breath for a few seconds."

a. "Swallow for me one time." A The patient is asked to swallow to make the thyroid lobe easier to palpate. B This is incorrect to palpate the right lobe. The patient flexes the neck toward the side being palpated. C This is incorrect to palpate the right lobe. The patient flexes the neck toward the side being palpated, but does not rotate the head. D This is not part of the thyroid palpation.

During the history, the patient states that she does not use many drugs. What is the nurse's appropriate response to this statement? a. "Tell me about the drugs you are using currently." b. "To some people six or seven is not many." c. "Do you mean prescription drugs or illicit drugs?" d. "How often are you using these drugs?"

a. "Tell me about the drugs you are using currently." A "Tell me about the drugs you are using currently" is an open-ended question that allows patients to provide further data. B "To some people six or seven is not many" is a comment that does not ask a question or obtain useful data. C "Do you mean prescription drugs or illicit street drugs?" is a closed-ended question that yields data about the type of drugs used only. D "How often are you using these drugs?" asks about frequency of drug use, which is not useful until the drugs are known.

A nurse is interviewing a patient who was diagnosed with type 2 diabetes mellitus 6 months ago. Since that time, the patient has gained weight and her blood glucose levels remain high. The nurse suspects that the patient is noncompliant with her diet. Which response by the nurse enhances data collection in this situation? a. "Tell me about what foods you eat and the frequency of your meals" b. "What symptoms do you notice when your blood sugar levels are high?" c. "You need to follow what the doctor has prescribed to manage your disease" d. "Tell me what you know about the cause of type 2 diabetes."

a. "Tell me about what foods you eat and the frequency of your meals" A "Tell me about what foods you eat and the frequency of your meals" gathers more data from the patient to help the nurse confirm if noncompliance is the reason for the weight gain and high glucose levels. B "What symptoms do you notice when your blood sugar levels are high?" does not help the nurse determine if the patient is noncompliant. It may be useful later when teaching the patient about her disease. C "You need to follow what the doctor has prescribed to manage your disease" does not provide additional data for the nurse and may be viewed as authoritarian and paternalistic. D "Tell me what you know about the cause of type 2 diabetes" assumes that the reason for the weight gain and high glucose levels is a lack of knowledge. A more therapeutic approach is to gather more data from the patient about how the diabetes has been managed.

During a physical examination, a 12-year-old girl expresses concern to the nurse that her breasts are different sizes. Which response is most appropriate for the nurse? a. "This happens normally to many girls your age. Full breast development takes an average of 3 years." b. "I can talk with your mother about a referral to a physician who can perform further examination and tests." c. "Have you started your menstrual period yet, because breast development is irregular until menstruation begins?" d. "This is called 'precocious breast development' and your breasts will become more equal just before your growth spurt starts."

a. "This happens normally to many girls your age. Full breast development takes an average of 3 years." A The right and left breasts may develop at different rates. It is important to reassure the patient that this is common and, in time, the development may equalize. Full development of the breast takes an average of 3 years (range 1.5 to 6 years). B Breasts may develop at different rates. C Menarche begins when the breasts reach Tanner stages 3 or 4, which is approximately age 12. D Precocious breast development is the development of breasts before 8 years old.

During an interview, a patient begins to cry and appears angry. Which response by the nurse is most therapeutic? a. "This topic prompted an emotional response, tell me what you are feeling." b. "This topic does not usually cause such an emotional response." c. "Calm down and tell me what is wrong." d. "I will leave you alone for a few minutes so you can pull yourself together."

a. "This topic prompted an emotional response, tell me what you are feeling." A Acknowledging the patient's feelings and encouraging their expression communicates acceptance of the emotion. Crying is a natural behavior and should be permitted. B "This topic does not usually cause such an emotional response" may be perceived by the patient as judgmental and it does not help the patient meet the current need. C Encouraging the patient to stop crying so that the nurse can help is not supportive of the patient's current need. The therapeutic action is to postpone further questioning until the patient is ready to proceed. D Leaving the room so that the patient can be alone is not supportive of the patient.

In assessing a patient's deep tendon reflexes, a nurse finds a patient has a 4+ triceps response. How does the nurse interpret this finding? a. A hyperactive response b. A diminished response c. An absent response d. An expected response

a. A hyperactive response A Deep tendon reflexes are graded from 0 to 4+ and 4+ is a hyperactive response. B A diminished response is 1+. C An absent response is 0. D An expected response is 2+.

After taking a brief health history, a nurse needs to complete a focused assessment on which patient? a. A male who works as a painter b. A male who plays basketball and hockey c. A female who recently moved into a college dormitory d. A female who has a history of gout

a. A male who works as a painter A The fumes and chemicals from the paint may expose the patient to respiratory irritants. A baseline pulmonary assessment needs to be documented. B This patient is not at risk for pulmonary disease. C This patient is not at risk for pulmonary disease. D This patient is not at risk for pulmonary disease.

A nurse shines the light from the ophthalmoscope into the eyes of a newborn and observes a bright, round, red-orange glow seen through both pupils. How does the nurse document this finding? a. An expected red reflex b. Eyelid capillary hemangiomas c. Bilateral conjunctivitis d. Ophthalmia neonatorum

a. An expected red reflex A The red reflex appears as a bright, round, red-orange glow seen through both pupils. B Eyelid capillary hemangiomas are abnormalities that appear on the eyelid, but disappear spontaneously. C Redness, lesions, nodules, discharge, or crusting of conjunctiva indicate conjunctivitis. D Ophthalmia neonatorum is an eye disorder that produces purulent conjunctivitis and keratitis.

Which activity illustrates the concept of secondary prevention? a. Annual mammogram b. Nutrition classes on low-fat cooking c. Education on living with diabetes mellitus d. Cardiac rehabilitation after coronary artery bypass surgery

a. Annual mammogram A A mammogram screens for breast cancer and is an example of secondary prevention to promote early detection of disease. B Nutrition classes are an example of primary prevention to prevent a disease from developing by promoting a healthy lifestyle. C Education about diabetes mellitus is an example of tertiary prevention directed toward minimizing the disability from chronic disease and helping the patient maximize his or her health. D Cardiac rehabilitation after coronary artery bypass surgery is an example of tertiary prevention directed toward minimizing the disability from chronic disease and helping the patient maximize his or her health.

A patient reports having difficulty swallowing. Based on this information, how does the nurse assess the appropriate cranial nerve? a. Ask the patient to stick out the tongue and move it in all directions. b. Ask the patient to move the head to the right and left. c. Observe the symmetry of the face when the patient talks. d. Assess for taste on the anterior part of the tongue.

a. Ask the patient to stick out the tongue and move it in all directions. A This tests the hypoglossal cranial nerve (CN XII) that is involved in swallowing. The nurse must correlate difficulty swallowing with the cranial nerves involved with that function and how to test them. The cranial nerves involved are IX, X, and XII. B This tests the function of the spinal accessory cranial nerve (CN XI). C This tests the motor function of the facial cranial nerve (CN VII). D This tests the sensory portion of the facial cranial nerve (CN VII).

How does a nurse respond to parents of a 5-year-old who are worried that their child has a protruding abdomen? a. Assesses the child to differentiate a normal "potbelly" from a hernia b. Suggests that the parents administer an appropriate dose of a laxative at bedtime c. Refers the parents to a nutritionist to develop an appropriate weight-loss diet for the child d. Informs the parents that a protruding abdomen is always an abnormal finding in this age group

a. Assesses the child to differentiate a normal "potbelly" from a hernia A Toddlers normally exhibit a rounded (potbelly) abdomen while both standing and lying down. B The nurse needs to collect more data before making this recommendation. C The nurse needs to collect more data before making this recommendation. D This is an inappropriate statement based on the description of this child.

. What instructions does the nurse give a patient before palpating the abdomen? a. Bend the knees. b. Take a deep breath and hold it. c. Take a deep breath and cough. d. Place the hands over the head.

a. Bend the knees. A Bend the knees to relax the abdominal muscles. B This action is not needed to assess the abdomen. C This action is used to detect bulges in the abdomen, but not used before palpation. D This action is not needed to assess the abdomen.

Which finding rules out defects in the cornea, lens, and vitreous chamber of an infant? a. Bilateral red reflex b. Symmetric corneal light reflex c. Bilateral blink reflex d. Symmetric eye movements

a. Bilateral red reflex A Presence of the red reflex eliminates the presence of most serious defects of the cornea, aqueous chamber, lens, and vitreous chamber. B Symmetric corneal light reflex tests for symmetric eye muscle function and eliminates the presence of strabismus. C Bilateral blink reflex tests the function of cranial nerve V (trigeminal). D Symmetry of extraocular muscles tests cranial nerves III (oculomotor), IV (trochlear), and VI (abducens).

A patient has right lower lobe pneumonia, creating a consolidation in that lung. In assessing for vocal fremitus, the nurse found increased fremitus over the right lower lung. What finding does the nurse anticipate when assessing vocal resonance to confirm the consolidation? a. Bronchophony reveals the patient's spoken "99" as clear and loud. b. No sounds are expected since sounds cannot be transmitted through consolidation. c. Egophony reveals indistinguishable sounds when the patient says "e-e-e." d. Whispered pectoriloquy reveals a muffled sound when the patient says "1-2-3."

a. Bronchophony reveals the patient's spoken "99" as clear and loud. A This is an abnormal finding and occurs in consolidation. B The abnormal finding is hearing a clear sound. C This is a normal finding. D This is a normal finding.

A patient complains of itching, swelling, and drainage from the eyes with a postnasal drip and sneezing. What type of nasal drainage does the nurse anticipate seeing during inspection of this patient's nares? a. Clear b. Malodorous c. Yellow d. Green

a. Clear A The patient has allergic rhinitis, which produces clear drainage. B Malodorous drainage is associated with bacterial infection, which is not consistent with the history given by this patient. C Yellow drainage is associated with bacterial infection, which is not consistent with the history given by this patient. D Green drainage is associated with bacterial infection, which is not consistent with the history given by this patient.

After collecting the data, the nurse begins data analysis with which action? a. Clustering data b. Documenting subjective data c. Reporting information to other health team members d. Documenting objective information

a. Clustering data A After collecting data, the nurse organizes or clusters the data so that the problems appear more clearly. To cluster data, the nurse interprets the assessment data collected. B Documenting subjective data is necessary for the medical record, but does not provide analysis. C Before reporting data to health team members, the nurse clusters and interprets data. D Documenting objective data is necessary for the medical record, but does not provide analysis.

When inspecting a patient's nasal mucous membrane, which finding does the nurse expect to see? a. Deep pink turbinates b. Red, edematous mucous membranes c. Septum that angles to the left d. Clear exudate

a. Deep pink turbinates A These are expected for a nasal inspection. B These indicate a local infection within the nose. C This is abnormal. D This occurs with nasal allergies.

During the history, a patient reports watery nasal drainage from allergies. Based on this information, what does the nurse expect to find on inspection of the nares? a. Enlarged and pale turbinates b. Polyps within the nares c. High vascularity of the turbinates d. Dry and dull turbinates

a. Enlarged and pale turbinates A Enlarged and pale turbinates are expected findings for allergic rhinitis. B Polyps within the nares is not an expected finding. C High vascularity of the turbinates is not an expected finding. D Dry and dull turbinates is not an expected finding.

Which behavior would be most indicative of hearing impairment in a 1-year-old child? a. Failure to respond to mother's voice b. Crying when a loud noise occurs unexpectedly c. Saying only single-syllable words d. Disinterest in playing with musical toys

a. Failure to respond to mother's voice A By age 4 to 6 months, an infant should turn the head toward the source of the sound and should respond to the parent's voice. B This behavior indicates an ability to hear loud noises. C This is an expected finding for a child this age. D Although an infant by age 4 to 6 months should respond to music toys, a disinterest does not indicate a hearing problem.

A nurse assesses a patient with a head injury who has slowing intellectual functioning, personality changes, and emotional lability. The nurse correlates these findings with which area of the brain? a. Frontal lobe b. Parietal lobe c. Thalamus d. Temporal lobe

a. Frontal lobe A The frontal lobe controls intellectual function, awareness of self, personality, and autonomic responses related to emotion. B The parietal lobe receives sensory input such as position sense, touch, shape, and texture of objects. C The thalamus is a relay and integration station from the spinal cord to the cerebral cortex and other parts of the brain. D The temporal lobe contains the primary auditory cortex. It also interprets auditory, visual, and somatic sensory inputs that are stored in thought and memory.

A patient tells the nurse, "I've been having pain in my belly for several days that gets worse after eating." Which datum from the symptom analysis is consistent with the nurse's suspicion of peptic ulcer disease? a. Gnawing epigastric pain radiates to the back or shoulder that worsens after eating. b. Sharp midepigastric pain radiates to the jaw. c. Intermittent cramping pain in the left lower quadrant is relieved by defecation. d. Colicky pain is felt near the umbilicus with vomiting and constipation.

a. Gnawing epigastric pain radiates to the back or shoulder that worsens after eating. A Gnawing epigastric pain that radiates to the back or shoulder and worsens after eating is a symptom that is consistent with peptic ulcer disease. B Sharp midepigastric pain that radiates to the jaw is not a symptom of peptic ulcer disease. C Intermittent cramping pain in the left lower quadrant relieved by defecation is a symptom of diverticular disease rather than peptic ulcer disease. D Colicky pain felt near the umbilicus with vomiting and constipation is a symptom of an intestinal obstruction rather than peptic ulcer disease.

What is the patient's expected response when the nurse is assessing stereognosis? a. Identifies an object placed in the hand b. Distinguishes numbers or letters traced in the palm of the hand c. Touches the index finger of the nondominant hand to the nose d. Walks heel to toe in a straight line

a. Identifies an object placed in the hand A A nurse tests stereognosis by asking the patient to close his or her eyes and placing a small, familiar object in the patient's hand, asking him or her to identify it. Stereognosis tests sensory nerve tracts and parietal lobe function. B This activity tests graphesthesia, a test of sensory nerve tracts and parietal lobe function. C This activity tests cerebellar function of the upper extremities. D This activity tests cerebellar function of the lower extremities.

The nurse recognizes which clinical finding as expected on palpation of the abdomen? a. Inability to palpate the spleen b. Left kidney rounded at 2 cm below the costal margin c. Slight tenderness of the gallbladder on light palpation d. Bounding pulsation of the aorta over the umbilicus

a. Inability to palpate the spleen A Inability to palpate the spleen is the expected finding on palpating the abdomen. B A rounded left kidney at 2 cm below the costal margin is not an expected finding. Kidneys are usually not palpated. C Slight tenderness of the gallbladder on light palpation is not an expected finding; the gallbladder is usually not palpable. D Bounding pulsation of the aorta over the umbilicus would be an abnormal finding, perhaps indicating an aneurysm.

After assessment of the nose and paranasal sinuses, which finding requires further investigation by the nurse? a. Nasal septum off the midline b. Nose in the midline of the face c. Middle turbinates deep pink in color d. Noiseless exchange of air from each naris

a. Nasal septum off the midline A A deviated septum is an abnormal finding that needs further investigation. B This is an expected finding. C This is a normal finding. D This is a normal finding.

A patient reports having migraine headaches on one side of the head that often start with an aura and last 1 to 3 days. As a part of the symptom analysis, the patient reports which associated symptoms of migraine headaches? a. Nausea, vomiting, or visual disturbances b. Nasal stuffiness or discharge c. Ringing in the ears or dizziness d. Red, watery eyes or drooping eyelids

a. Nausea, vomiting, or visual disturbances A These are symptoms associated with migraine headaches. B This is a symptom associated with cluster headaches rather than migraine headaches. C These symptoms are not associated with migraine headaches. D These are symptoms associated with cluster headaches rather than migraine headaches.

Which response does a nurse expect when testing ankle clonus of a healthy woman? a. No movement of the foot b. Plantar flexion of the foot c. Extension of the lower leg d. Dorsiflexion of the foot

a. No movement of the foot A No movement of the foot is the expected response from a healthy woman. B Plantar flexion of the foot is not a response to ankle clonus. C Extension of the lower leg is not a response to ankle clonus. D Dorsiflexion of the foot is an abnormal response of ankle clonus.

How does a nurse assess the competence of venous valves in patients who have varicose veins? a. Notes how quickly veins fill after lifting one leg above the level of the heart b. Assesses for Homan sign in both lower extremities while the patient is supine c. Assesses capillary refill on the toes of both feet while the patient is sitting in the chair d. Measures the circumference of both calves and compares the results

a. Notes how quickly veins fill after lifting one leg above the level of the heart A Noting how quickly veins fill after lifting one leg above the level of the heart is the procedure to test for incompetent veins. B Homan sign is an unreliable test for deep vein thrombosis. C Assessing capillary refill assesses perfusion (blood flow from arteries) rather than competence of venous valves. D Measuring the circumference of both calves and comparing the results is used to assess deep vein thrombosis.

Which nurse demonstrates culturally competent care for a female patient from Russia? a. Nurse A who asks the patient about cultural factors that influence health care b. Nurse B who interacts with every patient from Russia in the same manner c. Nurse C who learns the cultural variables of every culture, including Russia d. Nurse D who relies on her previous experience with patients from Russia

a. Nurse A who asks the patient about cultural factors that influence health care A Asking the patient about cultural factors that influence health care is demonstrating culturally competent care, along with interacting with each patient as a unique person who is a product of past experiences, beliefs, and values. B Interacting with every patient from Russia in the same manner does not allow for the uniqueness of each person within the same culture. C Learning the cultural variables of every group encountered can be valuable but it is impractical to learn about all cultures because each patient is unique. A better approach is to ask patients about their beliefs. D Relying on previous experience with patients from Russia does not allow for the uniqueness of each person within the same culture.

Which cranial nerve is assessed by using the Snellen visual acuity chart? a. Optic cranial nerve (CN II) b. Oculomotor cranial nerve (CN III) c. Abducens cranial nerve (CN IV) d. Trochlear cranial nerve (CN VI)

a. Optic cranial nerve (CN II) A The optic cranial nerve (CN II) provides vision tested by the Snellen visual acuity chart. B CN III controls pupillary constriction, eyelid movement, and eyeball movement. C CN IV controls eyeball movement. D CN VI controls eyeball movement.

A nurse notices a patient's chest wall moving in during inspiration and out during expiration. What additional assessment must the nurse perform immediately? a. Palpate for tracheal deviation. b. Auscultate for bronchovesicular breath sounds in the lung periphery. c. Palpate posterior thoracic muscles for tenderness. d. Auscultate for absence of breath sounds in the lung periphery.

a. Palpate for tracheal deviation. A Chest wall moving in during inspiration and out during expiration is paradoxical chest wall movement. It can be caused by a tension pneumothorax, which increases intrathoracic pressure in the thorax, causing tracheal deviation and indicating mediastinal shift. B Tension pneumothorax does not create bronchovesicular breath sounds in the lung periphery. C This is performed when the patient has air in the subcutaneous tissue or pleural friction rub. D Absent breath sounds may be found in pneumothorax, but if the patient has a tension pneumothorax, tracheal deviation is a more important sign.

A nurse notices abdominal distention when inspecting a patient's abdomen. What action does the nurse take next to gain further objective data? a. Place a measuring tape around the superior iliac crests. b. Assist the patient to turn on to the left side and then the right side. c. Ask the patient to cough while lying supine. d. Use the fingertips to sharply strike one side of the abdomen.

a. Place a measuring tape around the superior iliac crests. A This is the procedure for measuring abdominal girth. B This procedure is unnecessary. The distention will remain in a side-lying position. C Having the patient cough is used to assess for bulges rather than distention. D This is part of the procedure to test for a fluid wave, which is not indicated in this patient.

What does the nurse teach to parents to prevent sudden infant death syndrome (SIDS)? a. Place the baby on back to sleep. b. Place the baby on side to sleep. c. Not to feed the baby for 3 hours before sleep. d. Place the baby on her stomach to sleep.

a. Place the baby on back to sleep. A The American Academy of Pediatrics recommends positioning infants on their backs; the slogan to help people remember is "Back to Sleep." B The side-lying position is not recommended for sleep because of the risk of aspiration. C Not feeding the baby for 3 hours before sleep is not a prevention for SIDS. D The prone position is not recommended for sleep due to the risk of aspiration.

An adult patient comes to the clinic complaining of right ear pain. What technique does the nurse use to inspect this patient's auditory canal? a. Position the otoscope speculum 1.0 to 1.5 cm (about 0.5 inches) into the ear canal. b. Remove cerumen from each canal before inserting otoscope. c. Choose the smallest otoscope speculum that will fit the patient's ear comfortably. d. Pull the pinna slightly downward and backward before inserting the otoscope speculum.

a. Position the otoscope speculum 1.0 to 1.5 cm (about 0.5 inches) into the ear canal. A This is the correct technique. B Removing cerumen is not necessary. C The largest speculum that comfortably fits in the ear canal is the one that should be chosen. D For adults, the pinna is pulled up and backward to straighten the ear canal.

When inspecting a patient's eyes, the nurse assesses the presence of cranial nerve III (oculomotor nerve) by observing the eyelids open and close bilaterally. What other technique does a nurse use to test the function of this cranial nerve? a. Pupillary constriction to light b. Visual acuity c. Peripheral vision d. Presence of the red reflex

a. Pupillary constriction to light A Cranial nerve III (oculomotor) controls pupillary dilation and constriction, as well as eyelid movement. Pupil dilation and ptosis may occur when CN III is impaired. B Cranial nerve II (optic) provides vision. C Cranial nerve II (optic) provides peripheral vision. D The red reflex is not controlled by cranial nerve III, but is created by a light illuminating the retina.

On palpation the nurse determines that the patient's left thyroid lobe is larger than the right thyroid lobe. What is the nurse's most appropriate action at this time? a. Refer the patient to the health care provider for further evaluation. b. Document that the patient's thyroid is normal on palpation. c. Palpate the left thyroid lobe again using very firm pressure. d. Ask the patient to flex the chin toward his chest and palpate again.

a. Refer the patient to the health care provider for further evaluation. A The nurse found an abnormality that needs referral for follow-up. B This is not an appropriate action because the nurse found an abnormality. C Repeating the examination will yield the same abnormal finding. D Repeating the examination will yield the same abnormal finding.

A patient reports that he has coronary artery disease with ventricular hypertrophy. Based on these data, what finding should the nurse expect during assessment? a. S4 heart sound b. Clubbing of fingers c. Splitting of the S1 heart sound d. Pericardial friction rub

a. S4 heart sound A An S4 heart sound signifies a noncompliant or "stiff'' ventricle. Coronary artery disease is a major cause of a stiff ventricle. B Clubbing of fingers occurs due to chronic hypoxia rather than a stiff ventricle. C Splitting of the S1 heart sound indicates a valve problem rather than ventricular hypertrophy. When the mitral and tricuspid valves do not close at the same time, S1 sounds as if it were split into two sounds instead of one. D Pericardial friction rubs are caused by inflammation of the layers of the pericardial sac.

A nurse learns from a report that a patient has aortic stenosis. Where does the nurse place the stethoscope to hear this stenotic valve? a. Second intercostal space, right sternal border b. Second intercostal space, left sternal border c. Fourth intercostal space, left sternal border d. Fifth intercostal space, left midclavicular line

a. Second intercostal space, right sternal border A Second intercostal space, right sternal border is the location for listening to the aortic valve. B Second intercostal space, left sternal border is the location for listening to the pulmonic valve. C Fourth intercostal space, left sternal border is the location for listening to the tricuspid valve. D Fifth intercostal space, left midclavicular line is the location for listening to the mitral valve.

A nurse determines that a patient has a heart rate of 42 beats per minute. What might be a cause of this heart rate? a. Sinoatrial (SA) node failure b. Atrial bradycardia c. A well-conditioned heart muscle d. Left ventricular hypertrophy

a. Sinoatrial (SA) node failure A If the SA node is ineffective, the atrioventricular node may initiate contraction, but at a rate of 40 to 60 beats/min. B The heart rate reflects the ventricular rate rather than the atrial rate. C Although well-conditioned athletes may have slower heart rates, this rate is too slow for even an athlete. D Left ventricular hypertrophy alters the strength of contraction rather than the heart rate.

Which patient information does the nurse document in the patient's physical assessment? a. Slurred speech b. Immunizations c. Smoking habit d. Allergies

a. Slurred speech A Slurred speech should be noticed by the nurse and documented as objective data in the physical assessment. B Data on immunizations are collected from the patient, are subjective, and documented in the history. C A smoking habit is information that comes from the patient, making it subjective data that is documented in the history. D Allergies are information that come from the patient, making it subjective data that is documented in the history.

On palpation of the left upper quadrant of the abdomen of a female patient, the nurse notes tenderness. This finding may indicate a disorder in which organ? a. Spleen b. Gallbladder c. Sigmoid colon d. Left ovary

a. Spleen A The spleen is located in the left upper quadrant of the abdomen. B The gallbladder is located in the right upper quadrant of the abdomen. C The sigmoid colon is located in the left lower quadrant of the abdomen. D The left ovary is located in the left lower quadrant of the abdomen.

While giving a history, a male patient describes several events out of order that occurred in different decades in his life. What technique does the nurse use to understand the timeline of these events? a. State the order of events as understood and ask the patient to verify the order. b. Draw conclusions about the order of events from data given. c. Ask the patient to elaborate about these events. d. Ask the patient to repeat what he said about these events.

a. State the order of events as understood and ask the patient to verify the order. A State the order of events as understood and ask patient to verify the order is correct. This technique is useful when interviewing a patient who rambles or does not provide sequential data. B Drawing conclusions about the order of events is interpretation. In this example, the sequence of events is more relevant than an interpretation. The nurse may have difficulty interpreting an unclear sequence of events. C Asking the patient to elaborate about these events will not provide order to the sequence of events. D Asking the patient to repeat what he said about these events will not necessarily provide a sequence of events.

A nurse asks the patient to stand with feet together, arms resting at the sides, with eyes open and then with the eyes closed. Which response by the patient indicates an expected cerebellar function? a. Sways slightly and maintains upright posture with feet together b. Is unable to stand upright after turning around in a circle once c. Steps sideways when standing with feet together and eyes closed d. Has to move arms horizontally to maintain balance

a. Sways slightly and maintains upright posture with feet together A Maintaining balance indicates function of the cerebellum in the Romberg test. B Losing balance is an abnormal response, but turning in a circle is not a part of the Romberg test. C This is an abnormal response for the Romberg test (a positive Romberg test). D This is an abnormal response for the Romberg test (a positive Romberg test).

When using an ophthalmoscope to examine the internal eye, how does the nurse distinguish the retinal arteries from the retinal veins? a. The arteries are narrower than veins. b. The arteries are a darker red than veins. c. The arteries have no light reflex and the veins have a narrow band of light in the center. d. The arteries have prominent pulsations and veins have no pulsations.

a. The arteries are narrower than veins. A The artery-to-vein width should be 2:3 to 4:5. B Arteries are lighter red than veins. C Arteries have a narrow band of light in the center and veins have no light reflex. D Arteries show little to no pulsations and venous pulsations may be visible.

In inspecting the eyes and ears of an infant, the nurse documents which finding as normal? a. The external ear is in direct line with the outer margin of the eyelid. b. The ear lobe is within 10° of alignment with the outer margin of the eyelid. c. A lateral upward slant of the eyes aligns them with the helix of the ear. d. The inner margin of the eye is directly aligned with the helix of the ear.

a. The external ear is in direct line with the outer margin of the eyelid. A This is the expected alignment of the ears and eyes of an infant. B This is not an expected finding. C This finding occurs in infants with Down syndrome. D This is not an expected finding.

A nurse shines a light toward the bridge of the patient's nose and notices that the light reflection in the right cornea is at the 9 o'clock position and in the left cornea at the 9 o'clock position. What is the interpretation of this finding? a. The extraocular muscles of both eyes are intact. b. The cornea of each eye is transparent. c. The sclera of each eye is clear. d. The consensual reaction of both eyes is intact.

a. The extraocular muscles of both eyes are intact. A The reflection of the light in both eyes in the same location indicates muscles holding the eyes are symmetric. B The reflection of the light in both eyes in the same location indicates muscles holding the eyes are symmetric. C The reflection of the light in both eyes in the same location indicates muscles holding the eyes are symmetric. D Consensual reaction involves constriction of pupils.

Which finding indicates that this patient has a sensorineural hearing loss? a. The patient hears sound by air conduction longer than by bone conduction. b. The patient hears sound from a vibrating tuning fork in the affected ear only. c. The patient hears normal conversation at 40 dB and a whisper at 20 dB. d. The patient hears the rubbing of fingers together from a distance of 4 inches from each ear.

a. The patient hears sound by air conduction longer than by bone conduction. A In the Rinne test, hearing sound from a vibrating tuning fork longer by air conduction than by bone conduction is consistent with a sensorineural hearing loss. B This finding from the Weber test is consistent with a conduction hearing loss. C This is an expected finding using audiometry. D This is an expected finding using the finger rubbing screening hearing test.

A nurse assessing a patient who had a cerebrovascular accident involving the Broca area suspects expressive or nonfluent aphasia. What communication abilities does the nurse anticipate from this patient? a. The patient understands speech but is unable to translate ideas into meaningful speech. b. The patient is unable to comprehend speech and thus does not respond verbally. c. The patient is able to understand speech but has difficulty forming words, creating muffled speech. d. The patient is unable to comprehend speech and responds inappropriately to conversation.

a. The patient understands speech but is unable to translate ideas into meaningful speech. A The inability to translate ideas into meaningful speech or writing is termed expressive aphasia or nonfluent aphasia and is associated with lesions in the Broca area in the frontal lobe. B The inability to comprehend the speech of others is called receptive aphasia or fluent aphasia and is associated with lesions in the Wernicke area in the temporal lobe. C This speech pattern is more consistent with patients who have involvement of muscles of speech rather than neurologic deficits. D This speech pattern is not relevant to this patient.

Which patient should the nurse assess first? a. The patient whose respiratory rate is 26 breaths per minute and whose trachea deviates to the right. b. The patient who has pleuritic chest pain, bilateral crackles, a productive cough of yellow sputum, and fever. c. The patient who is short of breath, using pursed-lip breathing, and in a tripod position. d. The patient whose respiratory rate is 20 breaths/min, and has 8-word dyspnea and expiratory wheezes.

a. The patient whose respiratory rate is 26 breaths per minute and whose trachea deviates to the right. A This is a description of a left tension pneumothorax. The key manifestation is deviation of the trachea from midline, which indicates high intrathoracic pressure from the left that is pushing the mediastinum out of alignment. The respiratory rate indicates tachypnea. B This is a description of a patient with pneumonia who needs to be examined, but this is not a life-threatening condition. C This is a description of a patient with emphysema, a chronic disease. This patient may have these manifestations frequently and does not need to be examined immediately. D This is a description of a patient who is having an asthma attack, but it is not a life threatening attack; the respiratory rate is the upper limits of normal; the dyspnea is abnormal, but not far from normal; and the wheezing is on expiration only.

Which is an example of data a nurse collects during a physical examination? a. The patient's lack of hair and shiny skin over both shins b. The patient's stated concern about lack of money for prescriptions c. The patient's complaints of tingling sensations in the feet d. The patient's mother's statements that the patient is very nervous lately

a. The patient's lack of hair and shiny skin over both shins A The lack of hair and shiny skin over both shins are objective data or signs that are part of the physical examination B A patient's concerns about lack of money are subjective data and are part of the health history. C A patient's complaints of tingling sensations in the feet are subjective data and are part of the health history. D A patient's family statements are considered secondary data, are subjective data, and are part of the health history.

For which person is a shift assessment indicated? a. The person who had abdominal surgery yesterday b. The person who is unaware of his high serum glucose levels. c. The person who is being admitted to a long-term care facility. d. The person who is beginning rehabilitation after a knee replacement.

a. The person who had abdominal surgery yesterday A A shift assessment is most appropriate for the person who is recovering in the hospital from surgery. B A screening assessment is performed for the purpose of disease detection, in this case diabetes mellitus. C A comprehensive assessment is performed during admission to a facility to obtain a detailed history and complete physical examination. D An episodic or follow-up assessment is performed after knee replacement to evaluate the outcome of the procedure.

Which sound does a nurse expect to hear when percussing a patient's abdomen? a. Tympany over all quadrants b. Resonance over the upper quadrants and tympany in the lower quadrants c. Dull sounds over the upper quadrants and hollow sounds over the lower quadrants d. Dull sounds over the stomach and resonant sounds over the bladder

a. Tympany over all quadrants A Tympany over all quadrants is a normal finding, which represents the presence of gas. B Resonance in the upper quadrants and tympany over the lower quadrants are not normal findings. There would be tympany in the lower quadrants, but also in the upper quadrants. C Dull sounds over the lower quadrants and hollow sounds over the upper quadrants are not normal findings. There would be tympany over the lower quadrants, but also in the upper quadrants. D Dull sounds over the stomach and resonant sounds over the bladder are not normal findings.

A patient is being seen in the clinic for suspected nasal obstruction from a foreign body. The nurse recognizes which finding as most consistent with this diagnosis? a. Unilateral foul-smelling drainage b. Bilateral purulent green-yellow discharge c. Bilateral bloody discharge d. Unilateral watery discharge

a. Unilateral foul-smelling drainage A This is consistent with presence of a foreign object in one side of the nose. B This is consistent with a nasal or sinus infection. C This is consistent with localized trauma, such as a nasal fracture. D This is consistent with a history of head injury and may indicate skull fracture.

A nurse had previously heard crackles over both lungs of a patient. As the patient improves, what lung sounds does the nurse expect to hear in the patient's lungs? a. Vesicular breath sounds heard in peripheral lung fields b. Bronchial breath sounds heard over the bronchi c. Bronchovesicular breath sounds heard over the apices d. Rhonchi heard over the main bronchi

a. Vesicular breath sounds heard in peripheral lung fields A Vesicular breath sounds heard in peripheral lung fields are an expected finding for healthy lungs. B Bronchial breath sounds are heard over the trachea. C Bronchovesicular breath sounds are heard anteriorly near the sternal border first and second intercostals space. D Rhonchi are adventitious sounds indicating secretions in the bronchi.

A nurse expects which finding during a cardiovascular assessment of a healthy adult? a. Visible, consistent pulsations of the jugular vein b. Pink nail beds with a 90-degree angle at the base c. Capillary refill of the toes greater than 5 seconds d. Bruits heard on auscultation of the carotid arteries

a. Visible, consistent pulsations of the jugular vein A Visible, consistent pulsations of the jugular vein is an expected finding. B Pink nail beds with a 90-degree angle at the base is not a normal finding; the angle at the base should be 160 degrees. C Capillary refill of the toes greater than 5 seconds is not a normal finding. Capillary refills should be 2 seconds or less. D Bruits heard on auscultation of the carotid arteries is not a normal finding. Bruits indicate occlusion of a blood vessel.

Which finding indicates to a nurse that a neonate has a cephalhematoma? a. Well-defined edematous area over one cranial bone b. Molding of the cranium that causes generalized cerebral edema c. Diffuse edema over two or more cranial bones d. Anterior fontanelle that is deeply depressed

a. Well-defined edematous area over one cranial bone A This is a subperiosteal hematoma under the scalp that occurs secondary to birth trauma. The area, which appears as a soft, well-defined swelling over the cranial bone, generally is reabsorbed within the first month of life. B Molding is secondary to the head passing through the birth canal and generally lasts less than a week. C Cephalhematoma occurs over one cranial bone rather than several, and is well-defined rather than diffuse. D Anterior fontanelles are soft but not depressed.

Which technique does a nurse use to assess hip location of a newborn? a. With newborn's knees flexed, the nurse adducts the legs, then abducts them, moving the knees apart and down to touch the table. b. With the newborn supine, the nurse flexes and extends the hips, and then passively moves each leg through internal and external rotation. c. The nurse holds the newborn in a vertical position with the feet flat on the table and palpates each hip for location. d. With the newborn supine, the nurse measures the length of each leg from the trochanter to the lateral malleolus (ankle).

a. With newborn's knees flexed, the nurse adducts the legs, then abducts them, moving the knees apart and down to touch the table. A This describes the Barlow-Ortolani maneuver to assess hip location and determine dislocation. B This describes an incorrect technique. C This describes an incorrect technique. D This describes an incorrect technique.

A nurse is a preceptor for a nurse who just graduated from nursing school. When caring for a patient, the new graduate nurse begins to explain to the patient the purpose of completing a physical assessment. Which statement made by the new graduate nurse requires the preceptor to intervene? a. "I will use the information from my assessment to figure out if your antihypertensive medication is working effectively." b. "Nursing assessment data are used only to provide information about the effectiveness of your medical care." c. "Nurses use data from their patient's physical assessment to determine a patient's educational needs." d. "Information gained from physical assessment helps nurses better understand their patients' emotional needs."

b. "Nursing assessment data are used only to provide information about the effectiveness of your medical care." Nursing assessment data are used to evaluate the effectiveness of all aspects of a patient's care, not just the patient's medical care. Assessment data help to evaluate the effectiveness of medications and to determine a patient's health care needs, including the need for patient education. Nurses also use assessment data to identify patients' psychosocial and cultural needs.

A nurse is assessing the ability to learn of a patient who has recently experienced a stroke. Which question/statement will best assess the patient's ability to learn? a. "What do you want to know about strokes?" b. "Please read this handout and tell me what it means." c. "Do you feel strong enough to perform the tasks I will teach you?" d. "On a scale from 1 to 10, tell me where you rank your desire to learn."

b. "Please read this handout and tell me what it means." A patient's reading level affects ability to learn. One way to assess a patient's reading level and level of understanding is to ask the patient to read instructions from an educational handout and then explain their meaning. Reading level is often difficult to assess because patients who are functionally illiterate are often able to conceal it by using excuses such as not having the time or not being able to see. Asking patients what they want to know identifies previous learning and learning needs and preferences; it does not assess ability to learn. Motivation (desire to learn) is related to readiness to learn, not ability to learn. Just asking a patient if he or she feels strong is not as effective as actually assessing the patient's strength.

A nurse is determining if teaching is effective. Which finding best indicates learning has occurred? a. A nurse presents information about diabetes. b. A patient demonstrates how to inject insulin. c. A family member listens to a lecture on diabetes. d. A primary care provider hands a diabetes pamphlet to the patient.

b. A patient demonstrates how to inject insulin. Learning is the purposeful acquisition of new knowledge, attitudes, behaviors, and skills: patient demonstrates how to inject insulin. A new mother exhibits learning when she demonstrates how to bathe her newborn. A nurse presenting information and a primary care provider handing a pamphlet to a patient are examples of teaching. A family member listening to a lecture does not indicate that learning occurred; a change in knowledge, attitudes, behaviors, and/or skills must be evident.

A nurse is teaching a patient about heart failure. Which environment will the nurse use? a. A darkened, quiet room b. A well-lit, ventilated room c. A private room at 85° F temperature d. A group room for 10 to 12 patients with heart failure

b. A well-lit, ventilated room The ideal environment for learning is a room that is well lit and has good ventilation, appropriate furniture, and a comfortable temperature. Although a quiet room is appropriate, a darkened room interferes with the patient's ability to watch your actions, especially when demonstrating a skill or using visual aids such as posters or pamphlets. A room that is cold, hot, or stuffy makes the patient too uncomfortable to focus on the information being presented. Learning in a group of six or less is more effective and avoids distracting behaviors.

A nurse is performing a mental status examination and asks an adult patient what the statement "Don't cry over spilled milk" means. Which area is the nurse assessing? a. Long-term memory b. Abstract thinking c. Recent memory d. Knowledge

b. Abstract thinking For an individual to explain common phrases such as "A stitch in time saves nine" or "Don't cry over spilled milk" requires a higher level of intellectual function or abstract thinking. Knowledge-based assessment is factual. Assess knowledge by asking how much the patient knows about the illness or the reason for seeking health care. To assess past (long-term) memory, ask the patient to recall the maiden name of the patient's mother, a birthday, or a special date in history. It is best to ask open-ended questions rather than simple yes/no questions. Patients demonstrate immediate recall (recent memory) by repeating a series of numbers in the order in which they are presented or in reverse order.

A teen female patient reports intermittent abdominal pain for 12 hours. No dysuria is present. Which action will the nurse take when performing an abdominal assessment? a. Assess the area that is most tender first. b. Ask the patient about the color of her stools. c. Recommend that the patient take more laxatives. d. Avoid sexual references such as possible pregnancy.

b. Ask the patient about the color of her stools. Abdominal pain can be related to bowels. If stools are black or tarry (melena), this may indicate gastrointestinal alteration. The nurse should caution patients about the dangers of excessive use of laxatives or enemas. There is not enough information about the abdominal pain to recommend laxatives. Determine if the patient is pregnant, and note her last menstrual period. Pregnancy causes changes in abdominal shape and contour. Assess painful areas last to minimize discomfort and anxiety.

A patient has heart failure and kidney failure. The patient needs teaching about dialysis. Which nursing action is most appropriate for assessing this patient's learning needs? a. Assess the patient's total health care needs. b. Assess the patient's health literacy. c. Assess all sources of patient data. d. Assess the goals of patient care.

b. Assess the patient's health literacy. Because health literacy influences how you deliver teaching strategies, it is critical for you to assess a patient's health literacy before providing instruction. The nursing process requires assessment of all sources of data to determine a patient's total health care needs. Evaluation of the teaching process involves determining outcomes of the teaching/learning process and the achievement of learning objectives; assessing the goals of patient care is the evaluation component of the nursing process.

A patient learns that a normal adult heartbeat is 60 to 100 beats/min after a teaching session with a nurse. In which domain did learning take place? a. Kinesthetic b. Cognitive c. Affective d. Psychomotor

b. Cognitive The patient acquired knowledge, which is cognitive. Cognitive learning includes all intellectual skills and requires thinking. In the hierarchy of cognitive behaviors, the simplest behavior is acquiring knowledge. Kinesthetic is a type of learner who learns best with a hands-on approach. Affective learning deals with expression of feelings and development of attitudes, beliefs, or values. Psychomotor learning involves acquiring skills that require integration of mental and physical activities, such as the ability to walk or use an eating utensil.

A teen patient is tearful and reports locating lumps in her breasts. Other history obtained is that she is currently menstruating. Physical examination reveals soft and movable cysts in both breasts that are painful to palpation. The nurse also notes that the patient's nipples are erect, but the areola is wrinkled. Which action will the nurse take after talking with the health care provider? a. Reassure patient that her symptoms are normal. b. Discuss the possibility of fibrocystic disease as the probable cause. c. Consult a breast surgeon because of the abnormal nipples and areola. d. Tell the patient that the symptoms may get worse when her period ends.

b. Discuss the possibility of fibrocystic disease as the probable cause. A common benign condition of the breast is benign (fibrocystic) breast disease. This patient has symptoms of fibrocystic disease, which include bilateral lumpy, painful breasts sometimes accompanied by nipple discharge. Symptoms are more apparent during the menstrual period. When palpated, the cysts (lumps) are soft, well differentiated, and movable. Deep cysts feel hard. Although a common condition, benign breast disease is not normal; therefore, the nurse does not tell the patient that this is a normal finding. During examination of the nipples and areolae, the nipple sometimes becomes erect with wrinkling of the areola. Therefore, consulting a breast surgeon to treat her nipples and areolae is not appropriate.

While assessing the skin of an 82-year-old patient, a nurse discovers nonpainful, ruby red papules on the patient's trunk. What is the nurse's next action? a. Explain that the patient has basal cell carcinoma and should watch for spread. b. Document cherry angiomas as a normal older adult skin finding. c. Tell the patient that this is a benign squamous cell carcinoma. d. Record the presence of petechiae.

b. Document cherry angiomas as a normal older adult skin finding. The skin is normally free of lesions, except for common freckles or age-related changes such as skin tags, senile keratosis (thickening of skin), cherry angiomas (ruby red papules), and atrophic warts. Basal cell carcinoma is most common in sun-exposed areas and frequently occurs in a background of sun-damaged skin; it almost never spreads to other parts of the body. Squamous cell carcinoma is more serious than basal cell and develops on the outer layers of sun-exposed skin; these cells may travel to lymph nodes and throughout the body. Report abnormal lesions to the health care provider for further examination. Petechiae are nonblanching, pinpoint-size, red or purple spots on the skin caused by small hemorrhages in the skin layers.

An older-adult patient is taking aminoglycoside for a severe infection. Which assessment is the priority? a. Eyes b. Ears c. Skin d. Reflexes

b. Ears Older adults are especially at risk for hearing loss caused by ototoxicity (injury to auditory nerve) resulting from high maintenance doses of antibiotics (e.g., aminoglycosides). While eyes and skin are important, they are not the priority. Reflexes are expected to be diminished in older adults.

An older-adult patient is being seen for chronic entropion. Which condition will the nurse assess for in this patient? a. Ptosis b. Infection c. Borborygmi d. Exophthalmos

b. Infection The diagnosis of entropion can lead to lashes of the lids irritating the conjunctiva and cornea. Irritation can lead to infection. Exophthalmos is a bulging of the eyes and usually indicates hyperthyroidism. An abnormal drooping of the lid over the pupil is called ptosis. In the older adult, ptosis results from a loss of elasticity that accompanies aging. Hyperactive sounds are loud, "growling" sounds called borborygmi, which indicate increased GI motility.

The nurse is examining a female with vaginal discharge. Which position will the nurse place the patient for proper examination? a. Sitting b. Lithotomy c. Knee-chest d. Dorsal recumbent

b. Lithotomy Lithotomy is the position for examination of female genitalia. The lithotomy position provides for the maximum exposure of genitalia and allows the insertion of a vaginal speculum. Sitting does not allow adequate access for speculum insertion and is better used to visualize upper body parts. Dorsal recumbent is used to examine the head and neck, anterior thorax and lungs, breasts, axillae, heart, and abdomen. Knee-chest provides maximal exposure of the rectal area but is embarrassing and uncomfortable.

A nurse is trying to help a patient begin to accept the chronic nature of diabetes. Which teaching technique should the nurse use to enhance learning? a. Lecture b. Role play c. Demonstration d. Question and answer sessions

b. Role play Affective learning deals with expression of feelings and acceptance of attitudes, beliefs, or values. Role play and discussion (one-on-one and group) are effective teaching methods for the affective domain. Lecture and question and answer sessions are effective teaching methods for the cognitive domain. Demonstration is an effective teaching method for the psychomotor domain.

Strategies to include in a teaching plan for an adult who has repeatedly not followed the written discharge instructions would include a. individualized handout. b. instructional videos. c. Internet resources. d. self-help books.

b. instructional videos. An instructional video would provide a visual/auditory approach for discharge instructions. Repeatedly not following written instructions is a clue that the patient may not be able to read or understand the information. While assessing the literacy level of an adult patient can be challenging, the information that they have not been able to follow previous written instructions would suggest that the nurse use an alternate strategy that does not require a high degree of literacy. An individualized handout would be written, very similar to previous instructions, and would not address a concern about literacy. Internet resources generally require an individual to be able to read, and although videos are available through the Internet, this is not the best response. Self-help books would be appropriate for an individual who reads. There is a question about whether this patient is literate, so these would not be the best choice.

What does the nurse say to obtain more data about a patient's vague statement about diet such as, "My diet's okay"? a. "Eating a variety of meats, fruits, and vegetables each day is important." b. "Give me an example of the foods you eat in a typical day." c. "Go on." d. "Does your diet meet your needs or does it need improvement?"

b. "Give me an example of the foods you eat in a typical day." A "Eating a variety of meats, fruits, and vegetables each day is important." While this statement is true, it does not obtain data about what foods the patient consumes. B "Give me an example of the foods you eat in a typical day." This statement asks the patient to clarify the vague statement, "My diet is okay." C "Go on" encourages patients to continue talking, but does not help clarify what foods are consumed. D "Does your diet meet your needs or does it need improvement?" This response does not help clarify what foods the patient eats. Also it contains two questions rather than asking one question at a time.

In assessing a patient with renal disease, the nurse palpates edema in both ankles and feet. Based on this finding, what question does the nurse ask the patient? a. "Have you had any pain in your abdomen?" b. "Have you had an unexpected weight gain?" c. "Have you noticed a change in the color of your skin?" d. "Have you had any nausea or vomiting?"

b. "Have you had an unexpected weight gain?" A "This question does not relate to renal disease. The pain experienced with renal disease is usually flank pain over the costovertebral angle. B This question relating to weight gain and edema suggests fluid retention that occurs with renal or heart disease, particularly renal failure. C This question does not relate to renal disease. It might relate to liver or gallbladder disease if the change in skin color was yellow, indicating jaundice. D This question usually relates to disorders within the gastrointestinal tract itself and not renal disease.

A patient reports having abdominal distention. The nurse notices that the patient's sclerae are yellow. What question is appropriate for the nurse to ask in response to this information? a. "Has there been a change in your usual pattern of urination?" b. "Have you had any nausea or vomiting?" c. "Has there been a change in your bowel habits?" d. "Have you had indigestion or heartburn?"

b. "Have you had any nausea or vomiting?" A "Has there been a change in your usual pattern of urination?" is not a question related to the abdominal distention and jaundice. B "Have you had any nausea or vomiting?" is an appropriate question because the nurse suspects the patient may have a liver disease based on the abdominal distention and jaundice. The nurse interprets the relationship with data gathered from the history and the observation. C "Has there been a change in your bowel habits?" is a question that may be related to the abdominal distention, but not the jaundice. D "Have you had indigestion or heartburn?" is not a question related to the abdominal distention and jaundice. It applies more to gastric disorders, such as gastroesophageal reflux disease or hiatal hernia.

During an assessment for abdominal pain, a patient reports a colicky abdominal pain and pain in the right shoulder that gets worse after eating fried foods. What question does the nurse ask to confirm the suspicion of cholelithiasis? a. "Have you noticed any swelling in your ankles or feet at the end of the day?" b. "Have you noticed a change in the color of your urine or stools?" c. "Have you vomited up any blood in the last 24 hours?" d. "Have you experienced fever, chills, or sweating?"

b. "Have you noticed a change in the color of your urine or stools?" A This question is related to fluid retention, which may be asked if the patient has renal or heart failure. B Gallstones can obstruct the flow of bile to the gastrointestinal tract making urine darker and stools lighter in color. C This question applies if the patient has peptic ulcer disease or esophageal varices. D This question applies if the patient has gastroenteritis or a urinary tract infection.

Which tool is most appropriate for testing the vision of a 5-year-old child? a. Denver II test b. Snellen E chart c. Allen picture cards d. Snellen standard chart

b. Snellen E chart A Vision can be assessed when performing developmental tests such as the Denver II, but the Snellen E provides more objective data. B Use the Snellen E chart for children 3 to 6 years of age (see Chapter 11). Have children point their fingers in the direction of the "arms" of the E. C Use the Allen picture cards to screen for visual acuity in 2.5- to 3-year-old children. D The Snellen standard chart is used for adolescents and adults; it is too difficult to use for children.

Which question will give the nurse additional information about the nature of a patient's dyspnea? a. "How often do you see the physician?" b. "How has this condition affected your day-to-day activities?" c. "Do you have a cough that occurs with the dyspnea?" d. "Does your heart rate increase when you are short of breath?"

b. "How has this condition affected your day-to-day activities?" A This question does not relate specifically to the patient's dyspnea. B This question provides data about the severity of the dyspnea and what actions the patient has taken to cope with the dyspnea on a daily basis. C This question provides data, but does not give additional facts about the patient's dyspnea. D This is a closed-ended question that does not collect additional data about this episode of dyspnea.

A nurse is getting a history from a patient who is disabled from rheumatoid arthritis. Which question will provide data about this patient's functional ability? a. "When did your arthritis symptoms begin?" b. "How has your arthritis affected your daily life?" c. "Why did you come to the clinic today?" d. "How do you feel about your diagnosis of rheumatoid arthritis?"

b. "How has your arthritis affected your daily life?" A "When did your arthritis symptoms begin?" is a question asked as part of the history, but does not collect data about functional ability. B "How has your arthritis affected your daily life?" is a question that leads to data about the patient's ability to perform self-care activities or functional abilities. C "Why did you come to the clinic today?" is a question asked to obtain the chief complaint about a current problem, but does not focus directly on the functional assessment. D "How do you feel about your diagnosis of rheumatoid arthritis?" is a question to ask in the psychosocial history, but does not focus directly on the functional assessment.

An example of a health promotion question included in the health history is: a. "Do you have any allergies?" b. "How often are you exercising?" c. "What are you doing to relieve your leg pain?" d. "What kind of herbs are you using?"

b. "How often are you exercising?" A "Do you have any allergies?" is a question for the present health status rather than health promotion. B "How often are you exercising?" is a question about activities patients regularly perform to maintain health. C "What are you doing to relieve your leg pain?" is a question that is part of the symptom analysis. D "What kind of herbs are you using?" is a question for the present health status rather than health promotion.

Which patient's statement helps a nurse distinguish between chest pain originating from pericarditis rather than from angina? a. "No, I have not done anything to strain chest muscles." b. "If I take a deep breath, the pain gets much worse." c. "This pain feels like there's an elephant sitting on my chest." d. "Whenever this pain happens, it goes right away if I lie down."

b. "If I take a deep breath, the pain gets much worse." A Chest pain from muscle strain may be aggravated by movement of arms. B The chest pain from pericarditis is aggravated by deep breathing, coughing, or lying supine. C "This pain feels like there's an elephant sitting on my chest" is associated with a myocardial infarction. D Chest pain relieved by rest occurs with angina.

. To document the palpation of a pulse, the nurse is correct in making which notation about the rhythm? a. "Rhythm 100 beats/min" b. "Irregular rhythm" c. "Rhythm noted at +2" d. "Bounding rhythm"

b. "Irregular rhythm" A This notation refers to the rate rather than the rhythm. B The rhythm should be an equal pattern or spacing between beats. Irregular rhythms without any pattern should be noted. C This notation refers to the amplitude rather than the rhythm. D This notation refers to the contour rather than the rhythm.

A nurse informs a patient that her blood pressure is 128/78. The patient asks what the number 128 means. What is the nurse's appropriate response? The 128 represents the pressure in your blood vessels when: a. "The ventricles relax and the aortic and pulmonic valves open." b. "The ventricles contract and the mitral and tricuspid valves close." c. "The ventricles contract and the mitral and tricuspid valves open." d. "The ventricles relax and the aortic and pulmonic valves close."

b. "The ventricles contract and the mitral and tricuspid valves close." A The aortic and pulmonic valves open during systole, but ventricles fill during diastole. B During systole the ventricles contract, creating a pressure that closes the atrioventricular (AV) valves (mitral and tricuspid). C During systole the ventricles contract, creating a pressure that closes the AV valves (mitral and tricuspid). D The ventricles are relaxed and the aortic and pulmonic valves close during diastole, rather than systole.

Which statement is appropriate to use when beginning an interview with a new patient? a. "Have you ever been a patient in this clinic before?" b. "What is your purpose for coming to the clinic today?" c. "Tell me a little about yourself and your family." d. "Did you have any difficulty finding the clinic?"

b. "What is your purpose for coming to the clinic today?" A "Have you ever been a patient in this clinic before?" is a close-ended question that yields a "yes" or "no" response. This question may be asked on the first visit, but not as an opening question for a health interview. B "What is your purpose for coming to the clinic today?" is an open-ended question that focuses on the patient's reason for seeking care. C "Tell me a little about yourself and your family" is an open-ended question, but it is too general, and it is at least two questions: one about the patient and another about the family. D "Did you have any difficulty finding the clinic?" is a social question and does not focus on the patient's purpose for the visit.

While assessing edema on a male patient's lower leg, the nurse notices that there is a slight imprint of his fingers where he palpated the patient's leg. How does the nurse document this finding? a. No edema b. 1+ edema c. 2+ edema d. 3+ edema

b. 1+ edema A No pit left after palpation indicates no edema. B A barely perceptible pit is detected after palpation. C A deeper pit that rebounds in a few seconds after palpation is 2+ edema. D A deep pit that rebounds in 10 to 20 seconds after palpation is 3+ edema.

The nurse is assessing an adult patient's patellar reflex. Which finding will the nurse record as normal? a. 1+ b. 2+ c. 3+ d. 4+

b. 2+ Grade reflexes as follows: 0: No response; 1+: Sluggish or diminished; 2+: Active or expected response; 3+: More brisk than expected, slightly hyperactive; and 4+: Brisk and hyperactive with intermittent or transient clonus.

Which finding on assessment of a patient's eyes should the nurse document as abnormal? a. An Asian American patient with an upward slant to the palpebral fissure b. A Caucasian American patient whose sclerae are visible between the upper and lower lids and the iris c. An African American patient who has off-white sclerae with tiny black dots of pigmentation near the limbus d. An American Indian patient whose pupillary diameters are 5 mm bilaterally

b. A Caucasian American patient whose sclerae are visible between the upper and lower lids and the iris A An Asian American patient with an upward slant to the palpebral fissure has an expected racial variation. B A Caucasian American patient whose sclerae are visible between the upper and lower lids and the iris has eyeball protrusion beyond the supraorbital ridge, which indicates exophthalmos caused by hyperthyroidism. C An African American patient who has off-white sclerae with tiny black dots of pigmentation near the limbus has an expected racial variation. D An American Indian whose pupils are 5 mm bilaterally is an expected finding.

Which situation illustrates a screening assessment? a. A patient visits an obstetric clinic for the first time and the nurse conducts a detailed history and physical examination. b. A hospital sponsors a health fair at a local mall and provides cholesterol and blood pressure checks to mall patrons. c. The nurse in an urgent care center checks the vital signs of a patient who is complaining of leg pain. d. A patient newly diagnosed with diabetes mellitus comes to test his fasting blood glucose level.

b. A hospital sponsors a health fair at a local mall and provides cholesterol and blood pressure checks to mall patrons. A A detailed history and physical examination conducted during a first-time visit to an obstetric clinic is an example of a comprehensive assessment. B A health fair at a local mall that provides cholesterol and blood pressure checks is an example of a screening assessment focused on disease detection. C Assessing a patient complaining of leg pain in the triage area of an urgent care center is an example of a problem-based/focused assessment. D A patient's return appointment 1 month after today's office visit to report fasting blood glucose levels is an example of an episodic or follow-up assessment.

For which patient is a focused health history most appropriate? a. A new patient at the health clinic for an annual examination b. A patient admitted to the hospital with vomiting and abdominal pain c. A patient at the health care provider's office for a sport physical d. A patient discharged 11 months ago who is being readmitted today

b. A patient admitted to the hospital with vomiting and abdominal pain A A new patient at the health clinic for an annual examination needs a comprehensive history that includes biographic data, reason for seeking care, present health status, past medical history, family history, personal and psychosocial history, and a review of all body. B A patient admitted to the hospital with vomiting and abdominal pain benefits from a focused health history that limits data to the immediate problem. C A patient with a specific need, such as a sport physical, needs a history for an episodic assessment. D A patient discharged months ago who is being readmitted needs a history for a follow-up assessment that generally focuses on the specific problem or problems that caused the patient to be readmitted.

While taking a history, a nurse learns that a patient had rheumatic heart disease as a child. Based on this information, what abnormal data might this nurse expect to find during an examination? a. An extra beat just before the S2 heart sound heard during auscultation b. A raspy machine-like or blowing sound heard during auscultation c. A prominent thrust of the heart against the chest wall felt on palpation d. A visible indentation of pericardial tissue noted during inspection

b. A raspy machine-like or blowing sound heard during auscultation A An extra beat just before the S1 heart sound heard during auscultation is a description of the S4 heart sound that occurs when there is hypertrophy of the ventricle. B A raspy machine-like or blowing sound heard during auscultation is a description of a murmur that can develop after rheumatic heart disease. C A prominent thrust of the heart against the chest wall felt on palpation is a description of a heave, which may occur from left ventricular hypertrophy due to increased workload. D A visible indentation of pericardial tissue noted during inspection is a description of a retraction that begins in the intercostal spaces and occurs with increased respiratory effort.

A nurse assessing the hearing of a patient with presbycusis expects which finding on a test for hearing? a. Bone conduction will be longer than air conduction on the Rinne test (BC > AC). b. Air conduction will be longer than bone conduction on the Rinne test (AC > BC). c. Sound lateralizes to the affected ear on the Weber test. d. Sound lateralizes to both ears equally on the Weber test.

b. Air conduction will be longer than bone conduction on the Rinne test (AC > BC). A This finding indicates a conduction hearing loss, rather than a sensorineural hearing loss. B This finding indicates a sensorineural health loss, the most common cause of presbycusis. C This finding indicates a conduction hearing loss, rather than a sensorineural hearing loss. D This is a normal finding on this test.

During the assessment of a newborn within hours after birth, a nurse determines which finding as abnormal? a. Capillary refill time of less than 1 second b. Apical pulse felt at the second intercostal space c. Splitting of heart sounds d. Cyanosis of the hands and feet

b. Apical pulse felt at the second intercostal space A This is a normal finding; capillary refill in infants is rapid—less than 1 second. B This finding is abnormal because the apical pulse of the newborn normally is felt in the fourth or fifth intercostal space. C Splitting of heart sounds is common in infants until about 48 hours after birth because of the transition from fetal circulation to systemic and pulmonic circulation. D Acrocyanosis (cyanosis of hands and feet) without central cyanosis is of little concern and usually disappears within hours to days after birth.

The nurse is interviewing a woman with her husband present. The husband answers the questions for the wife most of the time. What is the most appropriate therapeutic nursing action to hear the patient's viewpoint? a. Continue the interview. b. Ask the husband to step out of the room. c. Ask another nurse to complete the interview. d. Tell the woman to speak up for herself.

b. Ask the husband to step out of the room. A Continuing the interview is not a therapeutic action because the nurse needs to obtain the patient's answers to the questions. B Asking the husband to step out of the room will allow the patient to answer questions in her own way. C Asking another nurse to complete the interview does not solve the problem that the husband is answering questions for his wife. D Telling the woman to speak up for herself does not solve the problem and may interfere with the therapeutic relationship between the patient and the nurse.

A nurse auscultates low-pitched, coarse snoring sounds in a patient's lungs during inhalation. What is the most appropriate action for the nurse to take at this time? a. Palpate the posterior thorax for vocal fremitus. b. Ask the patient to cough and repeat auscultation. c. Auscultate the posterior thorax for vocal sounds. d. Percuss the posterior thorax for tone.

b. Ask the patient to cough and repeat auscultation. A An abnormal vocal fremitus (decreased or increased vibrations) is not expected for this patient. B The sounds indicate rhonchi, or secretions in the bronchi. The first action to take is to determine if the rhonchi clear with coughing. If the rhonchi clear, there is no need to further investigate this finding. C Abnormal vocal sounds (clear and loud sounds) are not expected for this patient. D An abnormal percussion tone (hyperresonance or dull) is not expected for this patient.

How does the nurse test the function of the patient's spinal accessory nerve (CN XI)? a. Ask the patient to stick out the tongue and move it side to side. b. Ask the patient to shrug the shoulders against the resistance of the nurse's hands. c. Ask the patient to open the mouth and observe the uvula rise when he says "ah." d. Ask the patient to move the chin to the chest and then up toward the ceiling.

b. Ask the patient to shrug the shoulders against the resistance of the nurse's hands. A This is a test of the hypoglossal cranial nerve (XII). B This is the correct technique for assessing the spinal accessory cranial nerve (XI). C This is a test for cranial nerves IX (glossopharyngeal) and X (vagus). D This technique assesses the range motion of the neck.

What technique does a nurse use when performing deep palpation of a patient's abdomen? a. Places the left hand under the ribs to lift them up b. Asks the patient to breathe slowly through the mouth c. Positions the patient on the right side with knees flexed d. Uses the heel of the hand to depress the abdomen

b. Asks the patient to breathe slowly through the mouth A Placing the left hand under the ribs to lift them up is the technique for palpating the liver. B Asking the patient to breathe slowly through the mouth while the nurse uses the pads of the fingers to depress the abdomen is the correct procedure. C Positioning the patient on the right side with knees flexed is an alternate strategy for palpating the spleen. D Using the heel of the hand to depress the abdomen is not a correct technique; the pads of the fingers are used.

How does a nurse test the brachioradial deep tendon reflex? a. Uses the end of the handle on the reflex hammer to stroke the lateral aspect of the sole of the patient's foot from heel to ball b. Asks the patient to slightly pronate the relaxed forearm into the nurse's hand and strikes the appropriate tendon with the reflex hammer c. Holds the patient's relaxed arm with the elbow flexed at a 90-degree angle in one hand, and palpates and strikes the appropriate tendon just above the elbow with the flat end of the reflex hammer d. Holds the patient's relaxed arm with the elbow flexed at a 90-degree angle, places a thumb over the appropriate tendon in the antecubital fossa, and strikes the thumb with the pointed end of the reflex hammer

b. Asks the patient to slightly pronate the relaxed forearm into the nurse's hand and strikes the appropriate tendon with the reflex hammer A This is the technique to test plantar flexion, the Babinski reflex. B This is the technique to assess the brachioradial deep tendon reflex. C This is the technique to test the triceps deep tendon reflex. D This is the technique to test the biceps deep tendon reflex.

During assessment of an infant, the nurse notes that when the infant cries, the fontanelles bulge slightly. What is the most appropriate action for the nurse at this time? a. Note in the record that the child is microcephalic. b. Assess the fontanelles again when the child is not crying. c. Check the child for signs of malnutrition and dehydration. d. Use transillumination for further assessment of the skull.

b. Assess the fontanelles again when the child is not crying. A Microcephaly occurs when the head circumference is less than expected, which is not the finding here. B The fontanelles frequently bulge when an infant is crying. The nurse will palpate the anterior and posterior fontanelles for fullness while the infant is in an upright position and calm. C An elevated fontanelle when the infant is calm may indicate overhydration or fluid volume excess. D Transillumination is not needed in this case.

Which location does a nurse select when palpating a patient's liver? a. A (right lower quadrant) b. B (right upper quadrant) c. C (left upper quadrant) d. D (left lower quadrant)

b. B (right upper quadrant) A The majority of the liver is located in the right upper quadrant of the abdomen. B The majority of the liver is located in the right upper quadrant of the abdomen. C C is the left upper quadrant. D D is the left lower quadrant.

The nurse holds the patient's relaxed arm with elbow flexed at a 90-degree angle, places a thumb over a tendon in the antecubital fossa, and strikes the thumb with the pointed end of the reflex hammer. Which deep tendon reflex is the nurse assessing? a. Brachioradialis b. Biceps c. Triceps d. Deltoid

b. Biceps A The technique described is not the correct one for assessing the brachioradial deep tendon reflex. B This is the correct technique for assessing the biceps deep tendon reflex. C The technique described is not the correct one for assessing the triceps deep tendon reflex. D There is no reflex to test the deltoid muscle.

The nurse is comparing pitch and duration of the various types of a patient's breath sounds and recognizes which one of these as an expected finding? a. Bronchial sounds are low-pitched and have a 2:1 inspiratory-versus-expiratory ratio. b. Bronchovesicular sounds have a moderate pitch and 1:1 expiratory-versus-inspiratory ratio. c. Vesicular breath sounds are high-pitched and have a 1:2 inspiratory-versus-expiratory ratio. d. Wheezes are low-pitched and have a 2.5:1 inspiratory-versus-expiratory ratio.

b. Bronchovesicular sounds have a moderate pitch and 1:1 expiratory-versus-inspiratory ratio. A Bronchial sounds are high pitched with a duration of 1:2 inspiration-to-expiration is the correct statement. B Bronchovesicular sounds having a moderate pitch and 1:1 expiratory-versus-inspiratory ratio is a normal finding. C Vesicular sounds are low pitched with a duration of 2.5:1 inspiration-to-expiration is the correct statement. D Wheezes are high-pitched and have no specific duration because they are adventitious sounds.

A nurse is assessing a child who is able to dress herself, jump rope, identify colors, and follow rules when playing games. These are expected developmental achievements of a child of what age? a. 3 years old b. 4 years old c. 5 years old d. 6 years old

c. 5 years old A These are developmental behaviors too advanced for a 3-year-old child. B These are developmental behaviors too advanced for a 4-year-old child. C These are developmental behaviors consistent with a 5-year-old child. D These developmental behaviors are typically achieved and surpassed by a 6-year-old child.

After seeing the red reflex and retinal vessels through the ophthalmoscope, how does the nurse locate the optic disc? a. By rotating the diopter to the block (positive) numbers until the optic disc comes into focus b. By following the retinal vessels inward toward the nose until optic disc is seen c. By using the green beam light while looking outward toward the ear until the disc is seen d. By locating the macula and then looking temporally (toward the ear) until the disc is seen

b. By following the retinal vessels inward toward the nose until optic disc is seen A This procedure is used for patients who are myopic. B This procedure locates the optic disc. C The green beam is used to identify retinal hemorrhages. D The macula lies temporal to the optic disc; thus the optic disc is in the opposite direction.

The nurse moves a wisp of cotton lightly across the anterior scalp, paranasal sinuses, and lower jaw to test the function of which cranial nerve? a. CN IV (trochlear nerve) b. CN V (trigeminal nerve) c. CN VI (abducens nerve) d. CN VII (facial nerve)

b. CN V (trigeminal nerve) A The CN IV (trochlear nerve) supplies downward and inward eye movement. B The CN V (trigeminal cranial nerve) supplies sensation to the cornea, iris, lacrimal glands, conjunctiva, eyelids, forehead, nose, nasal and mouth mucosa, teeth, tongue, ear, and facial skin. C The CN VI (abducens nerve) supplies lateral eye movement. D The CN VII (facial nerve) supplies movement of facial expression muscles except the jaw, closes the eyes, and allows labial speech sounds (b, m, w, and rounded vowels).

A nurse is assessing a patient who was hit at the base of the skull with a blunt instrument causing a skull fracture. What assessment finding does this nurse anticipate during the inspection? a. Tinnitus, vertigo, and dizziness b. Clear drainage from the ear and nose c. Loss of hearing and smell d. Purulent drainage from the ear and bloody drainage from the nose

b. Clear drainage from the ear and nose A These are subjective and gathered during the history rather than inspection. Although the patient may report having dizziness or vertigo, the finding of tinnitus is inconsistent with a basilar skull fracture. B This may occur after a basilar skull fracture. The clear drainage may be cerebrospinal fluid. C This is inconsistent with a basilar skull fracture. D Purulent drainage is inconsistent with a basilar skull fracture, and bloody drainage usually does not come from the nose, but may be seen from the ear.

A nurse is assessing a patient who was diagnosed with emphysema and chronic bronchitis 5 years ago. During the assessment of this patient's integumentary system, what finding should the nurse correlate to this respiratory disease? a. Dry, flaky skin b. Clubbing of the fingers c. Hypertrophy of the nails d. Hair loss from the scalp

b. Clubbing of the fingers A Dry, flaky skin occurs with dehydration. B Clubbing of the fingers develops due to chronic hypoxemia, which occurs in chronic obstructive pulmonary disease. C Hypertrophy of the nails occurs with repeated trauma. D Hair loss from the scalp is alopecia, which occurs with many systemic diseases, but not chronic pulmonary disease.

In performing a respiratory assessment of a 1-month-old infant, the nurse recognizes which finding as abnormal? a. Sneezing b. Coughing c. Abdominal breathing d. Predominantly nose breathing

b. Coughing A Sneezing is a common finding for an infant and is therapeutic because it helps to clear the nose. B Coughing at this age is considered abnormal and indicates a problem. C Infants use abdominal breathing rather than diaphragmatic breathing. D Infants are obligate nose breathers until about 3 months old.

A patient has a herniated disk compressing the lumbar spine at L2, L3, and L4 that is impairing deep tendon reflexes. Which response does a nurse expect from this patient? a. Diminished contraction of the gastrocnemius muscle with plantar flexion of the foot b. Diminished contraction of the quadriceps muscle with extension of the lower leg c. Diminished plantar flexion of the toes d. Diminished dorsiflexion of the foot and flexion of the toes

b. Diminished contraction of the quadriceps muscle with extension of the lower leg A This is an abnormal response from the Achilles tendon reflex that is innervated from S1 and S2. B This is an abnormal response from the patellar deep tendon reflex that is innervated from L2, L3, and L4. C This is an abnormal response from the plantar reflex or a positive Babinski sign. D This is an abnormal response of ankle clonus.

What technique does a nurse use when palpating the right lobe of a patient's thyroid gland using the anterior approach? a. Pushes the cricoid process to the left with the right thumb b. Displaces the trachea to the right with the left thumb c. Manipulates the thyroid between the thumb and index finger d. Moves the sternocleidomastoid muscle to the right with the left thumb

b. Displaces the trachea to the right with the left thumb A This is not a correct technique. B This is the correct technique for palpating the thyroid gland using the anterior approach. C This is not a correct technique. D This is not a correct technique.

In assessing the eyes of a 4-month-old infant, a nurse shines a penlight in the infant's eyes and notices that the light reflection is not in the same location in each eye. What is the nurse's most appropriate response to this finding? a. Perform the cover-uncover test. b. Document it as an expected finding at this age. c. Document abnormal function of cranial nerves IV (trochlear) and VI (abducens). d. Refer the infant to an ophthalmologist.

b. Document it as an expected finding at this age. A Performing the cover-uncover test is not indicated unless there is asymmetric light reflex after 6 months of age. B Transient strabismus is common during the first few months of life due to a lack of binocular vision. If it continues after 6 months of age; however, a referral to an ophthalmologist is needed. C The data do not support this interpretation. D Referral is not needed unless the finding is present after the infant is 6 months old.

The patient describes her chest pain as "squeezing, crushing, and 12 on a scale of 10." This pain started more than an hour ago while she was resting, and she also feels nauseous. Based on these findings, the nurse should assess for which associated symptoms? a. Tachycardia, tachypnea, and hypertension b. Dyspnea, diaphoresis, and palpitations c. Hyperventilation, fatigue, anorexia, and emotional strain d. Fever, dyspnea, orthopnea, and friction rub

b. Dyspnea, diaphoresis, and palpitations A Tachycardia, tachypnea, and hypertension are symptoms associated with cocaine-induced chest pain. B Dyspnea, diaphoresis, and palpitations are symptoms associated with unstable angina. C Hyperventilation, fatigue, anorexia, and emotional strain are symptoms associated with panic disorder. D Fever, dyspnea, orthopnea, and friction rub are symptoms associated with pericarditis.

In assessing a patient with a tumor in the pons, the nurse expects to find which abnormalities due to pressure on cranial nerves? a. Dilated pupils and ptosis b. Facial asymmetry and impaired hearing c. Difficulty swallowing d. Impaired gag reflex

b. Facial asymmetry and impaired hearing A These abnormalities represent pressure on the oculomotor (CN III) that exits from the midbrain. B These abnormalities represent pressure on the facial and acoustic cranial nerves. The nurse correlates the cranial nerves that exit from the pons which are trigeminal (CN V), abducens (CN VI), facial (CN VII), and acoustic (CN VIII). C This abnormality represents pressure on the three cranial nerves that affect swallowing: glossopharyngeal (CN IX), vague (CN X), and hypoglossal (CN XII). These cranial nerves exit the brainstem in the medulla oblongata. D This reflex is controlled by the vagus cranial nerve (CN IX), which exits the brainstem in the medulla oblongata.

The nurse assesses the glossopharyngeal nerve (CN IX) by testing which reflex? a. Corneal reflex b. Gag reflex c. Blink reflex d. Cough reflex

b. Gag reflex A The corneal reflex is controlled by the trigeminal cranial nerve (CN V). B Movement of the posterior pharynx and gag reflex test is controlled by the glossopharyngeal cranial nerve (CN IX). C The blink reflex is another name for the corneal reflex. D The cough reflex is controlled from the medulla oblongata.

A patient reports having abdominal distention. The nurse observes that the patient's sclerae are yellow. Which abnormal finding does the nurse anticipate on examination of this patient's abdomen? a. Decreased bowel sounds in all quadrants b. Glistening or taut skin of the abdomen c. Bulge in the abdomen when coughing d. Bruit around the umbilicus

b. Glistening or taut skin of the abdomen A Decreased bowel sounds in all quadrants may be present if the abdominal distention was from an intestinal obstruction, but the observation of jaundice suggests liver disease, which does not decrease bowel sounds. B Glistening or taut skin of the abdomen is consistent with ascites that appear as abdominal distention. Jaundice and ascites suggest liver disease. There would also be an increase in abdominal girth. C A bulge in the abdomen when coughing is a finding associated with abdominal or incisional hernias. D Bruit around the umbilicus is a finding associated with an abdominal aortic aneurysm.

A nurse is auscultating the lungs of a healthy male patient and hears crackles on inspiration. What action can the nurse take to ensure this is an accurate finding? a. Make sure the bell of the stethoscope is used, rather than the diaphragm. b. Hold stethoscope firmly to prevent movement when placed over chest hair. c. Ask the patient not to talk while the nurse is listening to the lungs. d. Change the patient's position to ensure accurate sounds.

b. Hold stethoscope firmly to prevent movement when placed over chest hair. A Using the bell will provide inaccurate sounds, but not mimic crackles. B The stethoscope moving even slightly on chest hair can mimic the sound of crackles. C When the patient talks during auscultation, it does interfere with data collection, but the sound is a muffled voice. D Changing the position will not affect the outcome of the assessment if the initial problem remains.

How does the nurse accurately assess bowel sounds? a. Press the diaphragm of the stethoscope firmly against the abdomen in each quadrant. b. Hold the diaphragm of the stethoscope lightly against the abdomen in each quadrant. c. Press the bell of the stethoscope firmly against the abdomen in each quadrant. d. Hold the bell of the stethoscope lightly against the abdomen in each quadrant.

b. Hold the diaphragm of the stethoscope lightly against the abdomen in each quadrant. A Pressing the diaphragm of the stethoscope firmly against the abdomen in each quadrant is not the correct technique for listening to bowel sounds. B Holding the diaphragm lightly against the abdomen in each quadrant is the correct technique for listening to bowel sounds. C The bell is used to listen to vascular sounds of the abdomen, which are normally not heard. D The bell is used to listen to vascular sounds of the abdomen, which are normally not heard.

A patient is admitted with edema of the occipital lobe following a head injury. The nurse correlates which finding with damage to this area? a. Ipsilateral ptosis b. Impaired vision c. Pupillary constriction d. Increased intraocular pressure

b. Impaired vision A Ipsilateral ptosis (drooping of the eye lid) is controlled by the oculomotor cranial nerve (CN III) that is located in the midbrain. The nurse must correlate anatomy with function and assessment. B The occipital lobe contains the visual context. C Pupillary constriction is controlled by the oculomotor cranial nerve (CN III) that is located in the midbrain. D This abnormality is associated with glaucoma rather than injury to the occipital lobe.

A patient reports a productive cough with yellow sputum, fever, and a sharp pain when taking a deep breath to cough. Based on these data, what abnormal finding will the nurse anticipate on examination? a. Decreased breath sounds on auscultation b. Increased tactile fremitus and dull percussion tones c. Inspiratory wheezing found on auscultation d. Muffled sounds heard when the patient says "e-e-e"

b. Increased tactile fremitus and dull percussion tones A Decreased breath sounds on auscultation is consistent with emphysema or atelectasis when alveoli are narrowed or destroyed. B The data describe purulent sputum and inflammation of the pleura that may occur in pneumonia. Additional findings include increased tactile fremitus and dull percussion tones, indicating congested or consolidated lung tissues. C Inspiratory wheezing found on auscultation is consistent with narrowing of bronchi that may occur in asthma. D Muffled sounds heard when the patient says "e-e-e" is a normal finding on vocal resonance (bronchophony or egophony).

Which assessment technique is the nurse performing in the figure below? a. Direct percussion b. Indirect percussion c. Light palpation d. Deep palpation

b. Indirect percussion A Direct percussion is performed with one hand. B Indirect percussion is the technique shown. C Light palpation is performed using the pads of the fingers depressing the tissue 1 to 2 cm, usually on the abdomen. D Deep palpation is performed using the pads of the fingers depressing the tissue 4 to 6 cm, usually on the abdomen.

A nurse observes a student using the whisper test to screen a patient with hearing loss. Which behavior by the student requires a corrective comment from the nurse? a. Instructing the patient to cover the ear not being tested b. Standing beside the patient on the side of the ear being tested c. Shielding the mouth to prevent the patient from reading lips d. Whispering one or two syllable words and ask the patient to repeat what is heard

b. Standing beside the patient on the side of the ear being tested A This is the correct technique. B The student nurse should stand 1 to 2 feet in front or to the side of the patient. C This is the correct technique. D This is the correct technique.

A patient complains of a lesion in his nose. Which technique does a nurse use to inspect the nasal mucosa? a. Inserts a nasal speculum horizontally into the patient's affected nares b. Inserts a nasal speculum obliquely into the patient's affected nares c. Uses a light source from the ophthalmoscope d. Inserts a nasal speculum vertically into the patient's affected nares

b. Inserts a nasal speculum obliquely into the patient's affected nares A Horizontal insertion puts pressure on the nasal septum, which is painful. B This is the appropriate technique for inspecting the nares. C The alternate light source is from an otoscope, rather than an ophthalmoscope. The otoscope has an ear speculum that can be used when a nasal speculum is unavailable. D Vertical insertion obstructs the nurse's view of the internal nares.

When inspecting a patient's posterior wall of the pharynx and tonsils, a nurse documents which finding as abnormal? a. Both tonsils have a smooth surface. b. Left and right tonsils meet at the midline. c. Left and right tonsils extend beyond the posterior pillars. d. Both tonsils have a glistening appearance.

b. Left and right tonsils meet at the midline. A A smooth surface is expected for the tonsils. B This indicates an enlargement documented as 4+. C This is an expected finding for the tonsils. D This is an expected finding for the tonsils.

To assess the reflexes of a 5-month-old infant lying supine, the nurse turns the infant's head to the left side so that the chin is over the shoulder. What is the expected response for this reflex? a. Left arm and leg abduct and the right arm and leg adduct. b. Left arm and leg extend and the right arm and leg flex. c. Infant turns the chin from the left to the right side. d. Infant begins a sucking motion with the lips and tongue.

b. Left arm and leg extend and the right arm and leg flex. A The arms and legs flex and extend, rather than abduct and adduct. B This is the expected response for the tonic neck reflex that appears from birth to 6 weeks old and disappears at 4 to 6 months old. C This is not an expected finding for any infant reflexes. D This is a description of the sucking reflex.

When inspecting a patient's abdomen, the nurse notes which finding as abnormal? a. Protruding abdomen with skin that is lighter in color than the arms and legs b. Marked rhythmic pulsation to the left of the midline c. Faint, fine vascular network d. Small shadows created by changes in contour

b. Marked rhythmic pulsation to the left of the midline A Obesity may cause a protruding abdomen and although obesity is not an indicator of health, it does not necessarily indicate a disease is present. B Marked rhythmic pulsation to the left of the midline is an abnormal finding that may indicate an abdominal aortic aneurysm. C A faint, fine vascular network is a normal finding. If the vessels were engorged, it would be an abnormal finding. D Small shadows created by changes in contour are a normal finding and they are seen by using a light source to inspect the contour.

The nurse palpates the patient's jaw movement, placing two fingers in front of each ear and asking the patient to slowly open and close the mouth. What additional request does the nurse ask the patient to do to assess the jaw? a. Clinch the jaws together as tightly as possible. b. Move the lower jaw from side to side. c. Open the mouth as wide as possible, like a yawn. d. Move the lower jaw forward and backward several times.

b. Move the lower jaw from side to side. A This is not an assessment technique for the jaw. B This is the technique to complete assessment of the motion of the jaw. C This was completed when the nurse asked the patient to open and close the mouth. D This is not an assessment technique for the jaw.

When assessing the abdomen of a patient who has fluid in the peritoneal cavity, the nurse expects what change to occur when the patient turns from supine to the left side? a. Movement of the tympanic tones from lateral in the supine position to closer to midline when lying on the left side b. Movement of the dull tones from lateral in the supine position to closer to midline when lying on the left side c. Change in bowel sounds from hypoactive in the supine position to hyperactive when lying on the left side d. Change in bowel sounds from hyperactive in the supine position to hypoactive when lying on the left side

b. Movement of the dull tones from lateral in the supine position to closer to midline when lying on the left side A Movement of the tympanic tones from lateral in the supine position to closer to midline when lying on the left side is incorrect because the tone will be dull, rather than tympanic, due to the fluid. B Movement of the dull tones from lateral in the supine position to closer to midline when lying on the left side is the expected change when assessing for shifting dullness. C A change in bowel sounds from hypoactive in the supine position to hyperactive when lying on the left side is incorrect because bowel sounds would not be affected by the fluid. D A change in bowel sounds from hyperactive in the supine position to hypoactive when lying on the left side is incorrect because bowel sounds would not be affected by the fluid.

During an eye examination of an Asian patient, a nurse notices an involuntary rhythmical, horizontal movement of the patient's eyes. How does a nurse document this finding? a. An expected racial variation b. Nystagmus c. Exophthalmus d. Myopia

b. Nystagmus A This is not a racial variation. B An involuntary rhythmical, horizontal movement of the patient's eyes is a description of nystagmus. C Exophthalmus is the bulging of the eyeball forward, seen in patients with hyperthyroidism. D Myopia is an elongated eyeball found in patients who are nearsighted.

Which assessment technique is appropriate to measure the 8-month-old's vital signs during a well-baby check? a. Assess temperature using a rectal thermometer. b. Observe the infant's abdomen when counting respirations. c. Take the infant from the parent's arms to assess pulse. d. Measure blood pressure in the leg.

b. Observe the infant's abdomen when counting respirations. A Rectal temperatures should be taken as a last resort because children tend to fear intrusive procedures and because of the risk for rectal perforation. The recommended sites for temperature measurement in newborns, infants, and children to age 5 are the axillary or tympanic sites. B Infants usually breathe diaphragmatically, which requires observation of abdominal movement. C For the older infant ( 6 months) and toddler, the nurse may find that having the caregiver hold the baby or toddler decreases fear and distress, thus making it easier for the nurse to conduct the examination. D This infant is too young for blood pressure measurement. The National High Blood Pressure Education Program recommends that blood pressure be measured in children from age 3 through adolescence as part of routine health care visits.

A patient reports having difficulty swallowing. Based on this information, how does the nurse assess the cranial nerve related to swallowing? a. Ask the patient about feeling the blunt end of a paper clip along the jaw line. b. Observe the rising of the soft palate when the patient says "Ahh." c. Observe the symmetry of the face when the patient talks. d. Assess taste on the anterior part of the tongue.

b. Observe the rising of the soft palate when the patient says "Ahh." A This tests the sensory function of the trigeminal cranial nerve (CN V). B This tests the glossopharyngeal cranial nerve (CN IX), which is involved in swallowing. The nurse must correlate difficulty swallowing with the cranial nerves involved with that function and how to test it. The cranial nerves involved are IX, X, and XII. C This tests the motor function of the facial cranial nerve (CN VII). D This tests the sensory portion of the facial cranial nerve (CN VII).

A nurse who is assessing a patient's eyes finds that the pupils are equal, round, and react to light and accommodation (PERRLA). These findings verify the expected functioning of which cranial nerve? a. Optic cranial nerve (CN II) b. Oculomotor cranial nerve (CN III) c. Trochlear cranial nerve (CN IV) d. Abducens cranial nerve (CN VI)

b. Oculomotor cranial nerve (CN III) A The optic cranial nerve (CN II) provides vision. B The oculomotor cranial nerve (CN III) provides these eye functions. C The trochlear cranial nerve (CN IV) provides eye movement downward and inward. D The abducens cranial nerve (CN VI) provides lateral eye movement.

A patient is in a sitting position as the nurse palpates the temporal arteries and feels smooth, bilateral pulsations. What is the appropriate action for the nurse at this time? a. Auscultate the temporal arteries for bruits. b. Palpate the arteries with the patient in supine position. c. Document this as an expected finding. d. Measure the patient's blood pressure.

b. Palpate the arteries with the patient in supine position. c. Document this as an expected finding. A This is not necessary for this patient at this time. B This is not necessary for this patient at this time. C These are consistent with expected assessment. D This will be done as a part of the assessment, but does not relate to the expected palpation of this patient's temporal arteries.

Which part of the nervous system is a nurse assessing when he places a vibrating tuning fork on a patient's wrist or ankle? a. Frontal lobe and motor tracts b. Parietal lobe and sensory tracts c. Hypothalamus and sensory tracts d. Cerebellum and motor tracts

b. Parietal lobe and sensory tracts A The frontal lobe sends impulses governing movement through motor (efferent) tracts. B The parietal lobe receives sensory input, such as vibratory sense and pain, through sensory (afferent) tracts. C The hypothalamus functions include regulation of body temperature, hunger, and thirst and formation of autonomic nervous system responses. D The cerebellum provides equilibrium and coordination of movement.

A nurse shines a light toward the bridge of the patient's nose and notices that the light reflection in the right cornea is at the 2 o'clock position and in the left cornea at the 10 o'clock position. Based on these data, the nurse should take what action? a. Document these findings as normal. b. Perform the cover-uncover test. c. Perform the confrontation test. d. Document these findings as abnormal.

b. Perform the cover-uncover test. A The findings are abnormal. The light should appear in the same location in each cornea. B The nurse is performing the corneal light reflex test and the findings are abnormal. Thus, when the corneal light reflex is asymmetric, the cover-uncover test is performed to determine which eye has the weak extraocular muscle(s). C The confrontation test is used to assess peripheral visual fields and is not appropriate to perform when the corneal light reflex is asymmetric. D The asymmetric corneal light reflex is abnormal, but the cover-uncover test should follow the abnormal finding to determine which eye has the weak extraocular muscle(s).

In assessing a neonate, the nurse notices that one testicle has not descended into the scrotal sac. What is the most appropriate response for the nurse as a result of this finding? a. Document the findings and refer this neonate for further examination for an undescended testicle. b. Place a finger over the upper inguinal ring and gently push downward to try to push the testicle into the scrotum. c. Use a light source to transilluminate the affected scrotal sack to determine if fluid is preventing the descent of the testicle. d. Insert the fifth finger into the inguinal ring to palpate for a hernia that may have prevented the testicle from descending.

b. Place a finger over the upper inguinal ring and gently push downward to try to push the testicle into the scrotum. A This response is premature without determining if the testicle can be manually descended into the scrotum. B If the testicle can be pushed into the scrotum, it is considered descended even though it retracts into the inguinal canal. C Transillumination is used to detect a hydrocele, which is a common finding in infants but not related to an undescended testicle. D This response is not appropriate.

A nurse suspects a patient has a chest wall injury and wants to collect more data about thoracic expansion. Which is the appropriate technique to use? a. Place the palmar side of each hand against the lateral thorax at the level of the waist, ask the patient to take a deep breath, and observe lateral movement of the hands. b. Place both thumbs on either side of the patient's T9 to T10 spinal processes, extend fingers laterally, ask the patient to take a deep breath, and observe lateral movement of the thumbs. c. Place both thumbs on either side of the patient's T7 to T8 spinal processes, extend fingers laterally, ask the patient to exhale deeply, and observe lateral inward movement of the thumbs. d. Place the palmar side of each hand on the shoulders of the patient, ask the patient to sit up straight and take a deep breath, and observe symmetric movement of the shoulders.

b. Place both thumbs on either side of the patient's T9 to T10 spinal processes, extend fingers laterally, ask the patient to take a deep breath, and observe lateral movement of the thumbs. A The palms of the hands are not used and hands are not placed on the lateral thorax. B This is the correct technique to assess thoracic expansion. C The thoracic level is too high and the patient does not exhale. D The hands are not placed on the shoulders.

The nurse suspects respiratory distress in a newborn infant who exhibits which manifestation? a. Respiratory rate of 36 breaths/min b. Sternal retractions c. Nasal breathing d. Irregular breathing pattern

b. Sternal retractions A Respiratory rate of 36 per minute in the newborn and infant is within the expected range of 30 to 60 breaths/min. B Several respiratory findings indicate that an infant is in respiratory distress, including stridor, grunting, sternal or supraclavicular retractions, and nasal flaring. C Infants are obligate nose breathers until about 3 months old. D The respiratory pattern in the newborn may be irregular, having a Cheyne-Stokes type of pattern.

How does the nurse perform a Weber test to assess hearing function? a. Whispers three to four words into the patient's ear and asks him to repeat the words heard b. Places a vibrating tuning fork in the middle of the head and asks the patient if the sound is heard the same in both ears c. Places a set of headphones over both ears, plays several tones, and asks the patient to identify the sounds d. Places a vibrating tuning fork on the mastoid process and asks the patient to signal when he can no longer hear the sound

b. Places a vibrating tuning fork in the middle of the head and asks the patient if the sound is heard the same in both ears A This technique describes the whisper test. B This technique describes the Weber test. C This technique describes the use of an audiometer. D This technique describes part of the Rinne test.

To assess jaw movement of an adult patient, the nurse uses which technique? a. Asking the patient to open the mouth and then passively moving the patient's open jaw from side to side b. Placing two fingers in front of each ear and asking the patient to slowly open and close the mouth c. Asking the patient to open the mouth and to resist the nurse's attempt to close the mouth d. Using the pads of all fingers to feel along the mandible for tenderness and nodules

b. Placing two fingers in front of each ear and asking the patient to slowly open and close the mouth A The patient's jaw movement should be active, not passive. B This is the correct technique for palpating the jaw. C This technique assesses strength of the jaw, which is not typically evaluated. D Palpating under the middle of the mandible may reveal the submental lymph node.

A patient admitted with pneumonia reports that she takes insulin for diabetes mellitus. In which section of the history does the nurse document the insulin and diabetes? a. Past health history b. Present health status c. Reason for seeking care (chief complaint) d. History of present illness

b. Present health status A The past health history includes categories of childhood illness, surgeries, hospitalizations, accidents or injuries, immunizations, and obstetric history. B The present health status documents the current health conditions, which include chronic diseases and medications taken. In this case, diabetes and taking insulin are not the reason for seeking care, but need to be managed while the patient's pneumonia is being treated because they may affect the patient's recovery from pneumonia. C The reason for seeking care (chief complaint) is a brief statement of the patient's purpose for requesting the services of a health care provider. D History of present illness further investigates the history of the present problem; best accomplished by conducting a symptom analysis.

An adolescent patient appears reluctant to discuss sensitive issues with her parents present. What is the nurse's most appropriate intervention? a. Tell the patient that it is very important to be honest and specific. b. Provide time when the adolescent is alone with the nurse. c. Reassure the patient that anything said in the interview is considered confidential. d. Ask the parents to answer the questions if the patient is not willing to answer.

b. Provide time when the adolescent is alone with the nurse. A Although this statement is true, the adolescent should have time alone with the nurse, if needed, to answer or ask personal questions. B As children reach adolescence, they should be given the option to provide sensitive parts of the history without their parents present. C Although this statement is true, the adolescent should have time alone with the nurse, if needed, to answer or ask personal questions. D This intervention is not appropriate when the patient is present and able to answer questions. In addition, the parents may not know the information needed by the nurse about the adolescent.

A patient complains of nasal drainage and sinus headache. The nurse suspects a nasal infection and anticipates observing which finding during examination? a. Foul-smelling drainage b. Purulent green-yellow drainage c. Bloody drainage d. Watery drainage

b. Purulent green-yellow drainage A Foul-smelling drainage is consistent with a foreign object in the nose. B Purulent green-yellow drainage is consistent with a nasal or sinus infection. C Bloody drainage is consistent with trauma to the nose. D Watery drainage is consistent with a nasal allergy.

During the history, a patient reports blurred vision, seeing double at times, and a glare from headlights from oncoming cars at night. Based on this information, what finding does the nurse expect to find on assessment of this patient's eyes? a. Anterior chamber depth is shallow. b. Red reflex is absent. c. Extraocular muscle movement is asymmetric. d. Retinal arteries are wider than retinal veins.

b. Red reflex is absent. A Shallow anterior chamber depth occurs in glaucoma. B The symptoms suggest cataracts. The red reflex cannot be seen because the light cannot penetrate the opacity of the lens. C Extraocular muscle movement is asymmetric. Cataracts affect the lens rather than the eye muscles. D Retinal arteries are wider than retinal veins. Cataracts affect the lens rather than the retinal vessels.

A patient has had an infected facial wound for more than 3 months. How does the nurse expect the patient's enlarged lymph nodes to feel? a. Soft, edematous, and tender b. Round, tender, and movable c. Hard, nontender, and nonmobile d. Irregularly shaped, tender, and firm

b. Round, tender, and movable A These are not characteristics of lymph nodes associated with inflammation. B These are characteristics of enlarged lymph nodes associated with inflammation. C These are characteristics of enlarged lymph nodes associated with a malignancy. D These are not characteristics of lymph nodes associated with inflammation.

A community organization sponsors a health fair to increase awareness of colon cancer. At the health fair, colorectal cancer screening kits are distributed, and health care professionals answer questions, take blood pressure, and distribute literature. These activities are examples of _____ prevention. a. Primary b. Secondary c. Tertiary d. Risk factor

b. Secondary A Primary prevention is focused on preventing disease from developing through the promotion of a healthy lifestyle. B Secondary prevention consists of screening efforts to promote early detection of disease—in this scenario, colorectal cancer and hypertension. C Tertiary prevention is directed toward minimizing the disability from chronic disease and helping the patient maximize his or her health. D Risk factor prevention is part of primary prevention that focuses on preventing disease by managing risk factors.

What technique does the nurse use to test ankle clonus? a. Strokes the lateral aspect of the sole of the patient's foot from heel to ball with a reflex hammer b. Supports the patient's knee in flexed position and sharply dorsiflexes the foot and maintains the flexion c. Plantar flexes the ankle and strikes the appropriate tendon of the ankle with the hammer d. Everts the ankle and slowly moves the ankle into plantar flexion and quickly release the foot

b. Supports the patient's knee in flexed position and sharply dorsiflexes the foot and maintains the flexion A This is the technique for testing plantar flex or the Babinski reflex. B This is the correct technique for assessing ankle clonus. C This is not a correct technique for testing any reflex. D This is not a correct technique for testing any reflex.

During the Rinne test, a nurse determines that the patient hears the tuning fork held on the mastoid process for 15 seconds and hears the tuning fork held in front of the ear for 30 seconds. The same results are found in both ears. Based on this finding, what is the most appropriate response of the nurse? a. Repeat the test again using a 2000 Hz tuning fork. b. Tell the patient that this represents an expected finding. c. Refer the patient for additional testing to detect hearing abnormality. d. Perform a Weber test to confirm the findings of the Rinne test.

b. Tell the patient that this represents an expected finding. A This is unnecessary because the finding of the Rinne test was normal. B This is a normal finding. Air conduction (30 seconds) is twice as long as bone conduction (15 seconds). C This is unnecessary because the finding of the Rinne test was normal. D This is unnecessary because the finding of the Rinne test was normal.

What does the S2 heart sound represent? a. The beginning of systole. b. The closure of the aortic and pulmonic valves. c. The closure of the tricuspid and mitral values d. A split heard sound on exhalation

b. The closure of the aortic and pulmonic valves. A The beginning of systole is the S1 heart sound. B The second heart sound is made by the closing of these valves, which indicates the beginning of diastole. C The tricupid and mitral valves create the S1 heart sound. D A split sound on exhalation is not a correct statement.

During a well-baby check for several 4-month-old infants, a nurse recognizes that which infant needs further assessment of an abnormal finding? a. The infant who is unable to sit independently b. The infant whose head circumference and chest circumference are equal c. The infant whose weight has doubled since birth d. The infant whose length falls in the 90th percentile on growth charts

b. The infant whose head circumference and chest circumference are equal A This is not an expected motor skill for a 4-month-old; it is expected at 6 months of age. B At four months of age, the head circumference should be larger than the chest circumference. C This is a normal finding; infants generally double their birth weight by age 4 to 5 months. D This is not an abnormal finding, especially if weight is normal; the height of the parents should also be considered.

A patient is suspected of having a lung consolidation. A nurse uses the three techniques for assessing vocal resonance in this patient. What is the expected finding among the three procedures that will help eliminate consolidation as a problem? a. The nurse documents clearly hearing the patient say "99." b. The nurse documents hearing muffled sounds when the patient says "1-2-3." c. The nurse documents hearing no sounds when the patient says "e-e-e." d. The nurse documents clearly hearing the patient say "a-a-a."

b. The nurse documents hearing muffled sounds when the patient says "1-2-3." A Clear sounds are heard when a consolidation is present. B Muffled sounds of "1-2-3," "e-e-e," or "99" are heard when no consolidation is found. C Clear sounds are heard when a consolidation is present. D Clear sounds are heard when a consolidation is present.

Which patient in the eye clinic should the nurse assess first? a. The patient who reports a gradual clouding of vision b. The patient who complains of sudden loss of vision c. The patient who complains of double vision d. The patient who complains of poor night vision

b. The patient who complains of sudden loss of vision A A gradual clouding of vision is a symptom of cataracts that develop slowly and do not require immediate assessment. B Sudden vision loss may indicate a detached retina and requires immediate referral. C Double vision is a symptom of cataracts that develop slowly and do not require immediate assessment. D Poor night vision is a symptom of cataracts that develop slowly and do not require immediate assessment.

For which person is a screening assessment indicated? a. The person who had abdominal surgery yesterday b. The person who is unaware of his high serum glucose levels c. The person who is being admitted to a long-term care facility d. The person who is beginning rehabilitation after a knee replacement

b. The person who is unaware of his high serum glucose levels A A shift assessment is most appropriate for the person who is recovering in the hospital from surgery. B A screening assessment is performed for the purpose of disease detection. In this case this person may have diabetes mellitus. C A comprehensive assessment is performed during admission to a facility to obtain a detailed history and complete physical examination. D An episodic or follow-up assessment is performed after knee replacement to evaluate the outcome of the procedure.

A nurse inspects the abdomen for skin color, surface characteristics, and surface movement. What part of the abdominal assessment does the nurse perform next? a. Palpate lightly for tenderness and muscle tone. b. The tip of the middle finger of the dominant hand strikes the nail of the middle finger touching the skin of the abdomen. c. Palpate deeply for masses or aortic pulsation. d. Percuss for tones.

b. The tip of the middle finger of the dominant hand strikes the nail of the middle finger touching the skin of the abdomen. A Palpating lightly for tenderness and muscle tone is performed after auscultation. B Auscultation for bowel sounds occurs before palpating and percussing the abdomen. C Palpating deeply for masses or aortic pulsation is performed after light palpation. D Percussion for tones is performed after palpation.

Which finding during inspection of the mouth of a 1-month-old infant requires further investigation? a. A small loose tooth in the lower jaw b. Tongue overlapping the floor of the mouth c. Whitish epithelial cells on the roof of the mouth d. White patches on the tongue that scrape off easily

b. Tongue overlapping the floor of the mouth A This is not an abnormal finding; occasionally, a natal loose tooth may be found. B The infant's tongue should be appropriate to the size of the mouth and fit well into the floor of the mouth. C These small, white epithelial cells on the palate or gums are called Bohn nodules or Epstein pearls and are an expected finding. D Whitish patches seen along the mucosa that scrape off easily are milk deposits. White patches that scrape off but leave a red area that may bleed indicate candidiasis lesions.

Which valve does a nurse auscultate when the stethoscope is placed on the fourth intercostal space at the left of the sternal border? a. Pulmonic b. Tricuspid c. Mitral d. Aortic

b. Tricuspid A Pulmonic valve sounds are best heard in the second intercostal space at the left of the sternal border. B Tricuspid valve sounds are best heard in the fourth intercostal space at the left of the sternal border. C Mitral valve sounds are best heard in the fifth intercostal space at the midclavicular line. D Aortic valve sounds are best heard in the second intercostal space at the right of the sternal border.

A teenager comes to the clinic complaining about the whiteheads and blackhead on his face interfering with his social life. During the examination the nurse palpates an enlarged submental lymph node. Where is this lymph node located? a. In front of the ear b. Under the mandible c. At the base of the skull d. Along the angle of the jaw

b. Under the mandible A This is the location of the preauricular lymph nodes. B This is the location of the submental lymph node. C This is the location of the occipital lymph nodes. D This is the location of the parotid lymph nodes.

In assessing a patient with head injury, the nurse should be most concerned with which finding? a. Pain on palpation of the scalp b. Unilateral clear, watery nasal discharge c. A scalp laceration at the sight of injury d. Complaints of dizziness

b. Unilateral clear, watery nasal discharge A This is expected after a head injury and is not a cause for concern. B This may be cerebrospinal fluid, indicating a skull fracture. C This is expected after a head injury and is not a cause for concern. D This is expected after a head injury and is not a cause for concern.

A male patient is very talkative and shares much information that is not relevant to his history or the reason for his admission. Which action by the nurse improves data collection in this situation? a. Terminate the interview. b. Use closed-ended questions. c. Ask the patient to stay on the subject. d. Ask another nurse to complete the interview.

b. Use closed-ended questions. A Terminating the interview is not beneficial to the patient and does not allow data collection. B Using closed-ended questions is useful to obtain specific data when open-ended questions are not obtaining the needed data. C Asking the patient to stay on the subject is not therapeutic and may result in less data collection. D Asking another nurse to complete the interview may not be practical and interrupts the nurse-patient relationship that has been established.

Which technique used by the nurse encourages a patient to continue talking during an interview? a. Laughing and smiling during conversation b. Using phrases such as "Go on," and "Then?" c. Repeating what the patient said, but using different words d. Asking the patient to clarify a point

b. Using phrases such as "Go on," and "Then?" A Laughing and smiling during conversation may show attentiveness during the interview, but does not encourage more talking. B Using phrases such as "Go on" and "Then?" encourages the patient to continue talking. C Rephrasing what the patient has said is restatement. It confirms your interpretation of what they said, but does not encourage additional talking. D Asking the patient to clarify a point is done when the information is conflicting, vague, or ambiguous.

In assessing a patient with damage to the occipital lobe, the nurse correlates which clinical manifestation to this injury? a. Intentional tremors b. Visual changes c. Decreased hearing d. Inability to formulate words

b. Visual changes A Intentional tremors are caused by cerebellar problems. B The occipital lobe contains the visual cortex. C The temporal lobe contains the auditory cortex. D The ability to formulate words comes from the Broca area in the frontal lobe.

In which situation is the nurse's use of closed-ended questions most appropriate? a. When clarifying vague or conflicting data b. When obtaining a history from an overly-talkative patient c. When encouraging a patient to elaborate on details of his or her history d. When collecting data about the current health problem

b. When obtaining a history from an overly-talkative patient A When clarifying vague and conflicting data, the nurse needs to use open-ended questions to obtain data. B When obtaining a history from an overly-talkative patient, a nurse can resort to closed-ended questions to complete the data collection in a timely manner. C When encouraging the patient to elaborate on details of his or her history, the nurse needs to use open-ended questions to obtain the details. D When collecting data about the current problem, the patient needs to describe the symptoms that brought him or her to seek help. These details are not collected with closed-ended questions.

As a patient is walking down the hall, the nurse notices the patient's staggering, unsteady gait. What findings does the nurse anticipate on the neurologic examination? a. When the patient stands with feet together, eyes open and then closed, an upright posture is maintained. b. When the patient touches the end of each finger to the thumb of the same hand, a tremor is observed in the fingers. c. When the patient is giving a history to the nurse, a tremor is noticed as the patient's hands rest in the lap. d. When lying supine, the patient is able to move the heel of one foot down the shin of the other leg.

b. When the patient touches the end of each finger to the thumb of the same hand, a tremor is observed in the fingers. A This is a result of a negative Romberg test. This patient has a cerebellar problem, which would result in a positive Romberg test. B This patient has a cerebellar problem as evidenced by the staggering gait (noted at the beginning of the encounter) and the intention tremor on movement (noted during the examination). C This describes a tremor at rest that occurs in patients with parkinsonism rather than with cerebellar problems. D This describes a normal response on an examination of cerebellar function.

A nurse attends a seminar on teaching/learning. Which statement indicates the nurse has a good understanding of teaching/learning? a. "Teaching and learning can be separated." b. "Learning is an interactive process that promotes teaching." c. "Teaching is most effective when it responds to the learner's needs." d. "Learning consists of a conscious, deliberate set of actions designed to help the teacher."

c. "Teaching is most effective when it responds to the learner's needs." Teaching is most effective when it responds to the learner's needs. It is impossible to separate teaching from learning. Teaching is an interactive process that promotes learning. Teaching consists of a conscious, deliberate set of actions that help individuals gain new knowledge, change attitudes, adopt new behaviors, or perform new skills.

The nurse considers several new female patients to receive additional teaching on the need for more frequent Pap test and gynecological examinations. Which assessment findings reveal the patient at highest risk for cervical cancer and having the greatest need for patient education? a. 13 years old, nonsmoker, not sexually active b. 15 years old, social smoker, celibate c. 22 years old, smokes 1 pack of cigarettes per day, has multiple sexual partners d. 50 years old, stopped smoking 30 years ago, has history of multiple pregnancies

c. 22 years old, smokes 1 pack of cigarettes per day, has multiple sexual partners Females considered to be at higher risk include those who smoke, have multiple sex partners, and have a history of sexually transmitted infections. Of all the assessment findings listed, the 22-year-old smoker with multiple sexual partners has the greatest number of risk factors for cervical cancer. The other patients are at lower risk: not sexually active, celibate, and do not smoke.

Evaluate the outcomes of patient education for hypertension. a. 1, 3, 2, 4 b. 2, 3, 1, 4 c. 3, 1, 2, 4 d. 3, 2, 1, 4

c. 3, 1, 2, 4 Assessment is the first step of any teaching session, then diagnosing, planning (goals), implementation, and evaluation.

Which assessment finding will cause the nurse to begin teaching a patient because the patient is ready to learn? a. A patient has the ability to grasp and apply the elastic bandage. b. A patient has sufficient upper body strength to move from a bed to a wheelchair. c. A patient with a below-the-knee amputation is motivated about how to walk with assistive devices. d. A patient has normal eyesight to identify the markings on a syringe and coordination to handle a syringe.

c. A patient with a below-the-knee amputation is motivated about how to walk with assistive devices. Motivation underlies a person's desire or willingness to learn. Motivation is a force that acts on or within a person (e.g., an idea, emotion, or a physical need) to cause the person to behave in a particular way. For example, a patient with a below-the-knee amputation is motivated to learn how to walk with assistive devices, indicating a readiness to learn. Do not confuse readiness to learn with ability to learn. All the other answers are examples of ability to learn because this often depends on the patient's level of physical development and overall physical health. To learn psychomotor skills, a patient needs to possess a certain level of strength, coordination, and sensory acuity. For example, it is useless to teach a patient to transfer from a bed to a wheelchair if he or she has insufficient upper body strength. An older patient with poor eyesight or an inability to grasp objects tightly cannot learn to apply an elastic bandage or handle a syringe.

An advanced practice nurse is preparing to assess the external genitalia of a 25-year-old American woman of Chinese descent. Which action will the nurse do first? a. Place the patient in the lithotomy position. b. Drape the patient to enhance patient comfort. c. Assess the patient's feelings about the examination. d. Ask the patient if she would like her mother to be present in the room.

c. Assess the patient's feelings about the examination. Patients who are Chinese American often believe that examination of the external genitalia is offensive. Before proceeding with the examination, the nurse first determines how the patient feels about the procedure and explains the procedure to answer any questions and to help the patient feel comfortable with the assessment. Once the patient is ready to have her external genitalia examined, the nurse places the patient in the lithotomy position and drapes the patient appropriately. Typically, nurses ask adolescents if they want a parent present during the examination. The patient in this question is 25 years old; asking if she would like her mother to be present is inappropriate.

A patient with heart failure is learning to reduce salt in the diet. When will be the best time for the nurse to address this topic? a. At bedtime, while the patient is relaxed b. At bath time, when the nurse is cleaning the patient c. At lunchtime, while the nurse is preparing the food tray d. At medication time, when the nurse is administering patient medication

c. At lunchtime, while the nurse is preparing the food tray In this situation, because the teaching is about food, coordinating it with routine nursing care that involves food can be effective. Many nurses find that they are able to teach more effectively while delivering nursing care. For example, while hanging blood, you explain to the patient why the blood is necessary and the symptoms of a transfusion reaction that need to be reported immediately. At bedtime would be a good time to discuss routines that enhance sleep. At bath time would be a good time to describe skin care and how to prevent pressure ulcers. At medication time would be a good time to explain the purposes and side effects of the medication.

During a sexually transmitted illness presentation to high-school students, the nurse recommends the human papillomavirus (HPV) vaccine series. Which condition is the nurse trying to prevent? a. Breast cancer b. Ovarian cancer c. Cervical cancer d. Testicular cancer

c. Cervical cancer Human papillomavirus (HPV) infection increases the person's risk for cervical cancer. HPV vaccine is recommended for females aged 11 to 12 years but can be given to females ages 12 through 26; males can also receive the vaccine. HPV is not a risk factor for breast, ovarian, and testicular cancer.

The nurse completed assessments on several patients. Which assessment finding will the nurse record as normal? a. Pulse strength 3 b. 1+ pitting edema c. Constricting pupils when directly illuminated d. Hyperactive bowel sounds in all four quadrants

c. Constricting pupils when directly illuminated A normal finding is pupils constricting when directly illuminated with a penlight. A pulse strength of 3 indicates a full or increased pulse; 2 is normal. 1+ pitting edema is abnormal; there should be no edema for a normal finding. Hyperactive bowel sounds are abnormal and indicate increased GI motility; normal bowel sounds are active.

A nurse is preparing to teach a patient about smoking cessation. Which factors should the nurse assess to determine a patient's ability to learn? a. Sociocultural background and motivation b. Stage of grieving and overall physical health c. Developmental capabilities and physical capabilities d. Psychosocial adaptation to illness and active participation

c. Developmental capabilities and physical capabilities Developmental and physical capabilities reflect one's ability to learn. Sociocultural background and motivation are factors determining readiness to learn. Psychosocial adaptation to illness and active participation are factors in readiness to learn. Readiness to learn is related to the stage of grieving. Overall physical health does reflect ability to learn; however, because it is paired here with stage of grieving (which is a readiness to learn factor), this is incorrect.

A nurse is teaching a culturally diverse patient with a learning disability about nutritional needs. What must the nurse do first before starting the teaching session? a. Obtain pictures of food. b. Get an interpreter. c. Establish a rapport. d. Refer to a dietitian.

c. Establish a rapport. Establishing trust is important for all patients, especially culturally diverse and learning disabled patients, before starting teaching sessions. Obtaining pictures of food, getting an interpreter, and referring to a dietitian all occur after rapport/trust is established.

The patient has had a stroke that has affected the ability to speak. The patient becomes extremely frustrated when trying to speak. The patient responds correctly to questions and instructions but cannot form words coherently. Which type of aphasia is the patient experiencing? a. Sensory b. Receptive c. Expressive d. Combination

c. Expressive The two types of aphasias are sensory (or receptive) and motor (or expressive). The patient cannot form words coherently, indicating expressive or motor aphasia is present. The patient responds correctly to questions and instructions, indicating receptive or sensory aphasia is not present. Patients sometimes suffer a combination of receptive and expressive aphasia, but this is not the case here.

On admission, a patient weighs 250 pounds. The weight is recorded as 256 pounds on the second inpatient day. Which condition will the nurse assess for in this patient? a. Anorexia b. Weight loss c. Fluid retention d. Increased nutritional intake

c. Fluid retention This patient has gained 6 pounds in a 24-hour period. A weight gain of 5 pounds (2.3 kg) or more in a day indicates fluid retention problems, not nutritional intake. A weight loss is considered significant if the patient has lost more than 5% of body weight in a month or 10% in 6 months. A downward trend may indicate a reduction in nutritional reserves that may be caused by decreased intake such as anorexia.

The nurse is assessing skin turgor. Which technique will the nurse use? a. Press lightly on the forearm. b. Press lightly on the fingertips. c. Grasp a fold of skin on the sternal area. d. Grasp a fold of skin on the back of the hand.

c. Grasp a fold of skin on the sternal area. To assess skin turgor, grasp a fold of skin on the back of the forearm or sternal area with the fingertips and release. Since the skin on the back of the hand is normally loose and thin, turgor is not reliably assessed at that site. Pressing lightly on the forearm can be used to assess for pitting edema or pain or sense of touch. Pressing lightly on the fingertips and observing nail color is assessing capillary refill.

During a school physical examination, the nurse reviews the patient's current medical history. The nurse discovers the patient has allergies. Which assessment finding is consistent with allergies? a. Clubbing b. Yellow discharge c. Pale nasal mucosa d. Puffiness of nasal mucosa

c. Pale nasal mucosa Pale nasal mucosa with clear discharge indicates allergy. Clubbing is due to insufficient oxygenation at the periphery resulting from conditions such as chronic emphysema and congenital heart disease; it is noted in the nails. A sinus infection results in yellowish or greenish discharge. Habitual use of intranasal cocaine and opioids causes puffiness and increased vascularity of the nasal mucosa.

The patient presents to the clinic with dysuria and hematuria. How does the nurse proceed to assess for kidney inflammation? a. Uses deep palpation posteriorly. b. Lightly palpates each abdominal quadrant. c. Percusses posteriorly the costovertebral angle at the scapular line. d. Inspects abdomen for abnormal movement or shadows using indirect lighting.

c. Percusses posteriorly the costovertebral angle at the scapular line. With the patient sitting or standing erect, use direct or indirect percussion to assess for kidney inflammation. With the ulnar surface of the partially closed fist, percuss posteriorly the costovertebral angle at the scapular line. If the kidneys are inflamed, the patient feels tenderness during percussion. Use a systematic palpation approach for each quadrant of the abdomen to assess for muscular resistance, distention, abdominal tenderness, and superficial organs or masses. Light palpation would not detect kidney tenderness because the kidneys sit deep within the abdominal cavity. Posteriorly, the lower ribs and heavy back muscles protect the kidneys, so they cannot be palpated. Kidney inflammation will not cause abdominal movement. However, to inspect the abdomen for abnormal movement or shadows, the nurse should stand on the patient's right side and inspect from above the abdomen using direct light over the abdomen.

A nurse is teaching an older-adult patient about strokes. Which teaching technique is mostappropriate for the nurse to use? a. Speak in a high tone of voice to describe strokes. b. Use a pamphlet about strokes with large font in blues and greens. c. Provide specific information about strokes in short, small amounts. d. Begin the teaching session facing the teaching white board with stroke information.

c. Provide specific information about strokes in short, small amounts. With older adults, keep the teaching session short with small amounts of information. Also, if using written material, assess the patient's ability to read and use information that is printed in large type and in a color that contrasts highly with the background (e.g., black 14-point print on matte white paper). Avoid blues and greens because they are more difficult to see. Speak in a low tone of voice (lower tones are easier to hear than higher tones). Directly face the older-adult learner when speaking.

A patient has been taught how to change a colostomy bag but is having trouble measuring and manipulating the equipment and has many questions. What is the nurse's next action? a. Refer to a mental health specialist. b. Refer to a wound care specialist. c. Refer to an ostomy specialist. d. Refer to a dietitian.

c. Refer to an ostomy specialist. Resources that specialize in a particular health need (e.g., wound care or ostomy specialists) are integral to successful patient education. A mental health specialist is helpful for emotional issues rather than for physical problems. A dietitian is a resource for nutritional needs. A wound care specialist provides complex wound care.

A nurse is preparing to perform a complete physical examination on a weak, older-adult patient with bilateral basilar pneumonia. Which position will the nurse use? a. Prone b. Sims' c. Supine d. Lateral recumbent

c. Supine Supine is the most normally relaxed position. If the patient becomes short of breath easily, raise the head of the bed. Supine position would be easiest for a weak, older-adult person during the examination. Lateral recumbent and prone positions cause respiratory difficulty for any patient with respiratory difficulties. Sims' position is used for assessment of the rectum and the vagina.

A nurse is teaching a patient about healthy eating habits. Which learning objective/outcome for the affective domain will the nurse add to the teaching plan? a. The patient will state three facts about healthy eating. b. The patient will identify two foods for a healthy snack. c. The patient will verbalize the value of eating healthy. d. The patient will cook a meal with low-fat oil.

c. The patient will verbalize the value of eating healthy. Affective learning deals with expression of feelings and acceptance of attitudes, beliefs, or values. Having the patient value healthy eating habits falls within the affective domain. Stating three facts or identifying two foods for a healthy snack falls within the cognitive domain. Cooking falls within the psychomotor domain.

The nurse is assessing the tympanic membranes of an infant. Which action by the nurse demonstrates proper technique? a. Pulls the auricle upward and backward. b. Holds handle of the otoscope between the thumb and little finger. c. Uses an inverted otoscope grip while pulling the auricle downward and back. d. Places the handle of the otoscope between the thumb and index finger while pulling the auricle upward.

c. Uses an inverted otoscope grip while pulling the auricle downward and back. Using the inverted otoscope grip while pulling the auricle downward and back is a common approach with infant/child examinations because it prevents accidental movement of the otoscope deeper into the ear canal, as could occur with an unexpected pediatric reaction to the ear examination. The other techniques could result in injury to the infant's tympanic membrane. Insert the scope while pulling the auricle upward and backward in the adult and older child. Hold the handle of the otoscope in the space between the thumb and index finger, supported on the middle finger.

A nurse is auscultating different areas on an adult patient. Which technique should the nurse use during an assessment? a. Uses the bell to listen for lung sounds b. Uses the diaphragm to listen for bruits c. Uses the diaphragm to listen for bowel sounds d. Uses the bell to listen for high-pitched murmurs

c. Uses the diaphragm to listen for bowel sounds The bell is best for hearing low-pitched sounds such as vascular (bruits) and certain heart sounds (low-pitched murmurs), and the diaphragm is best for listening to high-pitched sounds such as bowel and lung sounds and high-pitched murmurs.

A toddler is going to have surgery on the right ear. Which teaching method is most appropriate for this developmental stage? a. Encourage independent learning. b. Develop a problem-solving scenario. c. Wrap a bandage around a stuffed animal's ear. d. Use discussion throughout the teaching session.

c. Wrap a bandage around a stuffed animal's ear. Use play to teach a procedure or activity (e.g., handling examination equipment, applying a bandage to a doll) to toddlers. Encouraging independent learning is for the young or middle adult. Use of discussion is for older children, adolescents, and adults, not for toddlers. Use problem solving to help adolescents make choices. Problem solving is too advanced for a toddler.

Barriers to patient education the nurse considers in implementing a teaching plan include a. family resources. b. high school education. c. hunger and pain. d. need perceived by patient.

c. hunger and pain. A patient who is hungry or in pain has limited ability to concentrate or learn. Family resources would be considered in developing a plan of care and could be an asset or a barrier to patient education. The patient's educational level would be considered in planning teaching strategies but would not be a barrier to education. A need perceived by a patient would provide motivation for learning and would not be a barrier.

The most appropriate resources to include when planning to provide patient education related to a goal in the psychomotor domain would be a. diagnosis-related support groups. b. Internet resources. c. manikin practice sessions. d. self-directed learning modules.

c. manikin practice sessions. A teaching goal in the psychomotor domain should be matched with teaching strategies in the psychomotor domain, such as demonstration, practice sessions with a manikin, and return demonstrations. Diagnosis-related support groups would be most effective with goals in the affective domain. Internet resources would be most effective for goals in the cognitive domain. Self-directed learning modules would be most effective for goals in the cognitive domain.

In taking a history from an adolescent girl about diet and nutrition, a nurse specifically asks which question? a. "How frequently do you eat fast food or junk food?" b. "Which carbonated drinks do you drink most often?" c. "Do you have any food restrictions or diet routines?" d. "What are your favorite fruits and vegetables?"

c. "Do you have any food restrictions or diet routines?" A Asking the frequency of fast food or junk food consumption does not give data about what food is eaten. B Knowing the amount of carbonated drinks provides more useful data. C Adolescents should be asked specifically about their perception of their current weight and behaviors associated with eating disorders, including food restrictions, extreme diet/exercise routines, binging or purging, and the use of laxatives to screen for eating disorders. D Knowing how frequently these foods are eaten provides more useful data.

What instructions does the nurse give the patient before palpating the right supraclavicular lymph nodes? a. "Lean your head backward and toward the right as far as comfortably possible." b. "Lie supine and turn your head away from the right side." c. "Draw up your shoulders forward, and flex your chin toward the right side." d. "Sit up, raise both arms over your head, and flex your chin away from the right side."

c. "Draw up your shoulders forward, and flex your chin toward the right side." A This is incorrect. The patient should draw up (hunch) the shoulders forward rather than leaning back. B The patient should be sitting, rather than lying down. C This is the technique for palpating the supraclavicular nodes. D The shoulders should be drawn up (hunched) forward, rather than raising the arms.

What instructions does the nurse give the patient before using the Snellen visual acuity chart? a. "Remove your eyeglasses before attempting to read the lowest line." b. "Stand 10 feet from the chart and read the first line aloud." c. "Hold a white card over one eye and read the smallest possible line." d. "Squint if necessary to improve the ability to read the largest letters."

c. "Hold a white card over one eye and read the smallest possible line." A Patients should wear their glasses when visual acuity is tested. B The patient should stand 20 feet from the Snellen chart. C This is the appropriate technique for using the Snellen chart. D The patient should not squint to see the chart.

The nurse is taking a health history on a patient who reports frequent headaches with pain in the front of the head, but sometimes felt in the back of the head. Which statement by the patient is most indicative of tension headaches? a. "I usually have nausea and vomiting with my headaches." b. "My whole head is constantly throbbing." c. "It feels like my head is in a vice." d. "The pain is on the left side over my eye, forehead, and cheek."

c. "It feels like my head is in a vice." A This is descriptive of migraines rather than tension headaches. B This is descriptive of migraines rather than tension headaches. C This is descriptive of tension headaches, which is consistent with the rest of the data reported by the patient. D This is consistent with cluster headaches rather than tension headaches.

During symptom analysis, the nurse helps the patient distinguish between dizziness and vertigo. Which description by the patient indicates vertigo? a. "I felt faint, like I was going to pass out." b. "I just could not keep my balance when I sat up." c. "It seemed that the room was spinning around." d. "I was afraid that I was going to lose consciousness."

c. "It seemed that the room was spinning around." A This is a description of lightheadedness, a form of dizziness. B This is a description of disequilibrium, a form of dizziness. C This is consistent with vertigo because it includes a sensation of motion. D This is a description of syncope, a form of dizziness.

While obtaining a symptom analysis from a patient who has an inner ear infection, the nurse helps the patient distinguish between dizziness and vertigo. Which description by the patient indicates vertigo? a. "I felt lightheaded when I stood up." b. "I just could not keep my balance when I sat up." c. "It seemed that the room was spinning around." d. "I was afraid that I was going to lose consciousness."

c. "It seemed that the room was spinning around." A This is a description of dizziness that is often associated with transient ischemia attacks. B This is a description of disequilibrium, a form of dizziness. C This report is consistent with vertigo because it includes a sensation of motion. D This is a description of presyncope, a form of dizziness.

Which statement by a mother makes the nurse assess the infant girl for cardiovascular problems? a. "She has gained 2 lb since our last visit." b. "She naps twice a day for almost 2 hours each time." c. "She gets so tired and out of breath when she takes her bottle." d. "She gets fussy after I feed her and seems to have lots of gas."

c. "She gets so tired and out of breath when she takes her bottle." A This is an expected weight gain for an infant. B This is an expected sleep pattern for an infant. C Heart problems are indicated when caregivers report the child stops eating to catch her breath. D This finding may be related to the food eaten, rather than a cardiovascular problem.

Which statement by the nurse demonstrates a patient-centered interview? a. "I need to complete this questionnaire about your medical and family history." b. "The hospital requires me to complete this assessment as soon as possible." c. "Tell me about the symptoms you've been having." d. "I've had the same symptoms that you've described."

c. "Tell me about the symptoms you've been having." A "I need to complete this questionnaire about your medical and family history" focuses on the nurse's need to complete the assessment rather than the needs of the patient. B "The hospital requires me to complete this assessment as soon as possible" focuses on the nurse's need to meet hospital requirements rather than the needs of the patient. C "Tell me about the symptoms you've been having" focuses on the needs of the patient so that the patient is free to share concerns, beliefs, and values in his or her own words. D "I've had the same symptoms that you've described" focuses on the nurse rather than on the patient.

A patient reports having abdominal distention and having vomited several times yesterday and today. What question is appropriate for the nurse to ask in response to this information? a. "Has there been a change in your usual pattern of urination?" b. "Did you have heartburn before the vomiting?" c. "What did the vomitus look like?" d. "Have you noticed a change in the color of your urine or stools?"

c. "What did the vomitus look like?" A "Has there been a change in your usual pattern of urination?" is not a question related to abdominal distention and vomiting. B Have you noticed a change in the color of your urine or stools?" is not a question related to abdominal distention and vomiting. It is related to elevated bilirubin from liver or gallbladder disease and is accompanied by jaundice. C "What did the vomitus look like?" is an appropriate question because the characteristics of the vomitus may help determine its cause. Acute gastritis leads to vomiting of stomach contents, obstruction of the bile duct results in greenish-yellow vomitus, and an intestinal obstruction may cause a fecal odor to the vomitus. D This is not a question related to the abdominal distention and vomiting. Heartburn applies more to gastric disorders, such as gastroesophageal reflux disease or hiatal hernia.

A 75-year-old male patient asks how to reduce his risk of esophageal cancer. What is the nurse's most appropriate response? a. "Don't worry about it, esophageal cancers have a low incidence in men." b. "You should not be concerned about esophageal cancer at your age." c. "You should consider limiting your alcohol intake to two drinks per day." d. "Increasing the fiber and protein in your diet can help you lower your risk."

c. "You should consider limiting your alcohol intake to two drinks per day." A Men have a rate three times that of women. B The risk increases with age, with the peak between 70 and 80 years. C "You should consider limiting your alcohol intake to two drinks per day. Long-term alcohol intake increases your risk for esophageal cancer." D Although fiber and protein are important for the diet, their intake does not affect the risk of esophageal cancer.

A patient tells the nurse that she has smoked two packs of cigarettes a day for 20 years. The nurse records this as how many pack-years? a. 10 b. 20 c. 40 d. 60

c. 40 A This incorrect calculation was made by dividing 20 years by 2 packs. B This is correct if the patient smoked 1 pack per day for 20 years. C Two packs of cigarettes ´ 20 years = 40 pack-years. D This is correct if the patient smoked 3 packs per day for 20 years or 2 packs a day for 30 years.

The paramedics transport an adult involved in a motor vehicle accident to the emergency department. On physical examination, the patient's level of consciousness is reported as opening eyes to pain and responding with inappropriate words and flexion withdrawal to painful stimuli. Which value will the nurse report for the patient's Glasgow Coma Scale score? a. 5 b. 7 c. 9 d. 11

c. 9 According to the guidelines of the Glasgow Coma Scale, the patient has a score of 9. Opening eyes to pain is 2 points; inappropriate word use is 3 points; and flexion withdrawal is 4 points. The total for this patient is 2 + 3 + 4 = 9.

A nurse refers which child for further assessment? a. A 2-year-old who has a jugular venous hum after playing b. A 4-year-old who has a resting heart rate of 100 c. A 5-year-old who positions herself in a squat after running a few feet d. A 7-year-old who has a strong femoral pulse readily detected on palpation

c. A 5-year-old who positions herself in a squat after running a few feet A An expected finding in children is a venous hum in the jugular vein. B This is an expected resting heart rate for a child; the expected range for a toddler is from 80 to 110 and for a school-age child from 60 to 110 beats/min. C Squatting may be a compensatory position for a child with a heart defect. D This is an expected finding.

When palpating the abdomen to determine a floating mass, a nurse presses on the abdomen at a 90-degree angle with the fingertips. Which finding indicates a mass? a. An increase in abdominal girth b. A complaint from the patient of a dull pain in the flank area c. A freely movable mass will float upward and touch the fingertips d. Fluid in the abdomen will shift upward and touch the fingertips

c. A freely movable mass will float upward and touch the fingertips A An increase in abdominal girth does not occur as a result of ballottement. B A complaint from the patient of a dull pain in the flank area is not an expected finding. C A freely movable mass floating upward and touching the fingertips is the expected finding (ballottement). D Fluid in the abdomen shifting upward and touching the fingertips does not occur; it is the mass on the abdomen that shifts upward.

When auscultating the heart of a patient with pericarditis, the nurse expects to hear which sound? a. A systolic murmur b. An S3 heart sound c. A friction rub d. An S4 heart sound

c. A friction rub A Most systolic murmurs are caused by obstruction of the outflow of the semilunar valves or by incompetent AV valves. B An S3 heart sound occurs when there is heart failure. C Two classic findings of pericarditis are pericardial friction rub and chest pain. D An S4 heart sound occurs when there is hypertrophy of the ventricle.

A nurse who is auscultating a patient's heart hears a harsh sound, a raspy machine-like blowing sound, after S1 and before S2. How does this nurse document this finding? a. An opening snap b. A diastolic murmur c. A systolic murmur d. A pericardial friction rub

c. A systolic murmur A An opening snap is caused by the opening of the mitral or tricuspid valve and is an abnormal sound heard in diastole when either valve is thickened, stenotic, or deformed. The sounds are high pitched and occur early in diastole. B A diastolic murmur is heard after the S2 heart sound at the beginning of diastole. C The blowing sound is a murmur. The nurse determines whether it is a systolic or a diastolic murmur based on where it is heard during the cardiac cycle. S1 indicates the beginning of systole; the sound is made by the closing of the mitral and tricuspid valves, which is followed by ventricular contraction or systole. D Pericardial friction rubs have a rubbing sound that is usually present in both diastole and systole, and is best heard over the apical area.

A nurse examines a patient with a pleural effusion and finds decreased fremitus. What additional abnormal finding should the nurse anticipate during further examination? a. An increase in the anteroposterior to lateral ratio b. Hyperresonance over the affected area c. Absent breath sounds in the affected area d. Increased vocal fremitus over the affected area

c. Absent breath sounds in the affected area A An increase in the anteroposterior to lateral ratio occurs in overinflated lungs as in emphysema. B Hyperresonance over the affected area occurs in overinflated lungs as in emphysema. C Absent breath sound in the affected area is anticipated because the fluid in the pleural space prevents breath sounds from being heard. D Increased vocal fremitus over the affected area is associated with consolidation that occurs with pneumonia or tumor. Fremitus is decreased to absent in pleural effusion.

A nurse suspects the patient has an infection of the maxillary sinuses. How can this suspicion be confirmed? a. Using a flashlight to illuminate the floor of the mouth b. Pressing gently with both thumbs into the eyebrow ridges c. Applying firm pressure with the thumbs below the cheekbones d. Standing behind the patient and asking him or her to slowly rotate the head

c. Applying firm pressure with the thumbs below the cheekbones A To transilluminate the maxillary sinuses, the nurse places the source of light lateral to the nose, just beneath the medial aspect of the eye, and looks through the patient's open mouth for illumination of the hard palate. B This palpates the frontal sinuses rather than the maxillary sinuses. C This palpates the maxillary sinuses to detect tenderness, which may indicate sinus congestion or infection. D This is not a correct technique to confirm infection of the maxillary sinuses.

Which disorder, if any, does a nurse screen for when examining a healthy adolescent? a. Muscle weakness b. Limited joint range of motion c. Curvature of the spine d. No screening is needed when the adolescent is healthy

c. Curvature of the spine A Screening for muscle weakness in a well adolescent is not indicated. B Screening for limited range of motion in a well adolescent is not indicated. C Adolescents are screened for scoliosis, kyphosis, and lordosis. Postural kyphosis is almost always accompanied by a compensatory lordosis, an abnormally concave lumbar curvature. D Adolescents are screened for scoliosis.

On inspection, a nurse finds the patient's anteroposterior diameter of the chest to be the same as the lateral diameter. Based on this finding, what additional data does the nurse anticipate? a. Increased vocal fremitus on palpation b. Dull tones heard on percussion c. Decreased breath sounds on auscultation d. Complaint of sharp chest pain on inspiration

c. Decreased breath sounds on auscultation A Increased fremitus occurs when the vibrations feel enhanced. This is found when lung tissues are congested or consolidated, which may occur in patients who have pneumonia or a tumor. B Dull tones may be heard in patients with pneumonia, pleural effusion, or atelectasis. C The equal anteroposterior and lateral diameters of the chest indicate air trapping from enlarged or destroyed alveoli. This air trapping causes decreased to absent breath sounds on auscultation. D Complaint of sharp chest pain on inspiration is pleuritic chest pain associated with pleural lining irritation and may occur in a patient with pleurisy or pneumonia.

Which techniques does a nurse use to palpate a patient's right kidney? a. Asks the patient to take a deep breath, elevates the patient's eleventh and twelfth ribs with the left hand, and deeply palpates for the right kidney with the right hand b. Asks the patient to exhale, elevates the patient's eleventh and twelfth ribs with the left hand, and deeply palpates for the right kidney with the right hand c. Asks the patient to take a deep breath, elevates the patient's right flank with the left hand, and deeply palpates for the right kidney with the right hand d. Asks the patient to exhale, elevates the patient's right flank with the left hand, and deeply palpates for the right kidney with the right hand

c. Asks the patient to take a deep breath, elevates the patient's right flank with the left hand, and deeply palpates for the right kidney with the right hand A Asking the patient to take a deep breath, elevating the patient's eleventh and twelfth ribs with the left hand, and deeply palpating for the right kidney with the right hand is incorrect because the flank is elevated rather than the ribs. B Asking the patient to exhale, elevating the patient's eleventh and twelfth ribs with the left hand, and deeply palpating for the right kidney with the right hand is incorrect because the flank is elevated rather than the ribs and the patient is asked to inhale rather than exhale. C Asking the patient to take a deep breath, elevating the patient's right flank with the left hand, and deeply palpating for the right kidney with the right hand is the correct technique. D Asking the patient to exhale, elevating the patient's right flank with the left hand, and deeply palpating for the right kidney with the right hand is incorrect because the patient is asked to inhale rather than exhale.

A patient complains of shortness of breath and having to sleep on three pillows to breathe comfortably at night. During the nurse's examination, what findings will suggest that the cause of this patient's dyspnea is due to heart disease rather than respiratory disease? a. Increased anteroposterior diameter b. Clubbing of the fingers c. Bilateral peripheral edema d. Increased tactile fremitus

c. Bilateral peripheral edema A This is seen with lung hyperinflation and may be associated with emphysema. B This is associated with chronic hypoxia and may be associated with cystic fibrosis or chronic obstructive pulmonary disease. C This indicates heart failure; dyspnea occurs because the heart cannot adequately perfuse the lungs. D This occurs when vibrations are enhanced and is associated with consolidation that may occur in pneumonia or tumor.

A nurse palpating the chest of a patient finds increased fremitus bilaterally. What is the significance of this finding? a. An expected finding b. Chronic obstructive pulmonary disease c. Bilateral pneumonia d. Bilateral pneumothorax

c. Bilateral pneumonia A An increase in fremitus from normal is not an expected finding. B Air trapping in chronic obstructive pulmonary disease causes a decreased fremitus. C Increased fremitus occurs when lung tissues are congested or consolidated, which may occur in patients who have pneumonia or a tumor. D Air in the pleural space causes a decreased fremitus.

During a Weber test, a patient with right ear hearing loss reports hearing sound longer in the right ear than the left ear. What results should the nurse expect to find from this patient during a Rinne test? a. Air conduction will be twice as long as bone conduction (2:1 ratio). b. Air conduction will be 1.5 times as long as bone conduction (1.5:1 ratio). c. Bone conduction will be longer than air conduction. d. Bone conduction will be equal to air conduction.

c. Bone conduction will be longer than air conduction. A This is an expected finding. B This finding is consistent with a sensorineural hearing loss, but this patient has a conduction hearing loss based on the results of the Weber test. C This finding from the Rinne test indicates a conduction hearing loss, which is consistent with the finding from the Weber test described in the question. D This finding is not consistent with the conductive hearing loss described.

A patient complains of right ear pain. What findings does the nurse anticipate on inspecting the patient's ears? a. Redness and edema of the pinna of the right ear b. Report of pain when the nurse manipulates the right ear c. Bulging and red tympanic membrane in the right ear d. Increased cerumen in the right ear canal

c. Bulging and red tympanic membrane in the right ear A Redness and edema of the pinna of the right ear is consistent with external ear pain that may be associated with otitis externa or swimmer's ear. B Report of pain when the nurse manipulates the right ear is consistent with external ear pain that may be associated with otitis externa or swimmer's ear. C Bulging and red tympanic membrane in the right ear is consistent with internal ear pain that may be associated with otitis media. D Increased cerumen in the right ear canal is not consistent with internal ear pain.

How does a nurse assess movements of the eyes? a. By assessing peripheral vision b. By noting the symmetry of the corneal light reflex c. By assessing the cardinal fields of gaze d. By performing the cover-uncover test

c. By assessing the cardinal fields of gaze A This tests the function of cranial nerve I (optic). B This indicates symmetry of eye muscles. C This tests the movement of the eye in all directions, which assesses the functions of the cranial nerves III (oculomotor), IV (abducens), and VI (trochlear). D This is performed after the corneal light reflex is abnormal, indicating asymmetric eye muscles.

A nurse inspects a patient's hands and notices clubbing of the fingers. The nurse correlates this finding with what condition? a. Pulmonary infection b. Trauma to the thorax c. Chronic hypoxemia d. Allergic reaction

c. Chronic hypoxemia A Pulmonary infection is acute and not associated with chronic hypoxia. B Trauma to the thorax is acute and not associated with chronic hypoxia. C Clubbing develops due to chronic hypoxemia, which occurs in chronic obstructive pulmonary disease. D Allergic reaction is acute and not associated with chronic hypoxia.

In reviewing the patient's record, the nurse notes that the patient has air in the subcutaneous tissue. The nurse validates that this patient has crepitus with which finding? a. Asymmetric expansion of the chest wall on inhalation b. Increased transmission of vocal vibrations on auscultation c. Crackling sensation under the skin of the chest on palpation d. Coarse grating sounds heard over the mediastinum on inspiration

c. Crackling sensation under the skin of the chest on palpation A Asymmetric chest expansion occurs with rib fracture or chest wall injury. B Increased vocal fremitus occurs with lung consolidation. C A crackling sensation is the finding when crepitus is present. D Coarse grating sounds heard over the mediastinum on inspiration does not validate crepitus.

A patient has a compression fracture of the cervical spine at C7 to C8 that is impairing deep tendon reflexes. Which response will the nurse expect from the affected deep tendon reflex? a. Diminished to absent pronation of the arm b. Diminished to absent flexion of the elbow c. Diminished to absent extension of the elbow d. Diminished to absent adduction of the upper arm

c. Diminished to absent extension of the elbow A Diminished to absent pronation of the arm is an abnormal response from the brachioradial deep tendon reflex that is innervated from C5 to C6. B Diminished to absent flexion of the elbow is an abnormal response from the biceps deep tendon reflex that is innervated from C5 to C6. C Diminished to absent extension of the elbow is an abnormal response from the triceps deep tendon reflex that is innervated from C6, C7, and C8. D Diminished to absent adduction of the upper arm is not a response of any deep tendon reflex.

On inspection of a female patient's abdomen, the nurse asks the patient to raise her head without using her arms and notes a midline bulge. What is the appropriate response of the nurse at this time? a. Ask the patient to cough to see if the bulge reappears. b. Auscultate the patient's abdomen for hypoactive bowel sounds. c. Document this as a normal finding and continue the examination. d. Perform light and deep palpation of the abdomen.

c. Document this as a normal finding and continue the examination. A Ask the patient to cough to see if the bulge reappears. A bulge that appears with coughing is an abnormal finding revealed by the increase in intrathoracic pressure during the cough. B Auscultating the patient's abdomen for hypoactive bowel sounds is not indicated because the bulge is a normal finding. C Document this as a normal finding and continue the examination. This is a normal finding on a patient D Performing light and deep palpation of the abdomen are not indicated because the bulge is a normal finding.

A nurse shines a light in the right pupil to test constriction and notices that the left pupil constricts as well. Based on these data, the nurse should take what action? a. Document this finding as an abnormal finding. b. Assess the patient for accommodation. c. Document this finding as a consensual reaction. d. Assess the patient's corneal light reflex.

c. Document this finding as a consensual reaction. d. Assess the patient's corneal light reflex. A This is a description of an expected finding—consensual reaction. B Accommodation is not assessed in response to consensual reaction; it tests the function of the oculomotor cranial nerve (CN III). C This is a description of expected consensual reaction. D This item describes a consensual reaction rather than a corneal light reflex.

A nurse is assessing a patient's peripheral circulation. Which finding indicates venous insufficiency of this patient's legs? a. Paresthesias and weak, thin peripheral pulses b. Leg pain that can be relieved by walking c. Edema that is worse at the end of the day d. Leg pain that increases when the legs are lowered

c. Edema that is worse at the end of the day A Paresthesias and weak, thin peripheral pulses are characteristics of arterial insufficiencies rather than venous. B Pain caused by arterial insufficiency gets worse by walking, because walking requires additional arterial blood. C Dependent edema is an indication of venous insufficiency. D Arterial pain is relieved by lowering the leg and aggravated by elevating the legs.

A nurse's presentation to patients on risk factors for oral cancer includes which fact? a. The peak incidence oral cancer is before 40 years of age. b. Women have a higher risk than men. c. Excessive alcohol consumption is a risk factor. d. Eating a low fiber diet is a risk factor.

c. Excessive alcohol consumption is a risk factor. A There is increased incidence after age 40 with peak incidence between ages 64 and 74. B There is a 2:1 men-to-women incidence. C Seventy-five to eighty percent of individuals who develop oral cancer consume excessive amounts of alcohol. D A low fiber diet increases the risk for colon cancer, but not oral cancer.

When assessing a patient with aortic valve stenosis, the nurse listens for which sound to detect a thrill? a. Sustained thrust of the heart against the chest wall during systole b. Visible sinking of the tissues between and around the ribs c. Fine, palpable vibration felt over the precordium d. Bounding pulse noted bilaterally

c. Fine, palpable vibration felt over the precordium A A sustained thrust of the heart against the chest wall during systole is a description of a lift. B A visible sinking of the tissues between and around the ribs is a description of a retraction. C A thrill is a palpable vibration over the precordium or artery. D A thrill feels like a palpable vibration rather than a bounding pulse.

While examining the genitalia of a 6-year-old girl, a nurse notices which finding as expected? a. Clear mucoid vaginal discharge b. Prepuce and clitoris are prominent c. Flat labia majora with thin labia minora d. Sparse pubic hair over the inner thighs

c. Flat labia majora with thin labia minora A Vaginal discharge is an abnormal finding at this age. B Normally at this age the clitoris is relatively small. C Until approximately age 7, the labia majora are flat, the labia minora are thin. D Evidence of pubic hair may be seen by the time the child reaches pubescence, usually between ages 8 and 11.

A nurse is having difficulty auscultating a patient's heart sounds because the lung sounds are too loud. What does the nurse ask the patient to do to improve hearing the heart sounds? a. Lie in a supine position. b. Cough. c. Hold his or her breath for a few seconds. d. Sit up and lean forward.

c. Hold his or her breath for a few seconds. A Lying in a supine position will not reduce the noise of breathing. B Coughing may clear some secretions, but when the lung sounds are so noisy that the heart sounds are difficult to hear, coughing is not sufficient to eliminate the noise from respirations. C Holding the breath for a few seconds eliminates the noise of breathing long enough to hear several cardiac cycles of heart sounds. The holding of the breath can be repeated if needed to hear the heart sounds again. D Sitting up and leaning forward brings the heart closer to the thoracic wall, but will not eliminate noise produced by the lungs.

A nurse assessing a 3-month-old infant suspects hydrocephalus based on which finding? a. Soft anterior fontanelle b. Lack of head control while sitting c. Increasing head circumference d. Marked asymmetry of the head

c. Increasing head circumference A This is an expected finding. Bulging fontanelle is a manifestation of hydrocephalus. B By 4 months old, most infants demonstrate head control by holding the head erect and midline when in an upright position. C The abnormal accumulation of cerebrospinal fluid increases the head circumference because the sutures of the skull are not ossified and the anterior fontanelle is not closed. D Marked asymmetry of the head may indicate craniosynostosis, a premature ossification of one or more of the cranial sutures.

How does a nurse collect baseline measurements of a 6-month-old infant? a. Measure the chest circumference around the lower ribs. b. Ask the parent how much the infant's weight has changed since birth. c. Measure the head just above the ears and eyebrows. d. Ask the parent to hold the infant while the nurse measures the length.

c. Measure the head just above the ears and eyebrows. A The nurse measures chest circumference, but the tape measure is placed around the nipples rather than the ribs. B An infant platform scale covered with a paper drape is used for weighing newborns, infants, and small children. C The nurse measures head circumference using this procedure until about 2 years old. D An infant's height is measured while the infant is lying supine.

The nurse documents which finding as normal after performing the Barlow-Ortolani maneuver on an infant? a. The clavicles are immobile and without crepitus. b. Each shoulder remains in a "hunched up" position. c. No clicking is noted when the hips are abducted and adducted. d. Both feet are held in the varus position when stroked on the soles.

c. No clicking is noted when the hips are abducted and adducted. A Normal findings include stable and smooth clavicles, without crepitus. B The Barlow-Ortolani maneuver assesses hip location. C The Barlow-Ortolani maneuver assesses hip location, and the movement should feel smooth and produce no clicking. D The feet should be flexible and not fixed. Normally, the hindfoot aligns with the lower leg and the forefoot turns inward slightly.

Which patient does the nurse identify as the one at greatest risk for hypertension? a. Woman with coronary artery disease b. Hispanic male c. Obese male with diabetes mellitus d. Postmenopausal woman

c. Obese male with diabetes mellitus A Although hypertension is a risk factor for coronary artery disease, coronary artery disease is not a risk factor for hypertension. B Although male gender is a risk factor, African-American men have a greater risk than Hispanic men. C Obese men with diabetes mellitus have three risk factors: obesity, gender, and comorbidity of diabetes mellitus. D Postmenopausal women do not have an increased risk for developing hypertension.

Which abnormal finding does the nurse anticipate finding on examination of this patient's abdomen? a. Decreased bowel sounds b. Bulge in the abdomen when coughing c. Palpable mass in the left lower quadrant d. Bruit around the umbilicus

c. Palpable mass in the left lower quadrant A Decreased bowel sounds are not expected if the patient is having bowel movements. B Bulge in the abdomen when coughing is a finding associated with abdominal or incisional hernias. C Palpable mass in the left lower quadrant is expected when interpreted with other data—age of the patient, intermittent cramping abdominal pain relieved by a bowel movement—as consistent with diverticular disease. D Bruit around the umbilicus is a finding associated with an abdominal aortic aneurysm.

When assessing an adult's liver, the nurse percusses the lower border and finds it to be 5 cm below the costal margin. What is the nurse's appropriate action at this time? a. Document this as an expected finding for this adult. b. Palpate the gallbladder for tenderness. c. Palpate the upper liver border on deep inspiration. d. Use the hooking technique to palpate the lower border of the liver.

c. Palpate the upper liver border on deep inspiration. A Documenting this as a normal finding for an adult patient is incorrect because this finding indicates an enlarged liver. B Palpating the gallbladder for tenderness is not indicated for an enlarged liver. C Palpating the upper border of the liver on deep inspiration is the correct technique to use when an enlarged liver is found (as indicated by the liver being percussed 5 cm below the costal margin). D Using the hooking technique to palpate the lower border of the liver is not needed because the liver is enlarged.

A nurse assesses neck range of movement of several adults. Which patient has an expected range of motion of the neck? a. Patient A is unable to resist the nurse's attempt to move the head upright. b. Patient B bends the head to the right and left (ear to shoulder) 15 degrees. c. Patient C flexes chin toward the chest 45 degrees. d. Patient D hyperextends the head 30 degrees from midline.

c. Patient C flexes chin toward the chest 45 degrees. A This finding is abnormal. B This finding is abnormal. The patient should be able to laterally bend the head 40 degrees from midline in each direction. C This is an expected finding. D The patient should be able to hyperextend the head 55 degrees from midline.

The nurse observes a patient rocking back and forth on the examination table in pain. Based on the patient's history, the nurse suspects kidney stones. What additional examination technique does the nurse perform to confirm this suspicion? a. Palpating the flank area for rebound tenderness b. Percussing the bladder for fullness c. Percussing the costal vertebral margins for tenderness d. Palpating McBurney point for tenderness

c. Percussing the costal vertebral margins for tenderness A Palpating the flank area for rebound tenderness is the correct location (flank area), but rebound tenderness is performed on the abdomen to detect peritoneal inflammation. B Percussing the bladder for fullness would provide data about bladder distention, but is not a technique to detect for kidney stones. C Percussing the costal vertebral margins for tenderness is the appropriate technique to detect kidney stones. The nurse recognizes the relationship between the history and the observation with further assessment techniques needed to confirm kidney stones. D Palpating McBurney point for tenderness is a technique to detect appendicitis.

A patient reports a history of snorting cocaine and is concerned about his bloody nasal drainage. What does the nurse expect to see on inspection of his nose? a. Deviated septum b. Pale turbinates c. Perforated nasal septum d. Localized erythema and edema

c. Perforated nasal septum A Deviated septum may be from birth or trauma to the nose, but not from cocaine use. B Pale turbinates are an indication of allergies. C Perforated nasal septum develops from cocaine use. D Localized erythema and edema are nonspecific and indicate inflammation somewhere in the nose.

A nurse determines that a patient's jugular venous pressure is 3.5 inches. What additional data does the nurse expect to find? a. Weight loss b. Tented skin turgor c. Peripheral edema d. Capillary refill greater than 5 seconds

c. Peripheral edema A Weight loss occurs with loss of fluid rather than fluid overload. B Tented skin turgor occurs with fluid loss rather than fluid overload. C The pressure should not rise more than 1 inch (2.5 cm) above the sternal angle. A pressure of 3.5 inches indicates fluid volume excess, which causes peripheral edema due to excessive fluid in blood vessels. D Capillary refill greater than 5 seconds occurs with arterial insufficiency rather than fluid overload.

During an eye assessment, a nurse asks the patient to cover one eye with a card as the nurse covers his or her eye directly opposite the patient's covered eye. The nurse moves an object into the field of vision and asks the patient to tell when the object can be seen. This assessment technique collects what data about the patient's eyes? a. Symmetry of extraocular muscles b. Visual acuity in the uncovered eye c. Peripheral vision of the uncovered eye d. Consensual reaction of the uncovered eye

c. Peripheral vision of the uncovered eye A Symmetry is tested by the corneal light reflex. B Visual acuity is tested using the Snellen chart. C This describes the confrontation test, which assesses peripheral vision. D Consensual reaction is tested by noticing the pupillary constriction of one eye when a light is being shown into the other eye.

A nurse is assessing for vocal (tactile) fremitus on a patient with pulmonary edema. Which is the appropriate technique to use? a. Systematically percuss the posterior chest wall following the same pattern that is used for auscultation and listen for a change in tone from resonant to dull. b. Place the pads of the fingers on the right and left thoraces and palpate the texture and consistency of the skin feeling for a crackly sensation under the fingers. c. Place the palms of the hands on the right and left thoraces, ask the patient to say "99," and feel for vibrations. d. Place both thumbs on either side of the patient's spinal processes, extend fingers laterally, ask the patient to take a deep breath, and feel for vibrations.

c. Place the palms of the hands on the right and left thoraces, ask the patient to say "99," and feel for vibrations. A This is the technique for percussing the thorax for tones. B This is the technique for detecting crepitus. C This is the correct technique for vocal fremitus. D This is not the correct technique.

How does a nurse assess the head circumference of an infant? a. Places a ruler behind the infant's head, noting the head width. b. Uses a plastic headband placed around the infant's head from crown to chin. c. Places a measuring tape around the head above the eyebrows and occipital prominence. d. Uses a measuring tape to find the distance between the ears and eyes and between the eyes and occiput.

c. Places a measuring tape around the head above the eyebrows and occipital prominence. A This is an incorrect technique. B This is an incorrect technique. C This is the correct technique for measuring head circumference. D This is an incorrect technique.

Which pulse may be a challenge for a nurse to palpate? a. Temporal b. Femoral c. Popliteal d. Dorsalis pedis

c. Popliteal A The temporal pulse is palpated over the temporal bone on each side of the head. B For the femoral pulse, palpate below the inguinal ligament, midway between the symphysis pubis and anterior superior iliac. C For the popliteal pulse, palpate the popliteal artery behind the knee in the popliteal fossa to assess perfusion. This pulse may be difficult to find. D For the dorsalis pedis pulse, palpate on the inner aspect of the ankle below and slightly behind the medial malleolus (ankle bone).

How does a nurse document a large, flat bluish capillary area on a neonate's cheek? a. Mongolian spot b. Stork bite (telangiectasis) c. Port-wine stain (nevus flammeus) d. Strawberry hemangioma

c. Port-wine stain (nevus flammeus) A A Mongolian spot appears as an irregularly shaped, darkened, flat area over the sacrum and buttocks. B A stork bite (telangiectasis) is a common vascular birthmark that appears as a small red or pink spot often seen on the back of the neck. C Port-wine stains appear as large, flat, bluish purple capillary areas. They are frequently found on the face along the distribution of the fifth cranial nerve (trigeminal). D A strawberry hemangioma appears as a slightly raised, reddened area with a sharp demarcation line that may be 2 to 3 cm in diameter.

What technique does a nurse use when palpating the right lobe of a patient's thyroid gland using the posterior approach? a. Pushes the cricoid process to the left with the right thumb and feels the right lobe with the left hand b. Uses the left hand to push the sternocleidomastoid muscle to the right and feels the lobe with the right hand c. Pushes the trachea to the right with the left hand and feels the right lobe with the right hand d. Places the fingers on either side of the trachea above the cricoid cartilage and feels the right lobe

c. Pushes the trachea to the right with the left hand and feels the right lobe with the right hand A This description is not a correct technique. In the posterior approach, the right lobe is felt with the right hand. B This description is not a correct technique. The trachea, not the muscle, is moved to the side. C This is the correct technique. D This description is not a correct technique. The fingers are placed below the cricoid cartilage.

A patient tells the nurse at the clinic, "I can never seem to get warm lately and feel tired all the time." The nurse records these data under which section of the health history? a. Past health history b. Present health status c. Reason for seeking care (chief complaint) d. Subjective assessment data

c. Reason for seeking care (chief complaint) A The past health history includes data about immunizations, surgeries, accidents, and childhood illnesses. B The present health status includes data the nurse obtains from the patient, often using a symptom analysis in which more data are collected about the patient's reason for seeking care. C The reason for seeking care (chief complaint) is the patient's reason for seeking care (also called the presenting problem). The patient's reason for seeking care is often recorded as a direct quote. D Subjective assessment data include information from the patient. In this example, the patient expresses the reason for seeking care, which is directly quoted and placed in quotation marks in the chief complaint section of the data sheet so that the patient's reason for seeking care can be easily identified.

The nurse places an 8-year-old boy in which position for examination of his genitalia? a. Supine with legs extended to either side b. Lying on his left side with knees bent c. Reclining with knees flexed d. Standing with legs spread apart

c. Reclining with knees flexed A The examination is easiest to perform if the child is sitting. B The examination is easiest to perform if the child is sitting. C The examination is easiest to perform if the child is sitting in either a slightly reclining position with his knees flexed or heels near the buttock or sitting with his knees spread and ankles crossed. D The examination is easiest to perform if the child is sitting.

A nurse reads in the history that a patient has a new onset of acute otitis media. Based on this information, how does the nurse expect this patient's tympanic membrane to appear? a. Dull b. Shiny c. Red d. Blue to deep red

c. Red A This indicates fibrosis or scarring. B This is normal for the tympanic membrane. C This indicates infection in the middle ear, such as otitis media. D This indicates blood behind the tympanic membrane, which may have occurred secondary to injury.

When auscultating a patient's abdomen using the bell of the stethoscope, the nurse hears soft, low-pitched murmurs over the right and left upper midline. What do these sounds indicate? a. Expected peristalsis b. Femoral artery stenosis c. Renal artery stenosis d. Hyperactive bowel sounds

c. Renal artery stenosis A Expected peristalsis would be heard using the diaphragm of the stethoscope and would be a gurgling sound. B Femoral artery stenosis is a vascular sound heard with the bell, but located in the lower abdomen. C Renal artery stenosis is a vascular sound heard with the bell and located in the upper abdomen. D Hyperactive bowel sounds would be heard using the diaphragm and would be present in all quadrants.

What changes in using the ophthalmoscope should the nurse need to make when inspecting the eye of a patient who is nearsighted? a. Holding the ophthalmoscope in the right hand when inspecting the patient's right eye b. Using the grid light of the lens aperture to visualize the internal structures of the eye c. Rotating the diopter to the red (minus) numbers d. Asking the patient to look directly into the ophthalmoscope light

c. Rotating the diopter to the red (minus) numbers A This procedure is performed with all patients having an internal eye examination. B The grid is used to estimate the size of lesions. C This compensates for the longer eyeball of a myopic patient. D This is an instruction given to the patient to visualize the macula.

The nurse asks the patient to stand with feet together, arms resting at the sides, with eyes open and then with the eyes closed. Which response by the patient indicates a problem in the cerebellum? a. Maintains balance when eyes are open, but loses balance with eyes closed b. Is unable to stand upright after turning around in a circle once c. Steps sideways when standing with feet together and eyes closed d. Sways slightly and maintains upright posture with feet together

c. Steps sideways when standing with feet together and eyes closed A This is an abnormal response, but is indicative of a proprioceptive problem rather than a cerebellar problem. B Losing balance is an abnormal response, but turning in a circle is not a part of the Romberg test. C This is an abnormal response documented as a "positive Romberg" and indicates cerebellar dysfunction. D This is an expected response documented as a "negative Romberg," indicating appropriate cerebellar function for balance.

When assessing an infant, the nurse recognizes which finding requires immediate attention? a. Cheyne-Stokes type of respiratory pattern b. 1:1 anteroposterior to lateral chest diameter c. Stridor and nasal flaring d. Bronchovesicular lung sounds in the periphery

c. Stridor and nasal flaring A The respiratory pattern in the newborn may be irregular, having a Cheyne-Stokes type of pattern. B Unlike the adult, the infant has a round thorax with an equal anteroposterior and lateral diameter. C Stridor and nasal flaring warrants immediate medical attention. Stridor is a high-pitched, piercing sound that is primarily heard in a distressed infant during inspiration. D The predominant breath sound heard in the peripheral lung fields of infants is bronchovesicular.

A nurse is teaching a patient how to manage chronic obstructive pulmonary disease (COPD). This intervention is an example of which level of health promotion? a. Primary prevention b. Secondary prevention c. Tertiary prevention d. Risk factor prevention

c. Tertiary prevention A The focus of primary prevention is to prevent a disease from developing by promoting a healthy lifestyle. B Secondary prevention consists of efforts to promote early detection of disease. C Teaching a patient how to live with a chronic disease is an example of tertiary prevention directed toward minimizing the disability from chronic disease and helping the patient maximize his or her health. D Risk factor prevention is part of primary prevention that focuses on preventing disease by managing risk factors.

What finding does a nurse expect when assessing a one-month old's eyes and vision? a. The newborn distinguishes most colors b. Tears when the newborn cries c. The newborn following a bright toy or light d. The newborn's blink reflex is present

c. The newborn following a bright toy or light A Testing for color vision occurs between ages 4 and 8 years. B There are no tears until about 2 to 3 months of age. C This is an accurate statement. D The blink reflex is present in normal newborns and infants before one month.

The nurse hears in a report that a patient has receptive or fluent aphasia. What communication abilities does the nurse anticipate from this patient? a. The patient understands speech but is unable to translate ideas into meaningful speech. b. The patient is able to understand speech but has difficulty forming words creating muffled speech. c. The patient is unable to comprehend speech and thus does not respond verbally. d. The patient is emotionally liable and cries easily, which interferes with the ability to communicate.

c. The patient is unable to comprehend speech and thus does not respond verbally. A The inability to translate ideas into meaningful speech or writing is termed expressive aphasia or nonfluent aphasia and is associated with lesions in the Broca area in the frontal lobe. B This speech pattern is more consistent with patients who have involvement of muscles of speech rather than neurologic deficits. C This deficit is called receptive aphasia or fluent aphasia and is associated with lesions in the Wernicke area in the temporal lobe. D This speech pattern is not relevant to this patient.

For which person is a comprehensive assessment indicated? a. The person who had abdominal surgery yesterday b. The person who is unaware of his high serum glucose levels c. The person who is being admitted to a long-term care facility d. The person who is beginning rehabilitation after a knee replacement

c. The person who is being admitted to a long-term care facility A A shift assessment is most appropriate for the person who is recovering in the hospital from surgery. B A screening assessment is performed for the purpose of disease detection, in this case diabetes mellitus. C A comprehensive assessment is performed during admission to a facility to obtain a detailed history and complete physical examination. D An episodic or follow-up assessment is performed after knee replacement to evaluate the outcome of the procedure.

A patient complains of sore throat, pain with swallowing, fever, and chills. The nurse suspects tonsillitis and plans to palpate the anterior cervical lymph nodes. Where does the nurse place his fingers to palpate these nodes? a. In front of the ears b. Under the mandibles c. Along the angle of the mandibles d. Adjacent to the sternocleidomastoid muscles

d. Adjacent to the sternocleidomastoid muscles A This is the location of the preauricular lymph nodes. B This is the location of the submental and submandibular lymph nodes. C This is the location of the parotid lymph nodes. D This is the location of the anterior cervical lymph nodes.

Which patient behavior indicates to the nurse that the patient's facial cranial nerve (CN VII) is intact? a. The patient's eyes move to the left, right, up, down, and obliquely. b. The patient moistens the lips with the tongue. c. The sides of the mouth are symmetric when the patient smiles. d. The patient's eyelids blink periodically.

c. The sides of the mouth are symmetric when the patient smiles. A This finding represents movement of the extraocular muscles, which are controlled by the oculomotor, trochlear, and abducens cranial nerves (CN III, IV, and VI, respectively). B This finding represents movement of the tongue, which is controlled by the hypoglossal cranial nerve (CN XII). C This finding represents facial symmetry, which is controlled by the facial cranial nerve (CN VII). D This finding represents function of the oculomotor cranial nerve (CN III).

While taking a history, the nurse observes that the patient's facial cranial nerves (CN VII) are intact based on which behaviors of the patient? a. The patient's eyes move to the left, right, up, down, and obliquely during conversation. b. The patient moistens the lips with the tongue. c. The sides of the mouth are symmetric when the patient smiles. d. The patient's eyelids blink periodically

c. The sides of the mouth are symmetric when the patient smiles. A This represents movement of the extraocular muscles, which are controlled by the oculomotor, trochlear, and abducens cranial nerves (CN III, IV, and VI, respectively). B This represents movement of the tongue, which is controlled by the hypoglossal cranial nerve (CN XII). C This represents facial symmetry, which is controlled by the facial cranial nerve (CN VII). D This represents function of the oculomotor cranial nerve (CN III).

To correctly percuss the abdomen, a nurse places the distal aspect of the middle finger of the nondominant hand against the skin of the abdomen, and the other fingers are spread apart and slightly lifted off the skin. How does the nurse use the fingers of the dominant hand? a. The pad of the middle finger strikes the distal interphalangeal joint of the middle finger touching the skin of the abdomen. b. The tip of the middle finger strikes the nail of the middle finger touching the skin of the abdomen. c. The tip of the middle finger strikes the distal interphalangeal joint of the middle finger touching the skin of the abdomen. d. The pads of the index and middle fingers strike the nail of the middle finger touching the skin of the abdomen.

c. The tip of the middle finger strikes the distal interphalangeal joint of the middle finger touching the skin of the abdomen. A This description is incorrect because the tip of the finger is used rather than the pad. B This description is incorrect because the distal joint is struck rather than the nail. C This is the correct technique. D This description is incorrect because the tip of the middle finger strikes the distal joint.

After two separate office visits, the nurse suspects that a patient is developing Stage 1 hypertension based on which consecutive blood pressure readings? a. Visit 1, 118/78; Visit 2, 116/76 b. Visit 1, 130/88; Visit 2, 134/88 c. Visit 1, 144/92; Visit 2, 150/90 d. Visit 1, 162/100; Visit 2, 166/104

c. Visit 1, 144/92; Visit 2, 150/90 A These readings are within normal limits. B These readings are prehypertension because the systolic pressures are 120 to 139 and diastolic pressures are greater than 80 mm Hg. C These readings are stage 1 because the systolic pressures are 140 to 159 and diastolic pressures are 90 mm Hg or greater. D These readings are stage 2 because the systolic pressures are greater than 160 and diastolic pressures are 100 mm Hg or greater.

A nurse uses which technique to assess a patient's peripheral vision? a. The nurse asks the patient to keep the head still and by moving the eyes only, follow the nurse's finger as it moves side to side, up and down, and obliquely. b. The nurse covers one of the patient's eyes with a card and observes the uncovered eye for movement, then removes the card and observes the just uncovered eye for movement. c. With the patient and nurse facing each other and a card covering their corresponding eyes, the nurse moves an object into the visual field and the patient reports when the object is seen. d. The nurse shines a light on both corneas at the same time and notes the location of the reflection in each eye.

c. With the patient and nurse facing each other and a card covering their corresponding eyes, the nurse moves an object into the visual field and the patient reports when the object is seen. A This technique tests extraocular muscle symmetry. B This cover-uncover technique is performed when the corneal light reflex is asymmetric. C This is the confrontation test that tests peripheral vision. D This describes the corneal light reflex that tests the symmetry of the eye muscles.

Having misplaced a stethoscope, a nurse borrows a colleague's stethoscope. The nurse next enters the patient's room and identifies self, washes hands with soap, and states the purpose of the visit. The nurse performs proper identification of the patient before auscultating the patient's lungs. Which critical health assessment step should the nurse have performed? a. Running warm water over stethoscope b. Draping stethoscope around the neck c. Rubbing stethoscope with betadine d. Cleaning stethoscope with alcohol

d. Cleaning stethoscope with alcohol Bacteria and viruses can be transferred from patient to patient when a stethoscope that is not clean is used. The stethoscope should be cleaned before use on each patient with isopropyl alcohol. Running water over the stethoscope does not kill bacteria. Betadine is an inappropriate cleaning solution and may damage the equipment. Draping the stethoscope around the neck is not advised.

During a history, a nurse notices a patient is short of breath, is using pursed-lip breathing, and maintains a tripod position. Based on these data, what abnormal finding should the nurse expect to find during the examination? a. Increased tactile fremitus b. Inspiratory and expiratory wheezing c. Tracheal deviation d. An increased anteroposterior diameter

d. An increased anteroposterior diameter A Increased tactile fremitus occurs when vibrations are enhanced and is associated with consolidation that may occur in pneumonia or tumor. B Inspiratory and expiratory wheezing is associated with asthma. C Tracheal deviation is associated with tension pneumothorax. D An increased anteroposterior diameter is consistent with emphysema.

A school nurse recognizes a belt buckle-shaped ecchymosis on a 7-year-old student. When privately asked about how the injury occurred, the student described falling on the playground. Which action will the nurse take next? a. Talk to the principal about how to proceed. b. Disregard the finding based upon child's response. c. Interview the patient in the presence of the teacher. d. Contact social services and report suspected abuse.

d. Contact social services and report suspected abuse. Most states mandate a report to a social service center if nurses suspect abuse or neglect. When abuse is suspected, the nurse interviews the patient in private, not with a teacher. Observe the behavior of the individual for any signs of frustration, explanations that do not fit his or her physical presentation, or signs of injury. The nurse knows how to proceed and does not need to talk to the principal about what to do. Disregarding the finding is not advised because victims often will not complain or report that they are in an abusive situation.

A nurse is teaching a patient's family member about permanent tube feedings at home. Which purpose of patient education is the nurse meeting? a. Health promotion b. Illness prevention c. Restoration of health d. Coping with impaired functions

d. Coping with impaired functions Teach family members to help the patient with health care management (e.g., giving medications through gastric tubes and doing passive range-of-motion exercises) when coping with impaired functions. Not all patients fully recover from illness or injury. Many have to learn to cope with permanent health alterations. Health promotion involves healthy people staying healthy, while illness prevention is prevention of diseases. Restoration of health occurs if the teaching is about a temporary tube feeding, not a permanent tube feeding.

A nurse is teaching a patient with a risk for hypertension how to take a blood pressure. Which action by the nurse is the priority? a. Assess laboratory results for high cholesterol and other data. b. Identify that teaching is the same as the nursing process. c. Perform nursing care therapies to address hypertension. d. Focus on a patient's learning needs and objectives.

d. Focus on a patient's learning needs and objectives. The teaching process focuses on the patient's learning needs, motivation, and ability to learn; writing learning objectives and goals is also included. Nursing and teaching processes are not the same. Assessing laboratory results for high cholesterol and performing nursing care therapies are all components of the nursing process, not the teaching process.

During a routine pediatric history and physical, the parents report that their child was a very small, premature infant that had to stay in the neonatal intensive care unit longer than usual. They state that the infant was yellow when born and developed an infection that required "every antibiotic under the sun" to reach a cure. Which exam is a priority for the nurse to conduct on the child? a. Cardiac b. Respiratory c. Ophthalmic d. Hearing acuity

d. Hearing acuity Hearing is the priority. Risk factors for hearing problems include low birth weight, nonbacterial intrauterine infection, and excessively high bilirubin levels. Hearing loss due to ototoxicity (injury to auditory nerves) can result from high maintenance doses of antibiotics. Cardiac, respiratory, and eye examinations are important assessments but are not relevant to this child's condition.

A nurse is teaching a patient who has low health literacy about chronic obstructive pulmonary disease (COPD) while giving COPD medications. Which technique is most appropriate for the nurse to use? a. Use complex analogies to describe COPD. b. Ask for feedback to assess understanding of COPD at the end of the session. c. Offer pamphlets about COPD written at the eighth grade level with large type. d. Include the most important information on COPD at the beginning of the session.

d. Include the most important information on COPD at the beginning of the session. Include the most important information at the beginning of the session for patients with literacy or learning disabilities. Also, use visual cues and simple, not complex, analogies when appropriate. Another technique is to frequently ask patients for feedback to determine whether they comprehend the information. Additionally, provide teaching materials that reflect the reading level of the patient, with attention given to short words and sentences, large type, and simple format (generally, information written on a fifth grade reading level is recommended for adult learners).

The nurse is urgently called to the gymnasium regarding an injured student. The student is crying in severe pain with a malformed fractured lower leg. Which proper sequence will the nurse follow to perform the initial assessment? a. Light palpation, deep palpation, and inspection b. Inspection, light palpation, and deep palpation c. Auscultation and light palpation d. Inspection and light palpation

d. Inspection and light palpation Inspection is the use of vision and hearing to distinguish normal from abnormal findings. Light palpation determines areas of tenderness and skin temperature, moisture, and texture. Deep palpation is used to examine the condition of organs, such as those in the abdomen. Caution is the rule with deep palpation. Deep palpation is performed after light palpation; however, deep palpation is not performed on a fractured leg. Auscultation is used to evaluate sound and is not used to assess a fractured leg.

A patient in the emergency department is reporting left lower abdominal pain. Which proper order will the nurse follow to perform the comprehensive abdominal examination? a. Percussion, palpation, auscultation b. Percussion, auscultation, palpation c. Inspection, palpation, auscultation d. Inspection, auscultation, palpation

d. Inspection, auscultation, palpation The order of an abdominal examination differs slightly from that of other assessments. Begin with inspection and follow with auscultation. By using auscultation before palpation, the chance of altering the frequency and character of bowel sounds is lessened.

A nurse is teaching a group of healthy adults about the benefits of flu immunizations. Which type of patient education is the nurse providing? a. Health analogies b. Restoration of health c. Coping with impaired functions d. Promotion of health and illness prevention

d. Promotion of health and illness prevention As a nurse, you are a visible, competent resource for patients who want to improve their physical and psychological well-being. In the school, home, clinic, or workplace, you promote health and prevent illness by providing information and skills that enable patients to assume healthier behaviors. Injured and ill patients need information and skills to help them regain or maintain their level of health; this is referred to as restoration of health. Not all patients fully recover from illness or injury. Many have to learn to cope with permanent health alterations; this is known as coping with impaired functions. Analogies supplement verbal instruction with familiar images that make complex information more real and understandable. For example, when explaining arterial blood pressure, use an analogy of the flow of water through a hose.

A nurse's goal is to provide teaching for restoration of health. Which situation indicates the nurse is meeting this goal? a. Teaching a family member to provide passive range of motion for a stroke patient b. Teaching a woman who recently had a hysterectomy about possible adoption c. Teaching expectant parents about changes in childbearing women d. Teaching a teenager with a broken leg how to use crutches

d. Teaching a teenager with a broken leg how to use crutches Injured or ill patients need information and skills to help them regain or maintain their levels of health. An example includes teaching a teenager with a broken leg how to use crutches. Not all patients fully recover from illness or injury. Many have to learn to cope with permanent health alterations. New knowledge and skills are often necessary for patients and/or family members to continue activities of daily living. Teaching family members to help the patient with health care management (e.g., giving medications through gastric tubes, doing passive range-of-motion exercises) is an example of coping with long-term impaired functions. For a woman with a hysterectomy, teaching about adoption is not restoration of health; restoration of health in this situation would involve activity restrictions and incision care if needed. In childbearing classes, you teach expectant parents about physical and psychological changes in the woman and about fetal development; this is part of health maintenance.

Upon assessment, the patient is breathing normally and has normal vesicular lung sounds. Which expected inspiratory-to-expiratory breath sounds will the nurse hear? a. The expiration phase is longer than the inspiration phase. b. The inspiratory phase lasts exactly as long as the expiratory phase. c. The expiration phase is 2 times longer than the inspiration phase. d. The inspiratory phase is 3 times longer than the expiratory phase.

d. The inspiratory phase is 3 times longer than the expiratory phase. Vesicular breath sounds are normal breath sounds; the inspiratory phase is 3 times longer than the expiratory phase. Bronchovesicular breath sounds have an inspiratory phase equal to the expiratory phase. Bronchial breath sounds have an expiration phase longer than the inspiration phase at a 3:2 ratio.

A nurse teaches a patient with heart failure healthy food choices. The patient states that eating yogurt is better than eating cake. Which element represents feedback? a. The nurse b. The patient c. The nurse teaching about healthy food choices d. The patient stating that eating yogurt is better than eating cake

d. The patient stating that eating yogurt is better than eating cake Feedback needs to demonstrate the success of the learner in achieving objectives (i.e., the learner verbalizes information or provides a return demonstration of skills learned). The nurse is the sender. The patient (learner) is the receiver. The teaching is the message.

Which learning objective/outcome has the highest priority for a patient with life-threatening, severe food allergies that require an EpiPen (epinephrine)? a. The patient will identify the main ingredients in several foods. b. The patient will list the side effects of epinephrine. c. The patient will learn about food labels. d. The patient will administer epinephrine.

d. The patient will administer epinephrine. Once you assist in meeting patient needs related to basic survival (how to give epinephrine), you can discuss other topics, such as nutritional needs and side effects of medications. For example, a patient recently diagnosed with coronary artery disease has deficient knowledge related to the illness and its implications. The patient benefits most by first learning about the correct way to take nitroglycerin and how long to wait before calling for help when chest pain occurs. Thus, in this situation, the patient benefits most by first learning about the correct way to take epinephrine. "The patient will learn about food labels" is not objective and measurable and is not correctly written.

A head and neck physical examination is completed on a 50-year-old female patient. All physical findings are normal except for fine brittle hair. Which laboratory test will the nurse expect to be ordered, based upon the physical findings? a. Oxygen saturation b. Liver function test c. Carbon monoxide d. Thyroid-stimulating hormone test

d. Thyroid-stimulating hormone test Thyroid disease can make hair thin and brittle. Liver function testing is indicated for a patient who has jaundice. Oxygen saturation will be used for cyanosis. Cherry-colored lips indicate carbon monoxide poisoning.

When describing patient education approaches, the nurse educator would explain that informal teaching is an approach that a. addresses group needs. b. follows formalized plans. c. has standardized content. d. often occurs one-to-one.

d. often occurs one-to-one. Informal teaching is individualized one-on-one teaching which represents the majority of patient education done by nurses that occurs when an intervention is explained or a question is answered. Group needs are often the focus of formal patient education courses or classes. Informal teaching does not necessarily follow a specific formalized plan. It may be planned with specific content, but it is individualized responses to patient needs. Formal teaching involves the use of a curriculum/course plan with standardized content.

The nurse educator would identify a need for further teaching when the student lists the types of learning as a. affective. b. cognitive. c. psychomotor. d. self-directed.

d. self-directed. Self-directed is one approach to learning but is not considered a type or domain of learning. Self-directed would be a cognitive way of learning. Affective (feelings/attitude), cognitive (knowledge), and psychomotor (skills/performance) are the main domains of learning.

Interrelated concepts to the professional nursing role a nurse manager would consider when addressing concerns about the quality of patient education include a. adherence. b. developmental level. c. motivation. d. technology.

d. technology. The interrelated concepts to the professional role of a nurse include health promotion, leadership, technology/informatics, quality, collaboration, and communication. Adherence, culture, developmental level, family dynamics, and motivation are considered interrelated concepts to patient attributes and preference.

On inspection of the external eye structures of an African American patient, the nurse notices the sclerae are not white, but appear a darker shade with tiny black dots of pigmentation near the limbus. How does the nurse document this finding? a. As an indication of a type of anemia b. As a hordeolum or sty c. As jaundice d. As an expected racial variation

d. As an expected racial variation A This may cause a pale conjunctiva. B This is an acute infection originating in the sebaceous gland of the eyelid. C Jaundice is a yellow color of the sclera associated with liver or gallbladder disease. D This as an expected racial variation.

A male patient tells the nurse that he rarely sleeps more than 4 hours a night and has not experienced any problems because of the lack of sleep. Which response by the nurse is most appropriate? a. "That is interesting." b. "Only 4 hours of sleep? How do you stay awake during the day?" c. "Really? Everyone needs more sleep than that." d. "Did I understand that you sleep 4 hours every night?"

d. "Did I understand that you sleep 4 hours every night?" A "That is interesting" does not provide an opportunity for the patient to explain any reason for the number of hours of sleep. B "Only 4 hours of sleep? How do you stay awake during the day?" questions the accuracy of the patient's data and may not encourage the patient to give further details. C "Really? Everyone needs more sleep than that" can be perceived as argumentative, but does not encourage further data from the patient. D "Did I understand that you sleep 4 hours every night?" is a reflection technique that allows the nurse to confirm and obtain additional information.

A patient reports having frequent heartburn. Which question does the nurse ask in response to this information? a. "Has your abdomen been distended when you feel the heartburn?" b. "What have you eaten in the last 24 hours?" c. "Is there a history of heart disease in your family?" d. "How long after eating do you have heartburn?"

d. "How long after eating do you have heartburn?" A The question "Has your abdomen been distended when you feel the heartburn?" is not related to the heartburn. Distention usually is related to intestinal obstruction or liver disease. B The question "What have you eaten in the last 24 hours?" relates more to gastroenteritis. Indigestion is usually caused by food eaten in the last meal rather than in the last 24 hours. C The question "Is there a history of heart disease in your family?" points to myocardial ischemia. Although heartburn may be a symptom of myocardial ischemia, asking the patient about the family history is not relevant in this case. D Asking "How long after eating do you have heartburn?" can aid in determining if the patient has gastroesophageal reflux disease or a hiatal hernia. Both are common disorders that cause indigestion a few hours after meals.

A patient answers questions quietly and appears sad. While answering questions about her marriage, she begins to cry. Which response by the nurse is appropriate in this situation? a. "Don't cry! I'll come back when you've settled down." b. "I only have a few more questions to ask, and then I'll leave you alone for a while." c. "Everyone has ups and downs in their marriage. What problems are you having?" d. "I see that you are upset. Is there something you'd like to discuss?"

d. "I see that you are upset. Is there something you'd like to discuss?" A "Don't cry! I'll come back when you've settled down" is not a therapeutic response. The nurse needs to support the patient rather than leave her. B "I only have a few more questions to ask, and then I'll leave you alone for a while" is not a therapeutic response. The nurse is more concerned about getting the history than the patient's response. C "Everyone has ups and downs in their marriage. What problems are you having?" is not a therapeutic response. The nurse is assuming there are problems in the marriage instead of collecting more data. D "I see that you are upset. Is there something you'd like to discuss?" shows that the nurse is attentive to the patient's feelings and does not make assumptions about the reason why the patient is crying. The crying may signify additional data the nurse needs to collect during this interview.

A nurse suspects a female patient is a victim of physical abuse. Which response is most likely to encourage the patient to confide in the nurse? a. "You've got a huge bruise on your face. Did your husband hit you?" b. "That bruise looks tender. I don't know how people can do that to one another." c. "If your boyfriend hit you, you can get a restraining order against him." d. "I've seen women who have been hurt by boyfriends or husbands. Does anyone hit you?"

d. "I've seen women who have been hurt by boyfriends or husbands. Does anyone hit you?" A "You've got a huge bruise on your face. Did your husband hit you?" assumes that domestic violence did occur, and the comment does not encourage the patient to divulge additional information. B "That bruise looks tender. I don't know how people can do that to one another" assumes that domestic violence did occur, and the comment does not encourage the patient to divulge additional information. C "If your boyfriend has hit you, you can get a restraining order against him" assumes that domestic violence did occur, and the comment does not encourage the patient to divulge additional information. D "I've seen women who have been hurt by boyfriends or husbands" is an example of a technique referred to as "permission giving" in which the nurse communicates that it is safe to discuss uncomfortable topics.

The nurse is taking a health history on a patient who reports frequent stabbing headaches occurring once a day lasting about an hour. Which statement by the patient is most indicative of cluster headaches? a. "I usually have nausea and vomiting with my headaches." b. "My whole head is constantly throbbing." c. "It feels like my head is in a vice." d. "The pain is on the left side over my eye, forehead, and cheek."

d. "The pain is on the left side over my eye, forehead, and cheek." A This is descriptive of migraines rather than cluster headaches. B This is descriptive of migraines rather than cluster headaches. C This is descriptive of tension rather than cluster headaches. D This description is consistent with cluster headaches.

An American Indian mother expresses concern about an irregularly shaped, dark area over her neonate's sacrum and buttocks. What is the nurse's most appropriate response to this mother? a. "This area will continue to grow until the infant is 10 to 15 months old." b. "This is a birth mark, which usually disappears by age 5 years." c. "This skin abnormality will require follow-up care." d. "This is a birth mark and they usually disappear by age 1 or 2 years."

d. "This is a birth mark and they usually disappear by age 1 or 2 years." A This description refers to cavernous hemangioma that requires frequent reassessment. B This description refers to a "stork bite" (telangiectasis). C This is an inappropriate response. D This description refers to a Mongolian spot.

A mother who sees her newborn just after vaginal delivery is distraught because the child's head is elongated. Which response is most appropriate by the nurse? a. "This is due to a small area of bleeding that will go away in 1 to 2 months." b. "This may indicate a congenital deformity; the pediatrician will evaluate this." c. "This will require surgery to prevent hydrocephalus from developing." d. "This is not unusual after a vaginal delivery and will go away in about a week."

d. "This is not unusual after a vaginal delivery and will go away in about a week." A This response refers to cephalohematoma, which is not described here. B The newborn has molding that will resolve in a week. C The newborn has molding that will resolve in a week. D This is molding, which occurs when cranial bones override each other. Molding is secondary to the head passing through the birth canal and generally lasts less than a week.

The patient reports having a persistent cough for the past 2 weeks and that the cough disrupts sleep and has not been helped by over-the-counter cough medicines. Which question is most appropriate for the nurse to ask next? a. "So what do you think is causing this persistent cough?" b. "Have you tried taking sleeping pills to help you sleep?" c. "Did you think this will just go away on its own?" d. "What other symptoms have you noticed related to this cough?"

d. "What other symptoms have you noticed related to this cough?" A The answer to the question "So what do you think is causing this persistent cough?" is a guess by the patient and does not provide useful data. B "Have you tried taking sleeping pills to help you sleep?" does not focus on the cough, which is what is disturbing the patient's sleep. C "Did you think this will just go away on its own?" does not provide useful data and criticizes the patient's lack of action. D "What other symptoms have you noticed related to this cough?" is part of a symptom analysis to provide more data.

Which question is an example of an open-ended question? a. "Have you experienced this pain before?" b. "Do you have someone to help you at home?" c. "How many times a day do you use your inhaler?" d. "What were you doing when you felt the pain?"

d. "What were you doing when you felt the pain?" A "Have you experienced this pain before?" is closed-ended, which can obtain a "yes" or "no" answer to the question without any additional data. B "Do you have someone to help you at home?" is closed-ended, which can obtain a "yes" or "no" answer to the question without any additional data. C "How many times a day do you use your inhaler?" is closed-ended, which can obtain an answer of a specific number without any additional data. D What were you doing when you felt the pain?" is a broadly-stated question that encourages a free-flowing, open response.

While taking a history, the nurse notices that the patient's family member repeats most of the questions to the patient in a loud voice. Based on this information, what finding does the nurse anticipate when assessing this patient's hearing using an audioscope? a. 5 dB hearing loss at all frequencies b. 10 dB hearing loss at all frequencies c. 20 dB hearing loss at all frequencies d. 40 dB hearing loss at all frequencies

d. 40 dB hearing loss at all frequencies A This decibel level is not tested by an audioscope . B A 10 dB loss in high frequencies results in difficulty hearing quiet sounds, such as a heartbeat. C A 20 dB loss in high frequencies results in difficulty hearing high-pitched consonants, such as a whisper. D A 40 dB loss in all frequencies causes moderate difficulty in hearing normal speech.

After assessment of each child, the nurse determines which child needs to be referred for further evaluation? a. A 4-year-old child with a predominantly nasal breathing pattern b. A 6-year-old child with a 1:2 anteroposterior-to-transverse-chest ratio c. A 7-year-old child with a predominantly thoracic breathing pattern d. A 9-year-old child with bronchovesicular breath sounds in peripheral lungs

d. A 9-year-old child with bronchovesicular breath sounds in peripheral lungs A At this age, children have a nasal and abdominal breathing pattern and do not develop a thoracic breathing pattern until 6 or 7 years of age. B By age 5 or 6, the rounded thorax of the child approximates the 1:2 ratio of anteroposterior to lateral diameter of the adult. C By age 6 or 7, the child's breathing pattern should change from primarily nasal and abdominal to thoracic in girls and abdominal in boys. D Although bronchovesicular breath sounds in peripheral lung areas are expected in a young child, vesicular sounds are expected at this age.

A female Korean patient accompanied by her husband and son comes to the emergency department (ED) complaining of abdominal pain. The patient speaks and understands Korean only. Which person is the appropriate choice for the nurse to use to get a history from this patient? a. The patient's husband who speaks Korean and English b. The patient's son who speaks Korean and English c. A male technician who works in the ED who speaks Korean and English d. A female interpreter who speaks Korean and English and is available by phone

d. A female interpreter who speaks Korean and English and is available by phone A The patient's husband who speaks Korean and English is not the best choice because he is a family member and may alter the meaning of what is said. B The patient's son who speaks Korean and English is not the best choice because he is a family member and may alter the meaning of what is said. C A male technician working in the ED who speaks Korean and English is not a good choice because the patient may feel uncomfortable giving a history to a stranger who is male. D A female interpreter who speaks Korean and English and is available by phone is the best choice because she can communicate with the patient and is the same gender as the patient.

A patient comes to the clinic for evaluation after a sinus infection. To evaluate the therapy, the nurse uses transillumination to assess the sinuses and notes which finding indicating recovery from a frontal sinus infection? a. The soft palate illuminates brightly when the light source is placed against the lateral nose. b. No illumination is noted when the light source is placed firmly against the lateral nose. c. A bright glow illuminates the hard palate when the light source is placed against each temporal bone. d. A reddish light is noted above the eyebrows when the light is placed against each supraorbital rim.

d. A reddish light is noted above the eyebrows when the light is placed against each supraorbital rim. A This describes incorrect technique for transillumination. B An absence of a glow during transillumination of the sinuses may indicate that the sinuses are congested. C This describes incorrect technique for transillumination. D Finding a reddish light above the eyebrows when the light is placed against each supraorbital rim is consistent with frontal sinuses free of infection.

A nurse examines a patient's auditory canal and tympanic membrane with an otoscope and observes which finding as normal? a. Clear fluid lining the auditory canal b. A firm tympanic membrane without fluctuation with puffs of air c. A small hole within the cone of light d. A shiny, translucent tympanic membrane

d. A shiny, translucent tympanic membrane A Clear fluid or bloody drainage following a head injury may indicate a basilar skull fracture. B An expected response is that the tympanic membrane slightly fluctuates with puffs of air. C A cone of light is expected, but a hole indicates perforation. D A shiny, translucent tympanic membrane is an expected finding.

A nurse tests a reflex on a 9-month-old infant's right foot by stroking the surface of the infant's foot, moving from the sole laterally up and across to the great toe. What is the expected response? a. Flexion of the right toes b. Extension of the right ankle c. Dorsiflexion of the right foot d. Fanning of the toes of the right foot

d. Fanning of the toes of the right foot A This is not an expected response from any reflex. B This is not an expected response from any reflex. C This is part of the clonus reflex test. D This is the expected response for the Babinski reflex.

A patient comes to the emergency department and tells the triage nurse that he is "having a heart attack." What is the nurse's top priority at this time? a. Determine the patient's personal data and insurance coverage. b. Ask the patient to take a seat in the waiting room until his name is called. c. Request that a nurse collect data for a comprehensive history. d. Ask a nurse to start a focused assessment of this patient now.

d. Ask a nurse to start a focused assessment of this patient now. A Personal data and insurance information will be obtained, but in this situation, these data can wait until after the patient is assessed. B Rather than asking the patient to wait, the nurse needs to begin data collection, such as vital signs, immediately to determine the patient's health status. C A comprehensive history is not indicated in this situation at this time. Some subjective data will be collected, such as allergies and medical history related to cardiovascular disease. D The nurse needs to begin an assessment as soon as possible that is focused on this patient's cardiovascular system. The type of health assessment performed by the nurse is also driven by patient need.

A nurse is interviewing a male patient who reports he has not had a tetanus immunization in about 15 years because he had a "bad reaction" to the last tetanus immunization. What is the most appropriate response by the nurse in this case? a. Notify the health care provider that this immunization cannot be given. b. Document that the patient is allergic to the tetanus vaccine. c. Give the vaccine after explaining that adverse reactions are rare. d. Ask the patient to describe the "bad reaction" to the vaccine in more detail.

d. Ask the patient to describe the "bad reaction" to the vaccine in more detail. A The immunization should not be eliminated at this time. Additional facts are needed to determine the type of reaction the patient experienced. B Documenting an allergy to the tetanus vaccine may be an error because there are insufficient data to make that determination at this time. C Giving the vaccine may be an error if the patient is allergic to the vaccine and additional data indicates that may be the case. D The nurse needs to collect more data about the reaction from the patient to determine the type of reaction experienced. The nurse is trying to assess the relationship between the "reaction" reported by the patient and an allergic reaction.

What technique does the nurse use to test the patellar deep tendon reflex? a. Using the end of the handle on the reflex hammer, the nurse strokes the lateral aspect of the sole of the patient's foot from heel to ball. b. Ask the patient to flex one knee to 90 degrees, while the nurse dorsiflexes the ankle and strikes the appropriate tendon on the foot with the flat end of the reflex hammer. c. Ask the patient to flex one knee to 45 degrees, while the nurse plantar flexes the ankle and strikes the appropriate tendon of the ankle with the pointed end of the reflex hammer. d. Ask the patient to flex one knee to 90 degrees, while the nurse strikes the appropriate tendon in the knee with the blunt end of the reflex hammer.

d. Ask the patient to flex one knee to 90 degrees, while the nurse strikes the appropriate tendon in the knee with the blunt end of the reflex hammer. A This is the technique for testing plantar flex or the Babinski reflex. B This is the technique for testing the Achilles deep tendon reflex. C This is not a correct technique for testing any reflex. D This is the technique for testing the patella deep tendon reflex.

Which technique should the nurse use to obtain more data about a patient's vague or ambiguous statement? a. Laughing and smiling during conversation b. Using phrases such as "Go on," and "Then?" c. Repeating what the patient has said, but using different words d. Asking the patient to explain a point

d. Asking the patient to explain a point A Laughing and smiling during conversation may show attentiveness during the interview, but does not help to clarify vague information. B Using phrases such as "Go on" and "Then?" encourages patients to continue talking, but does not help clarify. C Rephrasing what the patient has said is restatement. It confirms your interpretation of what they said, but does not encourage additional talking. D Asking the patient to explain a point is clarification, which is used to obtain more information about conflicting, vague, or ambiguous statements.

During a symptom analysis, a patient describes his productive cough and states his sputum is thick and yellow. Based on these data, the nurse suspects which factor as the cause of these symptoms? a. Virus b. Allergy c. Fungus d. Bacteria

d. Bacteria A A virus usually produces a nonproductive cough. B An allergy usually produces clear sputum. C A fungus usually produces few symptoms. The sputum used to diagnose the fungus is obtained from tracheal aspiration rather than the patient coughing up the sputum. D Bacteria usually produce sputum that is yellow or green in color.

What is the earliest and most sensitive indication of altered cerebral function? a. Unequal pupils b. Loss of deep tendon reflexes c. Paralysis on one side of the body d. Change in level of consciousness

d. Change in level of consciousness A Pupillary function represents function of the oculomotor cranial nerve and the midbrain. B Deep tendon reflexes represent function of the spinal cord and reflex arcs. C Movement represents function of the spinal cord and posterior frontal lobe. D Maintaining consciousness represents the functions of and communication between the frontal lobe and reticular activating system.

A nurse's presentation to patients on risk factors for macular degeneration includes which fact? a. The peak incidence is before 60 years of age. b. Women have a higher risk than men. c. Eating a low fat diet causes a vitamin A deficiency, which increases risk. d. Cigarette smokers have twice the risk as nonsmokers.

d. Cigarette smokers have twice the risk as nonsmokers. A Macular degeneration exists in 25% of those between ages 65 and 74 years, and 33% of those older than 75 years. B There are no differences between genders for macular degeneration. C A diet high in monosaturated, polyunsaturated, and vegetable fats increases risk. D Smoking is a risk factor for macular degeneration.

A nurse holds the patient's relaxed arm with the elbow flexed at a 90-degree angle, places a thumb over the appropriate tendon in the antecubital fossa, and strikes the thumb with the pointed end of the reflex hammer. What is the expected response for this deep tendon reflex? a. Flexion of the left elbow b. Pronation of the left forearm c. Supination of the left arm d. Extension of the left elbow

d. Extension of the left elbow A Pronation of the left forearm is a normal response for the brachioradialis deep tendon reflex. B Supination of the left arm is not a response of any deep tendon reflex. C Extension of the left elbow is the normal response of the triceps deep tendon reflex. D Flexion of the left elbow is a normal response for the biceps deep tendon reflex.

A patient reports having leg pain while walking that is relieved with rest. Based on these data, the nurse expects which finding on inspection and palpation of this patient? a. 1+ edema of the feet and ankles bilaterally b. The circumference of the right leg is larger than the left leg c. Patchy petechiae and purpura of the lower extremities d. Cool feet with capillary refill of toes greater than 3 seconds

d. Cool feet with capillary refill of toes greater than 3 seconds A Edema of 1+ of the feet and ankles bilaterally is an indication of a venous problem rather than an arterial problem. B When one leg is larger in circumference than the other, it could be due to lymphedema or a deep vein thrombosis. C Petechiae and purpura of the lower extremities indicate a bleeding problem, such as low platelets, rather than an arterial problem. D The pain while walking that is relieved by rest is called intermittent claudication and is an indication of arterial insufficiency. Cool feet and prolonged capillary refill also occur due to arterial insufficiency.

A nurse is completing a symptom analysis with a patient complaining of chest pain. When asked what makes the chest pain worse, the patient reports that coughing and sneezing increase the chest pain. Based on these data, what does the nurse suspect as the cause of this patient's chest pain? a. Stable angina b. Esophageal reflux disease c. Mitral valve prolapse d. Costochondritis

d. Costochondritis A Physical exertion, emotional stress, and cold worsen the chest pain associated with stable angina. B A spicy or acidic meal, alcohol, or lying supine may worsen the chest pain associated with esophageal reflux. C Only occasional position changes worsen the chest pain associated with mitral valve prolapse. D Coughing, deep breathing, laughing, and sneezing worsen the chest pain associated with costochondritis.

A nurse in the emergency department is assessing a patient with a moderate left pneumothorax. What does this nurse expect to find during the respiratory examination? a. Increased fremitus over the left chest b. Tracheal deviation to the left side c. Hyporesonant percussion tones over the left chest d. Distant to absent breath sounds over the left chest

d. Distant to absent breath sounds over the left chest A Increased fremitus occurs over lung consolidation as in lobar pneumonia or tumor. B If this patient had a tension pneumothorax, the trachea would deviate to the right. C Hyperresonant percussion tones are heard when the lung is overinflated as in emphysema. D The air separating the lung from the chest where the nurse is auscultating creates distant to absent breath sounds.

Using deep palpation of a patient's epigastrium, a nurse feels a rhythmic pulsation of the aorta. Based on this finding, what is the nurse's most appropriate response? a. Auscultate this area using the bell of the stethoscope. b. Percuss the area for tones. c. Ask the patient if there is pain in this area. d. Document this as a normal finding.

d. Document this as a normal finding. A Auscultating this area using the bell of the stethoscope is not necessary because this is a normal finding. Vascular sounds are usually not heard. B Percussing the area for tones is not necessary because this is a normal finding. C Asking the patient if there is pain in this area is not necessary because this is a normal finding. D Document this as a normal finding. The aorta is often palpable at the epigastrium.

A nurse expects which finding when assessing the abdomen of a patient who has been unable to void for 12 hours? a. Absent bowel sounds b. Hyperactive bowel sounds c. Tympanic tones over the lower abdomen d. Dull tones over the suprapubic area

d. Dull tones over the suprapubic area A Absent bowel sounds is incorrect because the bowel sounds would not be affected by a full bladder. B Hyperactive bowel sounds is incorrect because the bowel sounds would not be affected by a full bladder. C Tympanic tones over the lower abdomen is incorrect because tympany sound is created by gas in the abdomen. D Dull tones over the suprapubic area would be found. The urine in the bladder would create a dull sound when the bladder is percussed similar to the sounds when an abdominal mass is present.

While taking a history, a nurse learns that this patient experiences shortness of breath (dyspnea). If the cause of the dyspnea is a cardiovascular problem, the nurse expects which abnormal finding on examination? a. Flat jugular neck veins b. Red, shiny skin on the legs c. Weak, thready peripheral pulses d. Edema of the feet and ankles

d. Edema of the feet and ankles A Flat jugular veins indicate a fluid deficit, which is not associated with dyspnea. B Red, shiny skin on the legs is associated with peripheral arterial disease and is not associated with dyspnea. C Weak, thready peripheral pulses indicate fluid deficit, which is not associated with dyspnea. D This patient may have heart failure. Edema of the feet occurs with right ventricular heart failure. Dyspnea occurs with left ventricular heart failure.

Which activity illustrates the concept of primary prevention? a. Monthly breast self-examination b. Annual cervical (Papanicolaou test) examination c. Education about living with asthma d. Exercising three times a week

d. Exercising three times a week A Monthly breast self-examination is an example of secondary prevention and screening efforts to promote early detection of disease. B Annual cervical (Papanicolaou test) examination is an example of secondary prevention and screening efforts to promote early detection of disease. C Teaching a patient how to live with a chronic disease such as asthma is an example of tertiary prevention directed toward minimizing the disability from chronic disease and helping the patient maximize his or her health. D Exercising is an example of primary prevention that prevents disease from developing by maintaining a healthy lifestyle.

A nurse holds the patient's relaxed left arm, with elbow flexed at a 90-degree angle, in one hand. The nurse palpates and then strikes the appropriate tendon just above the elbow with either end of the reflex hammer. What is the expected response for this deep tendon reflex? a. Flexion of the left elbow b. Pronation of the left forearm c. Supination of the left arm d. Extension of the left elbow

d. Extension of the left elbow A Flexion of the left elbow would be a normal response for the biceps deep tendon reflex. B Pronation of the left forearm would be a normal response for the brachioradialis deep tendon reflex. C Supination of the left arm is not a response of any deep tendon reflex. D Extension of the left elbow is the normal response of the triceps deep tendon reflex.

A patient is admitted to the emergency department with a tracheal obstruction. What sound does the nurse expect to hear as this patient breathes? a. Dull sounds on percussion b. Soft, muffled rhonchi heard over the trachea c. Bubbling or rasping sounds heard over the trachea d. High-pitched sounds on inspiration and exhalation

d. High-pitched sounds on inspiration and exhalation A Dull sounds on percussion occur with pneumonia, pleural effusion, or atelectasis. B Soft, muffled rhonchi heard over the trachea is not a description of stridor. C Bubbling or rasping sounds heard over the trachea is not a description of stridor. D High-pitched sounds on inspiration and exhalation are consistent with stridor.

Which cranial nerve is assessed when a nurse asks a patient to stick out the tongue and move it side to side? a. Vagus nerve (CN X) b. Facial nerve (CN VII) c. Abducens nerve (CN VI) d. Hypoglossal nerve (CN XII)

d. Hypoglossal nerve (CN XII) A The vagus cranial nerve provides movement for voluntary muscles of phonation (guttural speech sounds) and swallowing. B The facial cranial nerve provides movement for facial expression muscles except the jaw, closes the eyes, and allows labial speech sounds (b, m, w, and rounded vowels). C The abducens cranial nerve provides for lateral eye movement. D The hypoglossal cranial nerve provides tongue movement for speech sound articulation (l, t, n) and swallowing.

What is the patient's expected response when the nurse is assessing graphesthesia? a. Lies supine and runs one heel along the opposite shin b. Identifies a familiar object placed in the hands c. Describes where a sensation of a vibrating tuning fork is felt d. Identifies a letter or number drawn in the hand

d. Identifies a letter or number drawn in the hand A This activity tests cerebellar function of the lower extremities. B This is a test of stereognosis that tests the function of the parietal lobe and sensory tracts. C This is a test of vibratory sense that tests sensory tracts. D This is a test of graphesthesia that assesses the parietal lobe and sensory tracts.

How is the first heart sound (S1) created? a. Pulmonic and tricuspid valves close. b. Mitral and aortic valves close. c. Aortic and pulmonic valves close. d. Mitral and tricuspid valves close.

d. Mitral and tricuspid valves close. A The pulmonic and tricuspid valves are the valves of the right side of the heart, and they do not close simultaneously in the cardiac cycle. B The mitral and aortic valves are the valves of the left side of the heart, and they do not close simultaneously in the cardiac cycle. C The aortic and pulmonic valves are the semilunar valves that create the second heart sound. D The first heart sound (S1) is made by the closing of the mitral (M1) and tricuspid (T1) valves.

Where does a nurse expect to hear bronchovesicular lung sounds in a healthy adult? a. In the lower lobes b. Over the trachea c. In the apices of the lungs d. Near the sternal border

d. Near the sternal border A Vesicular breath sounds are normally heard in the lower lobes. B Bronchial sounds are normally heard over the trachea. C Vesicular breath sounds are normally heard in the apices of the lungs. D Bronchovesicular breath sounds are normally heard over the central area of the anterior thorax around the sternal border.

What sound does a nurse expect to hear when using the bell of the stethoscope over the epigastric area of the abdomen of a healthy patient? a. Bowel sounds b. Venous hum c. Soft, low-pitched murmur d. No sounds

d. No sounds A Bowel sounds are heard with the diaphragm of the stethoscope. B Venous hum is not a normal finding. C Soft, low-pitched murmur is not a normal finding. D The bell is used to listen for vascular sounds and normally no vascular sounds are heard in the abdomen.

How does a nurse accurately palpate carotid pulses? a. Two fingers of each hand are placed firmly over the right and left temples at the same time. b. One finger is placed gently in the space between the biceps and triceps muscles. c. Two fingers are placed at the thumb side of the forearm at the wrist. d. One finger is placed along the right and then the left medial sternocleidomastoid muscle.

d. One finger is placed along the right and then the left medial sternocleidomastoid muscle. A Two fingers of each hand placed firmly over the right and left temples at the same time is the correct procedure for palpating the temporal pulse. B One finger placed gently in the space between the biceps and triceps muscles is the correct procedure for palpating the brachial pulse. C Two fingers placed at the thumb side of the forearm at the wrist is the correct procedure for palpating the radial pulse. D One finger placed along the right and then the left medial sternocleidomastoid muscle is the correct procedure for palpating the carotid pulses, checking each side separately.

During the history, a 65-year-old male patient reports smoking two packs of cigarettes a day for more than 40 years. With this knowledge, what does the nurse expect for during the examination of this patient's mouth? a. Cracks and erythema in the corners of the mouth b. Slightly rough papillae on the dorsal surface of the tongue c. Smooth or beefy, red-colored, edematous tongue d. Painless, nonhealing mouth ulcers

d. Painless, nonhealing mouth ulcers A This may be caused by vitamin B deficiencies. B These are an expected finding on the tongue. C This may be an indication of anemia. D This may indicate oral cancer.

Wearing gloves, the nurse grasps the patient's tongue with a gauze pad and palpates a small, firm nodule on the left side of the tongue. Based upon this finding, what is the nurse's appropriate response? a. Document that the patient's tongue is normal on palpation. b. Inspect the left submandibular salivary glands for redness. c. Ask the patient to move the tongue in all directions. d. Palpate cervical and submental lymph nodes for enlargement.

d. Palpate cervical and submental lymph nodes for enlargement. A The nodule is not an expected finding. B The salivary glands are not affected by a nodule of the tongue. C This assesses the hypoglossal cranial nerve or movement of the tongue, which is not related to the nodule found. D The nodules may indicate a malignancy of the tongue, which may also cause enlarged cervical or submental lymph nodes.

A nurse correlates a patient's altered stereognosis with a neurologic dysfunction in which part of the nervous system? a. Midbrain or pons b. Temporal lobe or ascending nerve tracts c. Frontal lobe or motor nerve tracts d. Parietal lobe or sensory nerve tracts

d. Parietal lobe or sensory nerve tracts A Sensory and motor tracts travel though the midbrain and pons, but they are not tested with stereognosis. B Ascending tracts carry sensory data, but the temporal lobe provides functions of hearing rather than perception of touch. C Motor tracts carry impulses for movement and they exit from the frontal lobe, which also helps to maintain consciousness. D A parietal lobe or sensory nerve tract dysfunction prevents a patient from identifying a familiar object by touch, which is a definition of stereognosis.

Which patient has the lowest risk for colon cancer? a. Patient A is 50 years old, is obese, and has type 2 diabetes mellitus. b. Patient B is 60 years old, has alcoholism, and smokes a pack of cigarettes daily. c. Patient C is 55 years old, has ulcerative colitis, and inflammatory bowel disease. d. Patient D is 45 years old and has diverticulosis.

d. Patient D is 45 years old and has diverticulosis. A Patient A has three risk factors for colon cancer. B Patient B has three risk factors for colon cancer. C Patient C has two risk factors for colon cancer. D Patient D has the lowest risk of colon cancer. Ninety percent of colon cancers occur in adults older than 50 years of age. Although this patient does have a disorder of the colon, it is not linked to an increased risk of colon cancer.

Which finding warrants a referral for additional evaluation? a. Earlobes hanging freely from the base of the pinna b. Ears having painless nodules less than 1 cm in diameter at the helix c. Ears measuring 8 cm in length d. Pinna is 20 degrees lower than the outer canthus of the eye

d. Pinna is 20 degrees lower than the outer canthus of the eye A Earlobes hanging freely from the base of the pinna is an expected finding. B This is called a Darwin tubercle. It is a normal deviation and may be noted at the helix of the ear. C A length of 8 cm is an expected finding. D The pinna of the ear should align directly with the outer canthus of the eye and be angled no more than 10 degrees from a vertical position.

How does the nurse perform a Rinne test of hearing function? a. Whispers several words to the patient and requests that the patient repeat the words heard b. Places a vibrating tuning fork in the middle of the head and asks the patient if the sound is heard the same in both ears or if it is louder in one ear than the other c. Places a set of headphones over both ears, plays several tones, and asks the patient to identify the sounds d. Places a vibrating tuning fork on the mastoid process until the patient no longer hears it, and then moves it in front of the ear until the patient no longer hears it

d. Places a vibrating tuning fork on the mastoid process until the patient no longer hears it, and then moves it in front of the ear until the patient no longer hears it A This technique describes the whisper test. B This technique describes the Weber test. C This technique describes the use of an audiometer. D This technique describes the Rinne test.

A nurse dorsiflexes a patient's right ankle 90 degrees and then uses a reflex hammer to strike the appropriate tendon. What is the expected response for this deep tendon reflex? a. Extension of the right lower leg b. Plantar flexion of the right toes c. Dorsiflexion of the right foot d. Plantar flexion of the right foot

d. Plantar flexion of the right foot A This is the expected response for the patellar deep tendon reflex. B This is the expected response for the plantar reflex (Babinski). C This is an incorrect response because the nurse is holding the patient's foot in dorsiflexion, therefore dorsiflexion would not be an expected response. D Plantar flexion is the expected response of the Achilles deep tendon reflex.

The patient reports right lower quadrant (RLQ) pain that is worse with coughing. Based on the patient's history, the nurse suspects appendicitis. What additional examination technique does the nurse perform to confirm this suspicion? a. Placing the hand over the lower right thigh and asking the patient to flex the knee while pushing down on the knee to resist it and noting if the patient complains of pain b. Palpating deeply a point of the abdomen, located halfway between the umbilicus and the left anterior iliac crest c. Asking the patient to flex the right hip and knee to 90 degrees, then abducting the leg and noting if the patient complains of pain d. Pressing down in an area away from the RLQ at a 90-degree angle to the abdomen, then releasing the fingers quickly and noting any complaint of pain

d. Pressing down in an area away from the RLQ at a 90-degree angle to the abdomen, then releasing the fingers quickly and noting any complaint of pain A This is an incorrect description of the iliopsoas muscle test. B This is an incorrect description of the testing for McBurney point. McBurney point is located to the right of the umbilicus. C This is an incorrect description of the obturator muscle test. D This describes rebound tenderness, which is performed to detect peritoneal inflammation.

A patient comes to the ambulatory surgery center for an elective procedure this morning. While giving the admission history, the patient states she is allergic to latex. What is the most appropriate response by the nurse at this time? a. Removing all latex products from the patient's room b. Using powdered gloves when providing care to this patient c. Informing the surgeon that the patient has type I hypersensitivity to latex d. Questioning the patient about symptoms experienced in the past with latex

d. Questioning the patient about symptoms experienced in the past with latex A Removing all latex products from the patient's room is unnecessary at this time because the latex allergy has not been confirmed. B Using powdered gloves when providing care to this patient is unnecessary at this time because the latex allergy has not been confirmed. C Informing the surgeon that the patient has type I hypersensitivity to latex is unnecessary at this time because the latex allergy has not been confirmed. D Questioning the patient about symptoms experienced in the past with latex is the appropriate response. When patients indicate an allergy to a medication or substance, ask them to describe what happens with exposure to determine whether the reaction is a side effect or an allergic reaction.

When inspecting a patient's abdomen, which finding does the nurse note as normal? a. Engorgement of veins around the umbilicus b. Sudden bulge at the umbilicus when coughing c. Visible peristalsis in all quadrants d. Silver-white striae extending from the umbilicus

d. Silver-white striae extending from the umbilicus A Engorgement of veins around the umbilicus is an abnormal finding. B Sudden bulge at the umbilicus when coughing is an abnormal finding and may indicate a hernia. C Visible peristalsis in all quadrants is an abnormal finding. D Silver-white striae extending from the umbilicus is a normal finding, particularly in women who have been pregnant or in any adult who has lost weight after having an obese abdomen.

On assessment of an infant's abdomen, the nurse notes which finding as normal? a. Easily palpable spleen b. Flat to slightly concave abdominal contour c. Lower liver border 2 inches below the costal margin d. Small protrusion between the rectus muscles when crying

d. Small protrusion between the rectus muscles when crying A The spleen is generally not palpable, although the tip may be felt in the left upper quadrant (far left costal margin). B Inspecting the abdomen of a healthy infant finds a symmetric, soft, and round abdomen with a slight protrusion. C The edge of the infant's liver should be 1 to 2 cm below the right rib cage (costal margin). D Diastasis swelling and a gap between the rectus muscles may be noted as an expect finding during crying.

. When palpating the right lobe of the patient's thyroid gland using the anterior approach, the nurse feels the tissue between which two structures? a. Sternocleidomastoid and the trapezius muscles b. Trapezius muscle and the trachea c. Cricoid process and the trachea d. Sternocleidomastoid muscle and the trachea

d. Sternocleidomastoid muscle and the trachea A This is not the correct location for palpating the thyroid gland using the anterior approach. B This is not the correct location for palpating the thyroid gland using the anterior approach. C This is not the correct location for palpating the thyroid gland using the anterior approach. D This is the correct location for palpating the thyroid gland using the anterior approach.

When examining lymph nodes in a 7-year-old child, the nurse records which finding as abnormal? a. "Shotty" nodes in the cervical areas b. Palpable submandibular nodes c. Nodes that are tender 1 week after a tetanus vaccination d. Tender, fixed nodes greater than 1 cm

d. Tender, fixed nodes greater than 1 cm A The term "shotty" may be used to describe small, firm, and mobile nodes, usually occurring as a normal variation in children. B Cervical and submandibular nodal enlargements are frequent in older children. C Enlarged, tender nodes may occur after immunizations or upper respiratory infection. D Abnormal findings are tender, fixed nodes greater than 1 cm.

After obtaining a history from the parents and inspecting the skin, the nurse determines which child needs further evaluation? a. The child who has a 1-cm red spot on the back of the neck, a fever of 100° F, and clear nasal drainage. b. The child who has a 2-cm slightly-raised, reddened area with a sharp demarcation line on the back of the neck. c. The child has a 2-cm abrasion on the right knee, a 3-cm abrasion on the left knee, and scrapes on both palms. d. The child who has several flat, bluish discolorations of the skin on the abdomen and back from 2 to 6 cm.

d. The child who has several flat, bluish discolorations of the skin on the abdomen and back from 2 to 6 cm. A This child has a stork bite birthmark on the back of the neck and an upper respiratory allergy or viral infection. B This is a strawberry hemangioma, a birthmark that disappears by 5 years of age. C This child probably fell down while running, skinned both knees, and tried to break the fall with the hands. D Bruising in unusual areas (such as upper arms, back, buttocks, and abdomen) or multiple bruises found at different stages of healing should be further investigated to determine if there is abuse.

The nurse documents which information in the patient's history? a. The patient's skin feels warm to the touch. b. The patient is scratching his arm. c. The patient's temperature is 100° F. d. The patient complains of itching.

d. The patient complains of itching. A The patient's warm skin is objective information gathered by the nurse through palpation, is also a sign, and is documented in the physical examination. B The patient's scratching is objective information gathered by the nurse through observation, is also a sign, and is documented in the physical examination. C The patient's elevated temperature is objective information gathered by the nurse through measurement, is also a sign, and is documented in the physical examination. D A patient's complaint of itching is subjective information, which means it is a symptom and is documented in the history.

How does a nurse recognize a patient's mydriasis? a. The lens of each of the patient's eyes is opaque. b. There is involuntary rhythmical, horizontal movement of the patient's eyes. c. There is a white opaque ring encircling the patient's limbus. d. The patient's pupils are 7 mm and do not constrict.

d. The patient's pupils are 7 mm and do not constrict. A An opaque lens is an abnormality that occurs when cataracts are present. B An involuntary rhythmical, horizontal movement of the patient's eyes is a description of nystagmus. C A white opaque ring encircling the patient's limbus is a description of corneal arcus seen in patients older than 60 years of age. D Mydriasis is pupil size greater than 6 mm and the pupil fails to constrict.

How does a nurse recognize normal accommodation? a. The patient has peripheral vision of 90 degrees left and right. b. The patient's eyes move up and down, side to side, and obliquely. c. The right pupil constricts when a light is shown in the left pupil. d. The patient's pupils dilate when looking toward a distant object.

d. The patient's pupils dilate when looking toward a distant object. A Normally a patient has 90 degrees peripheral vision temporally, but only 60 degrees nasally. B This is an expected finding, but is not a test for accommodation. It is a test of extraocular muscle function in the six cardinal fields of gaze. C This is an expected finding for consensual reaction, rather than accommodation. D This is an indication of accommodation.

For which person is an episodic or follow-up assessment indicated? a. The person who had abdominal surgery yesterday b. The person who is unaware of his high serum glucose levels c. The person who is being admitted to a long-term care facility d. The person who is beginning rehabilitation after a knee replacement

d. The person who is beginning rehabilitation after a knee replacement A A shift assessment is most appropriate for the person who is recovering in the hospital from surgery. B A screening assessment is performed for the purpose of disease detection, in this case diabetes mellitus. C A comprehensive assessment is performed during admission to a facility to obtain a detailed history and complete physical examination. D An episodic or follow-up assessment is performed after the knee replacement to evaluate the outcome of the procedure.

What technique does a nurse use to inspect the ear canal of a 1-year-old child? a. Uses a light source without a speculum to minimize any trauma to the ear canal b. Places the child in an upright position with the head flexed slightly downward c. Applies gentle traction to the lower portion of the ear and pulls upward and laterally d. Uses an assistant to hold the child's arms down and keep the child's head turned to one side

d. Uses an assistant to hold the child's arms down and keep the child's head turned to one side A A nurse uses the largest speculum that fits comfortably into the child's ear canal. B Because the nurse must have both hands free to hold the ear and maneuver the otoscope, another individual must act as a "holder." The child's head is turned to one side. C The correct technique is to grasp the lower portion of the pinna and apply gentle traction down and slightly backward (as opposed to pulling the pinna up and back for the adult). This maneuver straightens the canal of the ear. D Instruct the holder to secure the infant's arms at the sides with one hand, and turn and hold the infant's head to one side with the other hand.

A 4-year-old child has had a tonsillectomy and the nurse is preparing to ask him about his pain. Which technique is the most appropriate method for pain assessment for this patient? a. Asking him if the pain hurts "a little or a lot" b. Asking him to rate the pain on a scale of 0 to 10 c. Using the visual analog scale to rate the pain d. Using the Wong/Baker FACES rating scale

d. Using the Wong/Baker FACES rating scale A Using adjectives such as these is not reliable to assess pain in patients of any age. B This scale is appropriate for adolescents and adults, but a child cannot understand the concept of using numbers to rate pain. C This type of scale is appropriate for adults, but a child cannot understand the concept of using a straight line to rate pain. D This tool is appropriate for children who can point to the child's face that best represents how they are feeling.

During a musculoskeletal assessment of a school-age child, a nurse documents which finding as expected? a. A positive Trendelenburg sign on one side b. Lumbar lordosis, especially in African American children c. Varus rotation when the knees are greater than 1 inch apart d. Valgus rotation of less than 1 inch with the knees touching

d. Valgus rotation of less than 1 inch with the knees touching A Trendelenburg sign (or gait) tests for hip dysplasia and the function of the gluteus medius muscle. B Lordosis is seen more frequently in African American children but should not be seen in children older than 6 years of age. C Varus rotation (medial malleolus touching with knees greater than 1 inch [2.5 cm] apart) requires further evaluation for tibial torsion; it may be normal through 18 to 24 months of age. D Valgus rotation (medial malleolus greater than 1 inch [2.5 cm] apart with knees touching) is normal in 2- to 3.5-year-old children and may be present up to 12 years old.


Conjuntos de estudio relacionados

CISSP Practice Test Combined_good

View Set

Chapter 4 - The constitution of the United States of America

View Set

Speak Out Starter Unit 4 (2) Days, time phrases

View Set

Post Traumatic Stress Disorder/Syndrome

View Set

Unit 7 - Investment Analysis and Strategies

View Set

CME Workbook 1, Lesson 5, Page 44, Exercise 3 - answer

View Set

NUR 230 exam 1 clicker questions

View Set

Gov- Unit 3: Quiz 1 - The American Party System

View Set

Week Three: Cell Structure and Protists

View Set